10.07.2015 Views

Revista (format .pdf, 2.3 MB) - RECREAÅ¢II MATEMATICE

Revista (format .pdf, 2.3 MB) - RECREAÅ¢II MATEMATICE

Revista (format .pdf, 2.3 MB) - RECREAÅ¢II MATEMATICE

SHOW MORE
SHOW LESS
  • No tags were found...

Create successful ePaper yourself

Turn your PDF publications into a flip-book with our unique Google optimized e-Paper software.

Anul XIV, Nr. 1Ianuarie – Iunie 2012RECREAŢ II<strong>MATEMATICE</strong>REVISTĂ DE MATEMATICĂ PENTRU ELEVI Ş I PROFESORIe iπ =−1Asociaţ ia “Recreaţ ii Matematice”IAŞ I - 2012


Semnificaţia formulei de pe copertă:iπÎntr-o formă concisă, formula e = −1leagă cele patru ramuri fundamentaleale matematicii:ARITMETICA reprezentată de 1GEOMETRIAreprezentată de πALGEBRAreprezentată de iANALIZA MATEMATICĂreprezentată de eRedacţia revistei :Petru ASAFTEI, Temistocle BÎRSAN, Dan BRÂNZEI, Alexandru CĂRĂUŞU, ConstantinCHIRILĂ, Eugenia COHAL, Adrian CORDUNEANU, Mihai CRĂCIUN (Paşcani),Paraschiva GALIA, Paul GEORGESCU, Gheorghe ILIE, Gheorghe IUREA, GabrielMÎRŞANU, Gabriel POPA, Dan POPESCU (Suceava), Florin POPOVICI (Braşov), MariaRACU, Neculai ROMAN (Mirceşti), Ioan SĂCĂLEANU (Hârlău), Ioan ŞERDEAN(Orăştie), Dan TIBA (Bucureşti), Marian TETIVA (Bârlad), Lucian TUŢESCU (Craiova),Adrian ZANOSCHI, Titu ZVONARU (Comăneşti)Materialele se trimit la una dintre adresele: t_birsan@yahoo.com, profgpopa@yahoo.co.ukPagina web a revistei Recreații Matematice : http://www.recreatiimatematice.roCOPYRIGHT © 2011, ASOCIAŢIA “RECREAŢII <strong>MATEMATICE</strong>”Toate drepturile aparţin Asociaţiei “Recreaţii Matematice”. Reproducerea integrală sauparţială a textului sau a ilustraţiilor din această revistă este posibilă numai cu acordul prealabilscris al acesteia. Se consideră că autorii materialelor trimise redacţiei revistei sunt, în modimplicit, de acord cu publicarea lor, îşi asumă responsabilitatea conţinutului lor şi cedeazăAsociaţiei “Recreaţii Matematice” dreptul de proprietate intelectuală asupra acestora.TIPĂRITĂ LA BLUE SIM&Co IAŞIBd. Carol I, nr. 3-5Tel. 0332 111021, 0721 571705E-mail: simonaslf@yahoo.comISSN 1582 - 1765


Anul XIV, Nr. 1 Ianuarie – Iunie 2012RECREAŢ II<strong>MATEMATICE</strong>REVISTĂ DE MATEMATICĂ PENTRU ELEVI Ş I PROFESORIe iπ =−1Revistă cu apariţie semestrialăEDITURA „RECREAŢII <strong>MATEMATICE</strong>”IAŞI - 2012


Évariste Galois (1811-1832)În octombrie 2011 a fost celebrat bicentenarulnaşterii lui Évariste Galois, genial matematicianale cărui idei şi contribuţii au revoluţionat matematicasecolului al XIX-lea. În scurta sa viaţă depuţin peste 20 de ani, Évariste Galois a lăsat posterităţiio operă – restrânsă ca întindere la numai60 de pagini – în care este fundamentată noţiuneade grup pentru a caracteriza ecuaţiile algebrice rezolubileprin radicali.S-a născut la 25 oct. 1811 în Bourg-le-Reine,lângă Paris, fiind al doilea printre cei trei copii aifamiliei Galois. Tatăl său, republican înflăcărat,cu o bună reputaţie printre concetăţenii săi, a fostales în 1815 primar al micului orăşel. Mama sa, ceprovenea dintr-o familie de jurişti, a fost singurulprofesor al lui Évariste până la vârsta de 12 ani.În octombrie 1823 părinţii îl trimit la ColegiulRegal Louis-le-Grand din Paris. Aici este apreciat ca un elev inteligent, original,bizar şi nu suficient de metodic. Repetă al treilea an şcolar, deoarece rezultatele salela retorică nu sunt suficiente. La începutul anului 1827 schimbă profilul, trece la primaclasă cu profil matematic, unde descoperă şi este absorbit de matematică, neglijândcelelalte cursuri. Legenda spune că a parcurs manualul Elemente de geometrie al luiA.-M. Legendre în două zile. Este atras de memoriile originale ale lui J. Lagrange:Rezolvarea ecuaţiilor algebrice şi Teoria funcţiilor analitice şi este gata pentru studiiproprii. Se pregăteşte singur pentru a intra în celebra École Polytechnique, dar eşueazăîn vara anului 1828. În aprilie 1829 publică prima sa lucrare despre fracţiile continue înAnnales de mathématiques pures et appliquées (Annales de Gergonne). Apoi, în lunilemai şi iunie ale anului 1829, A.-L. Cauchy prezintă la Academia de Ştiinţe (Paris)primul său memoriu cuprinzând cercetările făcute asupra ecuaţiilor polinomiale rezolubile;dar, memoriul şi comentariile lui Cauchy au fost pierdute.Anul 1829 este marcat de mai multe evenimente importante în viaţa lui ÉvaristeGalois: sinucidera tatălui său, consecinţă a unor intrigi murdare, urmată, după douăsăptămâni, de al doilea eşec la École Polytechnique. Este admis, însă, la ÉcolePréparatoire (mai târziu numită École Normale Supérieure), care era tutelată deColegiul Louis-le-Grand; aici leagă o trainică prietenie cu Auguste Chevalier, careva contribui la recunoaşterea postumă a meritelor sale.Încurajat de Cauchy, depune în februarie 1930 la Academia de Ştiinţe o nouăversiune a studiilor sale: Mémoire sur les conditions de résolubilité des équationsradicaux în vederea concursului pentru marele premiu în matematici din iunie 1830;totodată, publică în Bulletin de Férussac o notă de prezentare a acestui memoriu.Tot în acest buletin ştiinţific, Galois publică în luna iunie alte două lucrări.La 28 iunie, Premiul Academiei este decernat matematicienilor N. Abel, cu titlupostum, şi C. G. J. Jacobi. Évariste Galois avea o mare admiraţie faţă de aceştia,1


dar s-a mirat că memoriul său n-a fost citat. A aflat că J. Fourier, secretarul deatunci al Academiei, la care a fost trimis memoriul, a decedat în luna mai; cummemoriul s-a pierdut, nu a mai fost luat în consideraţie.Pierderea celor două memorii, moartea tatălui, cele două eşecuri la École Polytechniqueau fost pentru tânărul Galois o mare decepţie, cu urmări în comportamentulsău. Critică sistemul şcolar şi militează pentru reformarea predării ştiinţelor. Pe dealtă parte, este un republican înflăcărat, gata să se sacrifice pentru idealurile republicane.Toate acestea au loc pe fondul unor evenimente politice agitate prin care treceaFranţa în acea perioadă, care culminează cu revoluţia din 1830 (Trois Glorieuse).Évariste Galois participă cu pasiune la evenimentele acelor zile, atrăgând asupra samăsuri coercitive din partea autorităţilor: este exmatriculat din şcoală în decembrie1830, este arestat şi trimis la închisoarea Sainte-Pélagie în două rânduri în cursulanului 1831.A.-L. Cauchy, fidel Bourbonilor, însoţeşte în exil pe Carol al X-lea. Raportor laAcademie este acum S.D. Poisson, care-l invită pe Galois să înainteze acestui fordin nou memoriul său asupra ecuaţiilor. Memoriul este prezentat pe 17 ian. 1831, iarrăspunsul lui Poisson se lasă mult aşteptat şi îi cauzează lui Galois, aflat în închisoreaSainte-Pélagie, o nouă dezamăgire: memoriul este respins pe motivul că argumentelenu sunt suficient de clare şi dezvoltate pentru a se putea face o idee asupra lui şi sesugerează o redactare mai completă.Eliberat din închisoare în aprilie 1832, Évariste Galois este antrenat într-un duelcu pistoale şi moare, în urma rănii primite, la 31 mai 1832. Asupra circumstanţelornefastului duel părerile sunt împărţite. Este înhumat pe 2 iunie în groapa comună acimitirului Montparnasse. Astfel ia sfârşit viaţa lui Évariste Galois, geniu nefericitşi nenorocos, care nu a cunoscut recunoaşterea marilor sale merite ştiinţifice.În ajunul duelului, Évariste Galois a scris trei scrisori patetice. Cea adresatădevotatului său prieten, Auguste Chevalier, este considerată ca testamentul său dematematician: aminteşte aici de descoperirile sale în teoria ecuaţiilor şi în privinţafuncţiilor definite prin integrale; schiţează apoi ceea ce astăzi numim teoria lui Galois,adăugînd câteva teoreme noi la conţinutul manuscrisului înaintat Academiei; cereprietenului ,,să roage public pe Jacobi sau pe Gauss să-şi spună părerea nu asupraadevărului, ci asupra importanţei teoremelor”.Alfred Galois, fratele mai mic al lui Évariste, şi Auguste Chevalier au copiatlucrările şi le-au trimis lui Jacobi, Gauss şi altora. În 1846, J. Liouville publicătoate lucrările lui Galois în revista sa Journal de Mathématiques pures et appliquées(Journal de Liouville), după ce în 1843 el informase Academia de Ştiinţe despre rezultateleinteresante conţinute în acestea.De originalitatea, profunzimea şi măreţia gândirii lui Évariste Galois, de putereade cuprindere a conceptului avansat de el, de magia unificatoare a teoriei grupurilor,lumea matematică îşi va da seama ceva mai târziu şi-i va recunoaşte locul cuvenitprintre matematicienii a căror operă a determinat schimbări semnificative în matematicăşi ştiinţă.2Redacţia revistei


Profesorul Adolf Haimovici - centenarul naşterii saleSe împlinesc 100 de ani de când a văzut luminazilei Adolf Haimovici, cel care în cursulvieţii se va distinge ca o personalitate proeminentăa matematicii ieşene şi româneşti, un dascăl ataşatînvăţământului românesc, un om respectat şi iubitde toţi cei care l-au cunoscut.Născut la Iaşi, la 1 octombrie 1912, avea să-şilege viaţa, activitatea şi destinul de acest oraş; aîndrăgit Iaşul şi nu ezita, atunci când avea prilejul,să-şi exprime ataşamentul faţă de el.Clasele primare le-a urmat la Şcoala primară,,Gh. Asachi”, apoi studiile secundare la LiceulNaţional (actualul Colegiu ,,Naţional”). În perioada1930-1934 urmează cursurile Facultăţii deŞtiinţe de la Universitatea din Iaşi încheind cuo licenţă în matematică şi una în ştiinţe fizicomatematice.În anul 1938 îşi susţine teza de doctoratcu subiectul Direcţii concurente şi direcţii paralele pe o varietate a unui spaţiuconform sub conducerea prof. Al. Myller. Până în 1945, cînd este încadrat ca asistentuniversitar la Facultatea de Ştiinţe, activează ca profesor secundar la diverse liceedin Bacău şi Iaşi. Urcă repede treptele ierarhiei universitare, în anul 1949 fiind profesortitular şi şef al Catedrei de ecuaţii diferenţiale. Până la momentul pensionării, înanul 1981, a predat numeroase şi foarte variate cursuri: ecuaţii diferenţiale, ecuaţiilefizicii matematice, teoria funcţiilor de o variabilă complexă, capitole speciale de teoriaoperatorilor etc.Profesorul Adolf Haimovici era unul dintre profesorii cei mai apropiaţi destudenţi. Cursurile predate erau la nivelul de înţelegere a studenţilor şi aveau claritate;deseori îşi întrerupea expunerea pentru a pune întrebări studenţilor sau pentrua indica punctele-cheie ale teoriei expuse sau ale unei demonstraţii. A militat înmod constant pentru primenirea programelor şi introducerea de cursuri noi. Pentrustudenţi, dar şi pentru viitorii specialişti, a publicat cursuri fundamentale ca:Ecuaţii diferenţiale şi integrale (1965), Ecuaţiile fizicii matematice şi elemente decalcul variaţional (1966). Este coautor la tratatul Geometrie analitică şi diferenţială(1951), lucrare ce a primit Premiul de Stat. Monografia Ecuaţii diferenţiale cu funcţiide mulţime ca necunoscută (1976) este o premieră în domeniul respectiv.Contribuţia cea mai importantă a prof. dr. doc. Adolf Haimovici esteopera sa ştiinţifică, ce cuprinde peste 140 memorii şi articole publicate în ţară saustrăinătate. A abordat subiecte şi teme din domenii foarte diverse ale matematicii,care pot fi grupate în trei direcţii principale de cercetare: 1) geometrie diferenţială(geometrie pe spaţii conforme, spaţii cu conexiune afină, spaţii cu conexiune neliniarăetc.); 2) ecuaţii funcţionale (diferenţiale, cu derivate parţiale, integro-diferenţiale);3) matematici aplicate (biomatematică, termodinamică, economie etc.). Cu toatăaceastă diversitate de preocupări în acord cu grija sa permanentă de abordare a unor3


domenii noi şi actuale, prof. Adolf Haimovici nu a părăsit niciodată cercetările degeometrie, fiind un membru important al şcolii de geometrie din Iaşi.Prof. dr. doc. Adolf Haimovici a sprijinit şi îndrumat un mare număr de tinericercetători în calitate de conducător de doctorat: peste 30 de teze de doctorat s-aufinalizat sub îndrumarea sa.A fost redactor responsabil al revistei Analele Ştiinţifice ale Universităţii,,Al. I. Cuza” din Iaşi, secţia Matematică, contribuind la ridicarea nivelului ştiinţifical acesteia. De asemenea, a fost director al Seminarului Matematic ,,Al. Myller” din1952 şi până în 1992. În această lungă perioadă de timp, prof. Adolf Haimovicia fost preocupat la început de restabilirea ordinii în biblioteca adusă din refugiu şiafectată de război, iar apoi de completarea colecţiilor de reviste, sporirea fondului decarte şi mărirea numărului de schimburi între Analele Ştiinţifice şi alte reviste. Înbibliotecă s-a statornicit un climat optim de studiu şi schimb de idei, iar SeminarulMatematic ,,Al. Myller” a devenit un adevărat institut de cercetare.Prof. dr. doc. Adolf Haimovici a fost o personalitate complexă; enumerămaici câteva din multele activităţi în care a fost implicat: conducător al Seminarului deecuaţii funcţionale şi analiză numerică, membru al Consiliului profesoral al Facultăţiide matematică şi Senatului Universităţii, iniţiator al unor noi direcţii de cercetare şi almodernizării unora tradiţionale, stăruinţa neobosită de promovare a învăţământuluiin<strong>format</strong>icii în facultate, crearea Laboratorului de in<strong>format</strong>ică al facultăţii, sprijinireaînfiinţării Centrului de calcul al Universităţii ş.a. A fost o prezenţă activă în comunitateamatematicienilor: participant la numeroase congrese şi colocvii din ţară şiinternaţionale, profesor invitat să ţină conferinţe în aproape toate ţările europene.Pentru cititorii revistei Recreaţii Matematice nu putem trece cu vederea o altăcoordonată a activităţii prof. Adolf Haimovici : a fost un animator al modernizăriiînvăţământului preuniversitar, în special al celui matematic. Încă de la înfiinţarea filialeiIaşi a Societăţii de Ştiinţe Matematice din România, în anul 1950, a făcut partedin conducerea acesteia, ca vicepreşedinte mai întâi – preşedinte fiind Al. Myller –,apoi ca preşedinte din 1956 şi până spre ultimii ani ai vieţii. Din bogata activitatedesfăşurată amintim : peste 60 de articole adresate elevilor şi profesorilor, multepublicate în Gazeta Matematică , conferinţe la cursurile de vară ale profesorilor saula cercurile de elevi, cărţi de matematică elementară dedicate elevilor şi profesorilor(Grupuri de transformări. Introducere elementară (1963)), manuale şcolare (Elementede geometrie a planului (1968), Elemente de geometrie în spaţiu ) etc.La facultate era numit simplu: Profesorul. Pentru că Profesorul putea fi găsitîn orice zi, pentru că era implicat în aproape toate activităţile facultăţii, pentru căera disponibil să asculte şi să ajute pe oricine. Nu puţini au găsit sprijin din parteaProfesorului în rezolvarea unor probleme strict personale.La 1 ianuarie 1993 a încetat din viaţă în plină activitate creatoare !Adolf Haimovici rămâne în conştiinţa posterităţii ca un strălucit om de ştiinţă,un eminent dascăl şi un model de comportament, care, cu talent şi muncă, cu dăruireşi pasiune, a contribuit în mod plenar la progresul ştiinţei şi învăţământului românesc.4Prof.dr. Temistocle BÎRSAN


O teoremă de reprezentare (II)Marian TETIVA 1Abstract. In this paper some (in general well-known) results on complete sequences are exposed,with applications to Erdős-Suranyi sequences. We start from the particular problem solved in anolder paper [10] (and from other similar problems) having the purpose to remind the readers thesebeautiful results.Keywords: complete sequence, Erdös-Suranyi sequence.MSC 2000: 11A67.1. Introducere: şiruri complete. Înainte de a vedea cum se poate generalizaenunţul despre care am vorbit în prima parte a acestei lucrări [10], avem nevoie deun rezultat privind şirurile complete. Un şir de numere întregi pozitive se numeştecomplet dacă orice număr întreg pozitiv se poate reprezenta ca suma unor termenidistincţi ai acestui şir. Rezultatul despre care vorbim se poate găsi în multe surse[2,3,4,6,9] şi se enunţă astfel:Propoziţia 1. Fie (a n ) un şir de numere întregi pozitive astfel încât a 1 = 1 şia n+1 ≤ a 1 + · · · + a n + 1 pentru orice n ≥ 1. Atunci (a n ) este complet.Mai precis, dacă notăm S n = a 1 +· · ·+a n pentru orice n ≥ 1, atunci se poate arătacă orice număr natural din intervalul [1, S n ] are o reprezentare ca sumă de termenidistincţi ai şirului (a n ) din mulţimea {a 1 , . . . , a n }.Considerând enunţul în această formă demonstraţia se face uşor prin inducţiedupă n; o lăsăm cititorului ca exerciţiu, cu atât mai mult cu cât ea poate fi găsităîn lucrările indicate. Cititorul se poate convinge uşor că, pentru şiruri crescătoare,reciproca este, de asemenea, adevărată: dacă şirul (a n ) este complet, atunci (pentruorice n ≥ 1) numărul a 1 + · · · + a n + 1 trebuie să poată fi exprimat ca o sumă determeni distincţi ai şirului. Evident, aceşti termeni nu pot fi toţi dintre a 1 , . . . , a n(căci suma lor nu ar depăşi pe a 1 + · · · + a n ), deci a 1 + · · · + a n + 1 = a i1 + · · · + a im ,unde indicii i 1 , . . . , i m sunt distincţi şi cel puţin unul dintre ei este mai mare ca n.Dacă i j ≥ n+1, avem a 1 +· · ·+a n +1 ≥ a ij ≥ a n+1 (aici folosim monotonia). Aşadarare loc:Propoziţia 1 ′ . Fie (a n ) un şir crescător de numere întregi pozitive cu a 1 = 1.Atunci (a n ) este complet dacă şi numai dacă a n+1 ≤ a 1 + · · · + a n + 1 pentru oricen ≥ 1.Se pot da multe exemple de şiruri complete: şirul numerelor naturale nenule (banal),şirul puterilor cu exponent întreg nenegativ ale lui 2 (de ce?), şirul (a n ) definitprin a k = k pentru orice 1 ≤ k ≤ a − 1 şi a n = a pentru orice n ≥ a, unde a ≥ 2este un număr natural fixat (iarăşi: de ce?). Vă propun cu această ocazie şi un primexerciţiu (dacă, desigur, nu socotim întrebările deja formulate în text şi lăsate fărărăspuns).1 Profesor, Colegiul Naţional ,,Gheorghe Roşca Codreanu”, Bârlad5


Exerciţiul 1. Fie b ≥ 2 un număr natural şi şirul1, . . . , 1, b, . . . , b, b 2 , . . . , b 2 , b 3 , . . .construit din blocuri de câte b − 1 termeni egali cu puterile lui b. Folosiţi acestşir şi Propoziţia 1 pentru a arăta că orice număr natural poate fi scris în baza b.Arătaţi deci că, pentru orice m ∈ N, există c 0 , c 1 , . . . , c k ∈ {0, 1, . . . , b −1} astfel încâtm = c 0 + c 1 b + · · · + c k b k .Puteţi găsi o formulă unitară pentru termenul general al acestui şir?2. O teoremă generală de reprezentare. Am văzut în prima parte a acesteilucrări că orice număr întreg impar se reprezintă într-o infinitate de feluri în forma±1 ± 2 ± · · · ± 2 k dacă alegem convenabil semnele şi numărul k. De asemenea, amamintit rezultatul lui Erdős şi Surányi [7] de prin 1960 conform căruia orice numărîntreg admite o infinitate de reprezentări de forma ±1 2 ± 2 2 ± · · · ± k 2 . Aceastădescendenţă justifică denumirea de şir Erdős-Surányi pe care M.O. Drimbe [6] odă unui şir (a n ) de numere întregi pozitive având proprietatea că orice număr întregm poate fi reprezentat într-o infinitate de moduri în forma m = ±a 1 ± a 2 ± · · · ± a kpentru anumite alegeri ale numărului k şi ale semnelor plus/minus. Rezultatul generalpe care-l putem obţine este următorul:Propoziţia 2. Fie (a n ) un şir de numere întregi pozitive astfel încât a 1 = 1 şia n+1 ≤ a 1 + · · · + a n + 1 pentru orice n ≥ 1.a) Dacă, în plus, şirul are o infinitate de termeni impari, atunci el este un şirErdős-Surányi.b) Dacă numărul termenilor impari din şir este finit, atunci, de la un anumit rangN, toate sumele S n = a 1 + · · · + a n au aceeaşi paritate. În acest caz, orice numărîntreg m de aceeaşi paritate cu sumele S n , n ≥ N, poate fi reprezentat într-o infinitatede moduri în forma m = ±a 1 ± a 2 ± · · · ± a k pentru anumite alegeri ale numărului kşi ale semnelor plus/minus.Demonstraţie. a) Dacă şirul are o infinitate de termeni impari e uşor de văzut căexistă o infinitate de indici p astfel încât sumele S p să fie numere pare şi, de asemenea,există o infinitate de indici p pentru care sumele S p respective sunt numere impare.Fie m un număr natural oarecare şi p suficient de mare astfel încât S p > m şi astfelîncât S p să aibă aceeaşi paritate cu m. Numărul l = (S p −m)/2 este natural şi mai micdecât S p , deci, conform ipotezei şi Propoziţiei 1, este suma câtorva termeni distincţiai şirului (a n ) cu indici cel mult egali cu p. Altfel spus, există u 1 , . . . , u p ∈ {0, 1}astfel încât l = u 1 a 1 + · · · + u p a p , adică12 (a 1 + · · · + a p − m) = u 1 a 1 + · · · + u p a p ⇔ m = (1 − 2u 1 )a 1 + · · · + (1 − 2u p )a p ;cum 1 − 2u 1 , . . . , 1 − 2u p ∈ {−1, 1} (iar p se poate alege într-o infinitate de moduri),teorema este demonstrată. Nu trebuie mare efort pentru a justifica şi partea a doua,nu?Punctul a) al acestei propoziţii e rezultatul principal din [6], iar o formulare maigenerală (care să ia în considerare şi situaţia în care şirul are doar un număr finit6


de termeni impari) se poate găsi în [8]. Scopul notei [6] era să arate că şirul numerelorprime are proprietatea Erdős-Suranyi; acesta este un rezultat clasic al luiH.E. Richert din 1949, iar M.O. Drimbe aplică în mod ingenios Propoziţia 2 unuişir construit cu ajutorul şirului numerelor prime pentru a-l demonstra. Pe de altăparte, J.L. Brown, Jr. demonstrează în [5] o generalizare a teoremei lui Richertcare se poate folosi pentru a demonstra că şi alte şiruri au proprietatea lui Erdős-Suranyi (de exemplu pentru şirul pătratelor perfecte); vom reveni şi noi în acest cercde idei, după un exerciţiu şi încă o observaţie (formulată tot ca exerciţiu).Exerciţiul 2. Folosiţi Propoziţiile 1 şi 2 pentru a arăta că şirul (F n ) al luiFibonacci definit prin F 1 = F 2 = 1 şi F n = F n−1 + F n−2 pentru orice n ≥ 3 estecomplet şi are proprietatea Erdős-Surányi.Exerciţiul 3. Observaţi cum este construit şirul complet (a n ) care începe cu2, 1, 5, 4, 14, 13, 41, 40, . . .şi arătaţi că pentru orice număr natural N, există n > N şi există m ∈ [1, S n ] naturalcare nu poate fi exprimat ca sumă de termeni distincţi ai şirului având toţi indicii celmult egali cu n.Acest exerciţiu arată că raţionamentul din [2] (preluat şi în [1]) făcut pentru ademonstra o formă mai generală a Propoziţiei 2 este greşit. Acest enunţ mai general(despre care nu ştiu dacă e adevărat) este următorul:Un şir (a n ) complet şi cu o infinitate de termeni impari are proprietatea Erdős-Surányi.Raţionamentul despre care vorbim este acelaşi pe care l-am prezentat şi noi maisus, doar că nimic nu ne asigură, pentru un şir complet oarecare, că orice număr maimic decît S n este suma unor termeni distincţi ai şirului având indici cel mult egali cun. Exemplul din Exerciţiul 3 arată clar asta, dar nu infirmă enunţul anterior (ci doarsubminează demonstraţia din [2]). Autorul rămâne pe mai departe curios cu privirela valabilitatea acestui enunţ şi mulţumeşte anticipat oricui îi va da un indiciu despreaceasta.Să mai remarcăm aici că Propoziţia 1 ′ reprezintă o caracterizare a şirurilor completemonotone şi cu primul termen egal cu 1; o astfel de caracterizare pentru şiruricomplete oarecare este de departe mai dificilă. Totuşi, cum se arată în [9], din oriceşir complet putem uşor construi un şir complet care să fie şi monoton şi să aibă primultermen egal cu 1 - prin eliminarea unor termeni şi rearanjarea unui număr finit dintrecei rămaşi (cititorul este invitat să reflecteze asupra acestei chestiuni înainte de aconsulta [9]), deci, într-un anume sens, ne putem mulţumi cu şirurile complete careau şi aceste proprietăţi.3. Din nou despre completitudine. E uşor de văzut că şirul pătratelor numerelornaturale nenule nu este complet şi, deci, nu se supune condiţiei din Propoziţiile1 sau 2. Prin urmare nu putem să arătăm că orice întreg se reprezintă în forma ±1 2 ±2 2 ± · · · ± k 2 folosind Propoziţia 2 (am sugerat o demonstraţie în [10], demonstraţiecare sigur le este cunoscută cititorilor cu preocupări în domeniul matematicii de concurs).De exemplu numărul 6 nu poate fi scris ca sumă de pătrate perfecte distincte7


· · · + a s+t−1 − a s+t , q + a s+1 + · · · + a s+t−1 ]. Datorită ipotezei ii), acest interval esteinclus în [p, q+a s+1 +· · ·+a s+t−1 ], prin urmare, datorită ipotezei de inducţie, x−a s+teste suma unor termeni ai lui (a n ) cu indici egali cel mult cu s + t − 1, ceea ce încheiedemonstraţia.Acum putem demonstra cvasicompletitudinea şirului pătratelor perfecte.Exerciţiul 4. Fie a n = n 2 pentru orice n ≥ 1. Arătaţi că (a n ) îndeplineştecondiţiile Teoremei 3 cu s = 10, p = 129 şi q = 256 şi deduceţi astfel că (a n ) estecvasicomplet - mai precis că orice m ∈ [129, 256 + 11 2 + · · · + (10 + t) 2 ] este suma unorpătrate perfecte distincte şi cel mult egale cu (10 + t) 2 , pentru orice t ≥ 1.4. Din nou despre proprietatea Erdős-Surányi. În fine, putem enunţaurmătoarea teoremă mai generală decât Propoziţia 2, care oferă condiţii suficientepentru ca un şir să aibă proprietatea Erdős-Surányi.Propoziţia 4. Fie (a n ) un şir de numere întregi pozitive care îndeplineşte condiţiilei) şi ii) din Propoziţia 3, precum şiiii) (a 1 + · · · + a s )/2 ≤ q,iv) şirul are o infinitate de termeni impari.Atunci acest şir are proprietatea Erdős-Suranyi.(Cititorul poate formula, desigur, un analog al punctului b) din Propoziţia 2.)Demonstraţie. Se vede imediat că ipoteza iii) implicăa 1 + · · · + a k2≤ q + a s+1 + · · · + a kpentru orice k ≥ s.Fie m un număr natural oarecare. Pentru o infinitate de numere k (suficient demari) a 1 + · · · + a k are aceeaşi paritate ca şi m (datorită ipotezei iv)) şi, de asemenea,avema 1 + · · · + a k − m≥ p2Atunci (a 1 +· · ·+a k −m)/2 este un întreg pozitiv cel puţin egal cu p şi mai mic decâta 1 + · · · + a k≤ q + a s+1 + · · · + a k ; conform Propoziţiei 3, acest număr este suma2unor termeni ai şirului (a n ) care au indici ce nu depăşesc pe k:a 1 + · · · + a k − m2= u 1 a 1 + · · · + u k a k ,pentru anumite numere u 1 , . . . , u k ∈ {0, 1}. De aici obţinem reprezentaream = (1 − 2u 1 )a 1 + · · · + (1 − 2u k )a k ,în care, desigur, 1 − 2u 1 , . . . , 1 − 2u k ∈ {−1, 1}. Asemenea reprezentare se poate găsipentru o infinitate de k şi demonstraţia este încheiată.Acum se poate dovedi şi pe această cale că şirul pătratelor perfecte are proprietateaErdős-Suranyi.9


Exerciţiul 5. Arătaţi că şirul pătratelor perfecte are proprietatea Erdős-Surányifolosind Propoziţia 4.Desigur, este o demonstraţie nepermis de complicată faţă de cea obişnuită (pecare, cum spuneam mai sus, fără îndoială că pasionaţii concursurilor de matematicăo cunosc, de exemplu din cartea lui Engel de metode şi strategii de rezolvare a unorasemenea probleme). Mai important e că am reuşit astfel să demonstrăm cvasicompletitudineaşirului pătratelor perfecte, care mai simplu nu prea se poate obţine -de altfel şi acest rezultat este unul clasic (a fost obţinut de R. Sprague în 1948;mai tîrziu s-a arătat că şirurile (n t ), cu t ≥ 2, sunt cvasicomplete şi au proprietateaErdős-Suranyi, acest fapt nebazându-se numaidecât pe cvasicompletitudine).Să încheiem tot în această notă veselă, a problemelor propuse şi lăsate fără rezolvări.Exerciţiul 6. Arătaţi că şirul 1, 4, 8, 9, 16, 25, 27, . . . <strong>format</strong> cu puterile numerelorîntregi pozitive cu exponent cel puţin 2 şi aşezate în ordine crescătoare are proprietateaErdős-Suranyi.Şi o ghicitoare a lui Erdős (pasionaţii o cunosc):Exerciţiul 7. Să se arate că şirul 1, 2, 3, 4, 6, 9, 12, 18, 24, 36, . . . <strong>format</strong> cu numerelede forma 2 a · 3 b (a, b numere naturale) este complet. Mai mult, orice numărîntreg pozitiv este suma unor termeni distincţi ai şirului dintre care nici unul nu dividepe altul (cu excepţia lui 1, desigur).Bibliografie1. D. Andrica, D. Văcăreţu – Representation Theorems And Almost Unimodal Sequences,Studia Univ. ,,Babeş-Bolyai”, Mathematica, vol. LI, nr. 4, December 2006.2. Cătălin Badea – Asupra şirurilor Erdős-Surányi, <strong>Revista</strong> Matematică din Timişoara,nr. 1, 1987, 10-13.3. J. L. Brown, Jr. – Note on Complete Sequences of Integers, The American MathematicalMonthly, Vol. 68, No. 6 (Jun. - Jul., 1961), 557-560.4. J. L. Brown, Jr. – Integer Representations and Complete Sequences, RMathematicsMagazine, Vol. 49, No. 1 (Jan., 1976), 30-32.5. J. L. Brown, Jr. – Generalization of Richertş Theorem, The American MathematicalMonthly, Vol. 83, No. 8 (Oct., 1976), 631-634.6. M. O. Drimbe – O problemă de reprezentare a numerelor întregi, Gazeta Matematică,seria B, nr 10-11, Octombrie-Noiembrie 1983, 382-383.7. P. Erdős, J. Surányi – Topics in the Theory of Numbers, Springer-Verlag, 2003,227-228.8. W. Y. Lee – On the Representation of Integers, Mathematics Magazine, Vol. 47,No. 3 (May, 1974), 150-152.9. E. Schissel – Characterizations of Three Types Of Completeness,http://www.fq.math.ca/Scanned/27-5/schissel.<strong>pdf</strong>.10. M. Tetiva – O problemă de reprezentare, Recreaţii Matematice, nr. 2, 2010, 123-127.10


Aplicaţii ale numerelor complexeîn geometria triunghiuluiFlorin STĂNESCU 1Abstract. In this Note, the equilateral and right-angled triangles are characterized in terms ofcertain relations involving the complex numbers associated with their vertices.Keywords: equilateral triangle, right-angled triangle, complex number.MSC 2000: 51M04.Ne propunem în cele ce urmează să găsim caracterizări ale triunghiurilor echilateraleşi ale triunghiurilor dreptunghice prin identităţi ce implică afixele vârfuriloracestora. Vom folosi pe parcurs binecunoscute relaţii metrice şi trigonometrice întriunghi (ale căror demonstraţii pot fi găsite în [1], precum şi unele inegalităţi uzualeîn triunghi (pentru care trimitem cititorul interesat la [2]).1. Preliminarii. Raportăm planul la un reper cartezian xOy şi fie ABC untriunghi înscris în cercul unitate. Notăm cu z 1 , z 2 , z 3 afixele vârfurilor A, B respectivC, unde |z 1 | = |z 2 | = |z 3 | = 1.Ortocentrul triunghiului are afixul h = z 1 + z 2 + z 3 , deci AH 2 = |(z 1 + z 2 + z 3 ) −z 1 | 2 = |z 2 + z 3 | 2 = (z 2 + z 3 )(z 2 + z 3 ) = |z 2 | 2 + |z 3 | 2 + z 2 z 3 + z 2 z 3 , prin urmarez 2 z 3 + z 2 z 3 = AH 2 − 2. Însă AH = 2R cos A (relaţia 15.6 din [1]) şi obţinem căz 2 z 3 + z 2 z 3 = 4 cos 2 A − 2 şi, întrucât z 2 = |z 2| 2= 1 , z 3 = 1 , vom aveaz 2 z 3(1) cos 2 A = (z 2 + z 3 ) 24z 2 z 3şi analoagele : cos 2 B = (z 1 + z 3 ) 24z 1 z 3, cos 2 C = (z 1 + z 2 ) 24z 1 z 2.Folosind faptul că sin 2 A + cos 2 A = 1, deducem că(2)sin 2 A = − (z 2 − z 3 ) 24z 2 z 3şi analoagele : sin 2 B = − (z 1 − z 3 ) 24z 1 z 3, sin 2 C = − (z 1 − z 2 ) 24z 1 z 2.Din (1) şi (2) rezultă(3) ctg 2 A = − z 2 + z 3, ctgz 2 − z 3‹22 B = − z 1 + z 3, ctgz 1 − z 3‹22 C = − z 1 + z 2.z 1 − z 2‹2Afixul centrului Ω al cercului medial al triunghiului ABC este 1 2 (z 1 +z 2 +z 3 ), deciAΩ = 1 2 |z 1 −z 2 −z 3 |. Pe de altă parte, aplicând teorema medianei în triunghiul AOH(unde Ω este mijlocul segmentului OH), obţinem că AΩ 2 = 2(AH2 + AO 2 ) − OH 2.4Folosind identităţile cos 2 A + cos 2 B + cos 2 C = 1 − 2 cos A cos B cos C şi OH 2 =R 2 (1 − 8 cos A · cos B cos C) (relaţiile 13.13 şi 15.10 din [1]), rezultă că(4) AΩ 2 + BΩ 2 + CΩ 2 =1 Profesor, Şcoala ,,Şerban Cioculescu”, Găeşti (Dâmboviţa)z 28 cos A cos B cos C + 11.411


2. Propoziţie. Fie z 1 , z 2 , z 3 numere complexe distincte de modul 1 şi A, B, Cpunctele de afixe z 1 , z 2 , respectiv z 3 . Dacă este adevărată una dintre egalităţile:(z 1 + z 2 ) 2+ (z 2 + z 3 ) 2+ (z 3 + z 1 ) 2(5)= 3;z 1 z 2 z 2 z 3 z 3 z 1 (6) z 1 z 2(z 1 − z 2 ) 2 + z 2 z 3(z 2 − z 3 ) 2 + z 3 z 1(z 3 − z 1 ) 2 = −1;z 1 + z 2+z 1 − z 2‹2z 2 + z 3+z 2 − z 3‹2z 3 + z 1(7)= −1;z 3 − z 1‹2(8) z 1 + z 2· z2 + z 3· z3 + z 1=iz 1 − z 2 z 2 − z 3 z 3 − z 1 3 √ şi △ABC ascuţiunghic;3|z 2 + z 3 | 4|z 1 − z 2 − z 3 | + |z 3 + z 1 | 4|z 2 − z 3 − z 1 | + |z 1 + z 2 | 2(9)|z 3 − z 1 − z 1 | = 3 2 ,atunci triunghiul ABC este echilateral.Demonstraţie. Ţinând seama de (2), relaţia (5) se poate scrie sub forma cos 2 A+cos 2 B + cos 2 C = 3 4 . Însă în orice triunghi are loc inegalitatea cos2 A + cos 2 B +cos 2 C ≥ 3 , egalitatea fiind atinsă în cazul în care △ABC este echilateral (inegalitatea4GU.1. din [2]). Rezultă că (5) atrage faptul că triunghiul este echilateral.În cazul egalităţilor (6) şi (7) procedăm similar, ţinând seama că inegalităţile1sin 2 A + 1sin 2 B + 1sin 2 C ≥ 4 şi ctg2 A+ctg 2 B +ctg 2 C ≥ 1, valabile în orice triunghi,devin egalităţi doar pentru triunghiul echilateral (GU.21. şi GU.50. din [2]).Relaţia (8) conduce, prin ridicare la pătrat şi folosind (3), la ctg 2 A · ctg 2 B ·ctg 2 C = 1 27 , adică tg A · tg B · tg C = 3√ 3 (am ţinut seama de faptul că △ABC esteascuţitunghic).Însă tg A + tg B + tg C = tg A · tg B · tg C, deci 27 = (tg A + tg B +tg C) 2 =Ptg 2 A+2Ptg A ctg B ≥ 3· 3ÈQtg 2 A+2·3· 3ÈQtg A · tg B = 9+18 = 27şi deducem că tg 2 A + tg 2 B + tg 2 C = 9. Astfel, se atinge egalitatea în inegalitateaGU.39. din [2] (tg 2 A + tg 2 B + tg 2 C ≥ 9), aşadar △ABC este echilateral.Observăm că (1) conduce, prin conjugare, la cos 2 A = (z 2 + z 3 ) 2, deci cos 4 A =4z 2 z 3[(z 2 + z 3 )(z 2 + z 3 )] 2= |z 2 + z 3 | 416z 2 z 2 · z 3 z 3 16|z 2 | 2 |z 3 | 2 = 1 16 |z 2 + z 3 | 4 ; rezultă că relaţia (9) se poaterescrie sub forma cos4 AAΩ+ cos4 BBΩobţinem că (AΩ+BΩ+CΩ) 2 ≤ 3(AΩ 2 +BΩ 2 +CΩ 2 ) = 3·3 · 1 + 114+ cos4 CCΩ = 3 . Pe de altă parte, utilizând (4)16 8 cos A · cos B · cos C + 11≤4= 9, deci AΩ + BΩ + CΩ ≤ 3. Inegalitatea lui Bergström arată că3=Xcos4 A16 AΩ ≥ (Pcos2 A)PAΩ2≥ (Pcos2 A) 2≥ 1 3 33=4‹23 16 ;12


ne-am folosit şi de inegalitatea cos 2 A + cos 2 B + cos 2 C ≥ 3 , amintită deja la (5).4Cum toate inegalităţile se transformă în egalităţi, rezultă că △ABC este echilateral.3. Propoziţie. Fie z 1 , z 2 , z 3 numere complexe distincte de modul 1 şi A, B, Cpunctele de afixe z 1 , z 2 , respectiv z 3 . Dacă este adevărată una dintre egalităţile:(10)(11)(12)z 1 + z 2+ z 2 + z 3+ z 3 + z 1= −2;z 3 z 1 z 2|z 1 + z 2 | 2 + |z 2 + z 3 | 2 + |z 3 + z 1 | 2 = 4|z 1 + z 2 + z 3 | 2 ;|z 1 − z 2 − z 3 | 2 + |z 2 − z 3 − z 1 | 2 + |z 3 − z 1 − z 2 | 2 = 11,atunci triunghiul ABC este dreptunghic.Demonstraţie. Relaţia (10) arată că −2 =Pz 1 + z 2z 3=Pz 3 (z 1 + z 2 ) =P(z 2 z 3 + z 2 z 3 ) =P(4 cos 2 A − 2), deci cos 2 A + cos 2 B + cos 2 C = 1. Rezultă că1 − 2 cos A · cos B · cos C = 1, adică cos A · cos B · cos C = 0 şi de aici urmează că△ABC este dreptunghic.Egalitatea (11) se rescrie sub forma 4 cos 2 A + 4 cos 2 B + 4 cos 2 C = 4OH 2 , deunde 1 − 2 cos A · cos B · cos C = 1 − 8 cos A · cos B · cos C, deci cos A · cos B · cos C = 0şi din nou obţinem că △ABC este dreptunghic.Din (12) rezultă, ţinând seama de (4), că 11 = 4(AΩ 2 +BΩ 2 +CΩ 2 ) = 11+8 cos A·cos B · cos C, prin urmare cos A · cos B · cos C = 0, adică △ABC este dreptunghic.Bibliografie1. T. Lalescu – Geometria triunghiului, Editura Tineretului, Bucureşti, 1958.2. I.V. Maftei, P.G. Popescu – Inegalităţi alese în matematică, Editura Niculescu,Bucureşti, 2005.Nichita StănescuAltă matematicăNoi ştim că unu ori unu fac unu,dar un inorog ori o parănu ştim cât face.Ştim că cinci fără patru fac unu,dar un nor fără o corabienu ştim cât face.Ştim, noi ştim că optîmpărţit la opt fac unu,dar un munte împărţit la o caprănu ştim cât face.Ştim că unu plus unu fac doi,dar eu şi cu tine,nu ştim, vai, nu ştim cât facem.Ah, dar o plapumăînmulţită cu un iepureface o roşcovană, desigur,o varză împărţită la un steagfac un porc,un cal fără un tramvaiface un înger,o conopidă plus un ou,face un astragal...Numai tu şi cu mineînmulţiţi şi împărţiţiadunaţi şi scăzuţirămânem aceiaşi...Pieri din mintea mea!Revino-mi în inimă!13


Extinderea unui rezultat al lui Johann HuddeCornelia-Livia BEJAN 1Abstract. We give here an extention to series of a result obtained by J. Hudde concerningpolynomial equations.Keywords: polynomial, series, radius of convergence.MSC 2000: 40A30, 40A05.Matematicianul olandez Johann Hudde (1628-1704) a descris două reguli care-ipoartă numele cu privire la ecuaţiile polinomiale [1].Amintim aici pe una dintre ele. Peste tot în această Notă vom considera toatenumerele care apar (coeficienţi, rădăcini etc.) reale.Teorema 1 (J. Hudde). Dacă r este o rădăcină dublă a ecuaţiei polinomiale(1) a n x n + a n−1 x n−1 + ... + a 1 x + a 0 = 0şi dacă b 0 , b 1 , ..., b n sunt numere în progresie aritmetică, atunci r este de asemeneao rădăcină a ecuaţiei(2) a n b n x n + a n−1 b n−1 x n−1 + ... + a 0 b 0 = 0.Acest rezultat se poate extinde prinTeorema 2. Fie dată ecuaţia (1) de mai sus. Atunci r este o rădăcină dublăa acestei ecuaţii dacă şi numai dacă pentru orice progresie aritmetică b 0 , b 1 , ..., b necuaţia (2) are pe r ca rădăcină.Demostraţie. Afirmaţia directă este dată de Teorema 1. Reciproc, fieb 0 , b 0 + q, ..., b 0 + nq şi c 0, c 0 + q, ..., c 0 + nqdouă progresii aritmetice cu aceeaşi raţie şi b 0 ≠ c 0 . Cum r verificăprecum şi ecuaţiaa n (b 0 + nq)x n + a n−1 [b 0 + (n − 1)q]x n−1 + ... + a 0 b 0 = 0,a n (c 0 + nq)x n + a n−1 [c 0 + (n − 1)q]x n−1 + ... + a 0 c 0 = 0,rezultă, prin scăderea acestora, că r verifică şi ecuaţia (1). În plus, luând progresiaaritmetică b k = k , k = 0, n , şi presupunând că r verifică ecuaţia (2) pentru aceastăprogresie, rezultă că r este o rădăcină dublă a ecuaţiei (1).Pentru serii, dăm următoarea teoremă de caracterizare:1 Prof. dr., Departamentul de Matematică şi In<strong>format</strong>ică, Univ. Tehnică ,,Gh. Asachi”, Iaşi14


Teorema 3. Fie ρ ∈ [0, ∞]. Atunci seria de puteri a n x∞Pi=1n are raza de convergenţăρ dacă şi numai dacă pentru orice progresie aritmetică infinită b 0 , b 1 , ..., b n , ...seria ∞Pi=1 a n b n x n are raza de convergenţă ρ.Demonstraţie. Dacă seria ∞Pi=1 a n x n are raza de convergenţă ρ, atunci şi seriaderivatăna n x∞Pi=1n−1 are raza de convergenţă ρ . Fie b n = b 0 + qn, ∀n ∈ N, oprogresie aritmetică infinită cu raţia q. Întrucât seria∞Xi=1∞Xi=1∞Xi=1∞Xi=1a n b n x n =a n b 0 x n +a n · qn · x n = b 0∞Xi=1a n x n + qx ·na n x n−1se scrie ca sumă a două serii cu aceeaşi rază de convergenţă ρ, rezultă implicaţiadirectă.Reciproc, dacă a n b n x∞Pi=1n are raza de convergenţă ρ pentru orice progresiearitmetică infinită, considerăm în particular b n = n şi obţinem că x · ∞Pi=1na n x n−1are raza de convergenţă ρ. Deoarece pentru orice progresie aritmetică cu b 0 ≠ 0, b n =b 0 + qn, ∀n ∈ N, seria ∞Pi=1 a n b n x n are raza de convergenţă ρ, rezultă că diferenţa∞Xi=1∞Xi=1∞Xi=1∞Xi=1a n b n x n − xq ·na n x n−1 =a n (b n − nq)x n = b 0a n x nreprezintă o serie cu raza de convergenţă ρ, ceea ce încheie demonstraţia.Corolar 4. Fie α ∈ R şi (a n ) n∈N ∗ un şir oarecare. Atunci şirul S n =a k αnPk=1k este convergent dacă şi numai dacă pentru orice progresie aritmetică infinităb 0 , b 1 , ..., b n , ... şirul T n = nPk=1 a k b k α k este convergent.Bibliografie1. B. Boyer – A history of mathematics, 2 −nd ed., John Wiley&Sons, Inc., 1991, p.373.Vizitaţi pagina web a revistei Recreaţii Matematice:http://www.recreatiimatematice.ro15


O condiţie de existentă a triunghiurilordreptunghice de arie şi perimetru dateNeculai STANCIU 1 , Titu ZVONARU 2Abstract. Given a right-angled triangle with area A and perimeter P , what conditions on Aand P guarantee that the triangle actually exists?Keywords: Heronian triangle, primitive triangle, right-angled triangles, area, perimeter.MSC 2000: 51M16, 51M71.1. Introducere. În lucrarea [16] au fost prezentate rezolvările următoarelorprobleme:Problema 1 ([3]). Determinaţi toate triunghiurile dreptunghice de arie A şiperimetru P cu lungimile laturilor a, b, c numere naturale şi cel mai mare divizorcomun al lor (a, b, c) = 1 astfel încât P 2∈ N. (Triunghiurile ale căror laturi verificăAcondiţiile de mai sus se numesc triunghiuri Heron. Rezultate deosebite în privinţa lora stabilit matematicianul indian K.R.S. Sastry în [7], [8], [9], [10] şi [11].)Problema 2. Determinaţi toate triunghiurile dreptunghice cu lungimile laturilornumere naturale care au:a) aria egală cu 84 unităţi de arie;b) perimetrul egal cu 24 unităţi de lungime.Problema 3. Determinaţi toate triunghiurile cu lungimile laturilor numere naturaleşi care au aria egală cu semiperimetrul. (Singurul astfel de triunghi este triunghiuldreptunghic cu lungimile laturilor: 3, 4 şi 5.)Problema 4. Determinaţi toate triunghiurile dreptunghice cu lungimile laturilornumere naturale şi care au aria egală cu perimetrul. (Există două triunghiuri dreptunghicepentru care A = P şi anume: triunghiul a = 10; b = 6; c = 8 şi triunghiula = 13; b = 12; c = 5.)Problema 5. Arătaţi că nu există triunghiuri Heron cu lungimile laturilor numereprime.Problema 6. Considerăm un triunghi dreptunghic cu perimetrul P şi aria Anumere naturale. Arătaţi că ipotenuza este număr natural dacă şi numai dacă P estenumăr natural par şi P |2A.Problema 7. Determinaţi toate triunghiurile care au lungimile laturilor numereprime şi pătratul ariei număr natural.Numim triunghi Super-Heron un triunghi care are lungimile laturilor numere naturaleconsecutive şi aria tot număr natural. Recent 3 , 2007, s-a demonstrat existenţaunui număr infinit de astfel de triunghiuri.1 Profesor, Şcoala Generală ,,George Emil Palade”, Buzău2 Comăneşti, tzvonaru@yahoo.com3 http://www.math.twsu.edu/ richardson/heronian/heronian.html16


Problema 8. Confirmaţi sau infirmaţi existenţă patrulaterelor inscriptibele Super-Heron. (Indicaţie. Folosiţi formula ariei patruterului inscriptibil dată de matematicianulindian Brahmagupta, sec.VII d.Hr.: A =È(p − a)(p − b)(p − c)(p − d), undea, b, c, d sunt lungimile laturilor patrulaterului, iar p este semiperimetrul acestuia.)Probleme asemănătoare au fost propuse recent în [14] şi [15].2. Rezultatul principal. În încheierea lucrării [16] s-a propus următorulExerciţiu. Stabiliţi o condiţie de existenţă a triunghiurilor dreptunghice de arieA şi perimetru P .Scopul acestui articol este de a stabili o condiţie de existenţă a triunghiurilordreptunghice de arie A şi perimetru P date.Mai mulţi autori au propus probleme care aveau ca date iniţiale A şi P fără a seexamina dacă triunghiul respectiv există în realitate (v. [12]).Se ştie că într-un triunghi oarecare avem:(1) P 2 ≥ 12 √ 3 · A ⇔ A ≤√336 P 2 .În relaţiile de mai sus avem egalitate dacă şi numai dacă triunghiul este echilateral.Vom demonstra relaţia (1) folosind inegalitatea izoperimetrică pentru poligoane:dintre toate poligoanele de perimetru dat şi acelaşi număr de laturi, poligonul regulatare aria maximă; dual, dintre toate poligoanele de arie dată şi acelaşi număr de laturi,poligonul regulat are perimetrul minim. Aşadar pentru triunghiuri avem:A ≤ A 3 = l2√ 3⇔ 1 4 A ≥ 4l 2√ P3⇒ P ≥ P 3 = 3l ⇔ P 2 ≥ 9l 22A ≥ 12√ 3 ∼ 20, 785.Vom stabili o condiţie asemănătoare de existenţă a triunghiurilor dreptunghicepornind de la următorulExerciţiu. Calculaţi lungimea ipotenuzei unui triunghi dreptunghic cu A = 8 şiP = 13.Soluţie. Este uşor de verificat că pentru A = 8, P = 13, relaţia (1) este adevărată.= 2APrima idee ar fi să rezolvăm sistemul¨aba + b + √ , unde a, b sunt catetelea 2 + b 2 = Ptriunghiului. Există însă şi o cale mai ,,frumoasă”(v. [13]). Dacă c este ipotenuză,avem: P −c = a+b ⇔ P 2 −2P c+c 2 = a 2 +2ab+b 2 a2 +b 2 =c=2ab=2A c2 +4A ⇔ P 2 −4A = 2P c,adică c = P 2 − 4A137. În cazul nostru avem c =2P26 .Să calculăm lungimile catetelor din sistemul8< :ab = 16a + b = 201 . Obţinem ecuaţia2626x 2 − 201x + 416 = 0 cu x 1,2 = 201 ± i√ 2863; aşadar, triunghiul considerat nu52există!17


În cazul unui triunghi dreptunghic condiţia de existenţă este:(∗) P ≥ 2(1 + √ 2) · √A⇔ P 2 ≥ 4(3 + 2 √ 2) · A ⇔ A ≤ 3 − 2√ 2P 2 .4Deci,Într-adevăr, ca mai sus, avem:P 2A ≥ 4(3 + 2√ 2) ∼ 23, 314.P − c = a + b ⇔ P 2 − 2P c + c 2 = a 2 + 2ab + b 2 a 2 +b 2 =c=2ab=2A c2 + 4A ⇔ P 2 − 4A = 2P c.Rezultă că c = P 2 − 4A. Din2PApoi,a + b = P − c = P − P 2 − 4A2P= P 2 + 4A2P⇒ b = P 2 + 4A2P− a.ab = 2A ⇒ aP 2 + 4A2PDar, ultima ecuaţie are soluţii reale dacă− a=2A ⇒ P a22 −P 24 + Aa + AP = 0.∆ a ≥ 0 ⇔P 2A24 + − 4 P 2AP ≥ 0 ⇔P− 6P2 4A22≥ 0.4A+1Rezolvând ultima inecuaţie, obţinem:echivalente cuP 2 ≥ 4A(3 + 2 √ 2) sau P 2 ≤ 4A(3 − 2 √ 2),P ≥ 2(1 + √ 2) √ A sau P ≤ 2( √ 2 − 1) √ A.Aceste ultime condiţii trebuiesc corelate şi cu condiţia P > 2 √ A, de existenţă aipotenuzei. Obţinem că triunghiul dreptunghic de arie A şi perimetru P există dacăşi numai dacă avem:P ≥ 2(1 + √ 2) √ A ⇔ A ≤ P 2 (3 − 2 √ 2)4⇔ A P 2 ≤ 3 − 2√ 24⇔ P 2A ≥ 4(3 + 2√ 2).În relaţiile de mai sus avem egalitate dacă şi numai dacă triunghiul este dreptunghicisoscel.Remarca 1. În 1904, Whitworth şi Biddle au arătat că există numai cincitriunghiuri Heron cu proprietatea A = P, şi anume, triunghiurile Heron cu laturile(6, 8, 10), (5, 12, 13), (6, 25, 29), (7, 15, 20) şi (9, 10, 17) ([1]).18


Remarca 2. Deşi Goehl, în [2], prezintă un algoritm general pentru determinareatuturor triunghiurilor dreptunghice de tip Heron cu proprietatea că A =mP (m ∈ N), problema este rezolvă în cazul general (pentru triunghiurile Heron oarecare)abia peste mai mult de 20 de ani de către Lubomir Markov ([4]), care, dupăpuţin timp revine chiar cu o metodă nouă ([5]). O metodă diferită pentru rezolvareaproblemei A = mP , pentru triunghiurile Heron oarecare este prezentată de cătreJizhou Li în [6].Bibliografie1. L. Dickson – History of the Theory of Numbers, Vol. II, Chelsea Publish-ing Co,NY, 1992 (reprint from 1923 edition).2. J. Goehl – Area=k(perimeter), Math. Teach. 79(1985), 330-33<strong>2.3</strong>. John F. Goehl, Jr. – Pythagorean Triangles with Square of Perimeter Equal to anInteger Multiple of Area, Forum Geometricorum, Vol. 9 (2009), 281-282.4. L. Markov – Pythagorean triples and the problem A = mP for triangles, Math.Mag. 79 (2006), 114-121.5. L. Markov – Heronian Triangles Whose Areas are Integer Multiple of their Perimeters,Forum Geometricorum, (2007), 129-135.6. http://www.mathlab.mtu.edu/∼jizhoul/Site/Home Page files/Goehl ′ s%20Problem.<strong>pdf</strong>7. K.R.S. Sastry – Heron Problems, Math. And Comput. Ed., 29(1995), 192-202.8. K.R.S. Sastry – Heron Triangles: A New Perspective, Aust. Math. Soc. Gazette,26 (1999), 160-168.9. K.R.S. Sastry – Heron Triangles, Math. And Comput. Ed., 35 (2001), 51-60.10. K.R.S. Sastry – A Heron Difference, Crux mathematicorum with MathematicalMayhem, 27(2001), 22-26.11. K.R.S. Sastry – If (a, b, c) is Heron, can (s − a, s − b, s − c) also be Heron?, CruxMathematicorum with Mathematical Mayhem, 28 (2002), 23-27.12. Problema E:14152, Gazeta Matematică - seria B, nr. 3/2011, 154.13. Problema VIII.293, <strong>Revista</strong> de Matematică din Timişoara, nr. 4/2010, 17.14. Problema S:E11.170, Gazeta Matematică - Supliment cu exerciţii, Gazeta Matematică- seria B, nr. 5/2011, 7.15. Problema S:E11.163, Gazeta Matematică - Supliment cu exerciţii, Gazeta Matematică- seria B, nr. 5/2011, 7.16. T. Zvonaru – Triunghiuri de arie şi perimetru date; în Articole şi note matematice,Vol. IV, Societatea de Ştiinţe Matematice din România - Filiala Râmnicu Sărat,Editura Rafet, 2011, 120-129.19


O demonstraţie geometricăa inegalităţii lui BlundonMarius DRĂGAN 1 , Mihai HAIVAS 2 , I.V. MAFTEI 3Abstract. It is presented a geometrical proof of Blundon ′ s inequality.Keywords: circumcenter, orthocenter, incenter, Blundon ′ s inequality.MSC 2000: 51M16.Inegalitatea lui Blundon a fost şi continuă să fie obiect de preocupare în multelucrări de geometria triunghiului ([1], [2], [3] etc.). Ne propunem în această Notă sădăm o nouă demonstraţie acestei inegalităţi:Propoziţie.În orice triunghi au loc inegalităţile:2R 2 + 10Rr − r 2 − 2(R − 2r)ÈR(R − 2r) ≤ p 2 ≤(1)≤ 2R 2 + 10Rr − r 2 + 2(R − 2r)ÈR(R − 2r)(notaţiile utilizate sunt cele obişnuite).Vom prezenta o demonstraţie geometrică, care are în centrul ei anumite relaţiigeometrice în triunghiul OHI.Considerăm cunoscute următoarele formule pentru distanţele dintre punctele O, Hşi I:(2)(3)(4)OI 2 = R(R − 2r),OH 2 = 9R 2 + 8Rr + 2r 2 − 2p 2 ,HI 2 = 4R 2 + 4Rr + 3r 2 − p 2 .Lema 1. În triunghiul OHI, dacă m I notează lungimea medianei corespunzătoarelaturii OH, avem relaţia(5) m I = 1 (R−2r) ([4], p. 123, f. (3)).2Demonstraţie. Cu teorema medianei şi ţinând seama de (2), (3), (4), avem:4m 2 I = 2(OI 2 + HI 2 ) − OH 2 ⇔4m 2 I = 2(R 2 − 2Rr + 4R 2 + 4Rr + 3r 2 − p 2 ) − (9R 2 + 8Rr + 2r 2 − 2p 2 ) ⇔4m 2 I = R 2 − 4Rr + 4r 2 ⇔4m 2 I = (R − 2r) 2 ⇔m I = 1 (R − 2r).21 Profesor, Colegiul Tehnic ,,Mircea cel Bătrân”, Bucureşti2 Cercetător ştiinţific, Inst. Cerc. Econ. ,,Gh. Zane”, Iaşi3 Profesor, Colegiul Naţional ,,Sf. Sava”, Bucureşti20


Lema 2. În triunghiul OHI are loc inegalitatea:(6) OI ≥ 2m I .Demonstraţie. Într-adevăr,OI 2 − 4m 2 I = R(R − 2r) − (R − 2r) 2 = 2r(R − 2r) ≥ 0.Putem acum să trecem laDemonstraţia Propoziţiei. Notăm cu I ′ simetricul punctului I în raport cu mijloculsegmentului OH (adică punctul O 9 – centrul cercului celor nouăpuncte). În triunghiul HII′ , putem scrie (ţinem seamaIcă HI ′ = OI):echivalent cuOI − 2m I ≤ HI ≤ OI + 2m I ,sau, utilizând (2), (4) şi (5),OI 2 − 4OI · m I + 4m 2 I ≤ HI 2 ≤ OI 2 + 4OI · m I + 4m 2 IR(R − 2r) − 2(R − 2r)ÈR(R − 2r) + (R − 2r) 2 ≤ 4R 2 + 4Rr + 3r 2 − p 2 ≤de unde deducem căO≤ R(R − 2r) + 2(R − 2r)ÈR(R − 2R) + (R − 2r) 2 ,2R 2 +10Rr−r 2 −2(R−2r)ÈR(R − 2r) ≤ p 2 ≤ 2R 2 +10Rr−r 2 +2(R−2r)ÈR(R − 2r),adică inegalitatea lui Blundon.Observaţie. Dacă triunghiul dat este echilateral, în care caz R = 2r, avem,evident, egalitate în ambii membri din (1). În [3] sunt aduse precizări în privinţacondiţiilor în care are loc egalitatea în membrul stâng din (1) şi, separat, în membruldrept.Bibliografie1. W.J. Blundon – Problem 1935, The Amer. Math. Monthly, 73 (1966), 1122.2. W.J. Blundon – Inequalities associated with a triangle, Canad. Math. Bull., 8(1965), 615-626.3. A. Lupaş – Asupra unor inegalităţi geometrice, <strong>Revista</strong> Matematică din Timişoara,nr. 1/1984, 21–23.4. C.P. Niculescu – Caracterul algebric al inegalitătii lui Blundon, Gazeta Matematică,nr. 7-8/1999, 270-275.5. D. Sachelarie – Geometria triunghiului. Anul 2000, Matrix Rom, Bucureşti, 2000.21IO 9H


Dubla inegalitate a lui Blundon revizitatăTemistocle BÎRSAN 1Abstract. In the case of Blundon ′ s inequalities, the conditions when the equality holds in theleft hand side are different from the ones in the right hand side. The purpose of the present note isto establish this well known result in a geometric way.Keywords: Blundon ′ s inequalities, circumcenter, incenter, nine-point center.MSC 2000: 51M16.1. Introducere. În 1965, W.J. Blundon a arătat în [2] că într-un triunghioarecare are loc următoarea dublă inegalitate:Propoziţia 1.În orice triunghi are loc dubla inegalitate:(1)2R 2 +10Rr − r 2 −2 (R − 2r)ÈR(R − 2r)≤ p 2 ≤≤ 2R 2 +10Rr − r 2 +2 (R − 2r)ÈR(R − 2r),unde p, R, r au semnificaţiile obişnuite.În [4], Al. Lupaş stabileşte pe cale algebrică condiţiile în care inegalităţile din(1) devin egalităţi:Propoziţia 2. 1) Are loc egalitate în partea stângă a relaţiilor (1) dacă şi numaidacă triunghiul este echilateral sau este isoscel cu baza mai mare ca laturile egale.2) Are loc egalitate în partea dreaptă dacă şi numai dacă triunghiul este echilateralsau este isoscel cu baza mai mică decât laturile egale.D. Andrica şi C. Barbu în [1] şi M. Drăgan, M. Haivas şi I.V. Mafteiîn [3] (v. pag. 20-21 ale acestui număr de Recreaţii Matematice) dau demonstraţiigeometrice diferite inegalităţilor lui Blundon, fără a ajunge geometric la condiţiile deegalitate menţionate în Propoziţia 1.Scopul acestei Note este de a da inegalităţilor (1) o demonstraţie geometrică simplăşi completă (incluzând condiţiile de egalitate).2. Consideraţii preliminare. Amintim mai întâi că într-un triunghi oarecaresunt adevărate relaţiile următoare:(2)(3)(4)OI 2 = R(R − 2r),OH 2 = 9R 2 + 8Rr + 2r 2 − 2p 2 = 9R 2 − (a 2 + b 2 + c 2 ),OO 9 = 1 2 OH,unde O 9 notează centrul cercului celor nouă puncte (mijlocul segmentului [OH]),celelalte notaţii fiind binecunoscute, şi relaţia lui Sondat (de ex., [6], p. 118):(5) S OIH = 1 |(a − b)(b − c)(c − a)| .8r 1 Prof. dr., Univ. Tehnică ,,Gh. Asachi”, Iaşi22


Se ştie că punctul I aparţine dreptei lui Euler [OH] dacă şi numai dacă triunghiuleste isoscel, după cum rezultă imediat din (5).Fie ABC un triunghi isoscel cu BC = a şi AB = AC = l. În acest caz, puncteleO, H, I, O 9 , A şi A ′ (mijlocul laturii [BC]) sunt coliniare, se află pe axa triunghiului.Ţinând seama de formulele R = abc4S şi r = S , deducem căp(6) R =l 2√4l2 − a 2 , r = a√ 4l 2 − a 22(2l + a) .Putem acum să demonstrăm cu uşurinţă următoareaLemă. În triunghiul isoscel ABC , cu BC = a şi AB = AC = l, are locrelaţia:(7) OI 2 − OO9 2 = (l − a)3 (3l + a)4(4l 2 − a 2 .)Demonstraţie. Coroborând relaţiile (2), (3), (4) şi (6), putem scrie:OI 2 − OO 2 9 = R(R − 2r) − 1 4”9R 2 − (2l 2 + a 2 )—= 1 4€2l 2 + a 2 − 5R 2 − 8RrŠ=1=4(4l 2 − a 2 )€3l 4 + 6a 2 l 2 − a 4 − 8al 3Š==de unde relaţia cerută.14(4l 2 − a 2 )”€l 4 − a 4Š+€2l 4 − 2al 3Š+€6a 2 l 2 − 6al 3Š—=l − a=4(4l 2 − a 2 )€3l 3 + a 2 l + a 3 − 5al 2Š=l − a=4(4l 2 − a 2 )”€3l 3 − 3al 2Š+€a 2 l − al 2Š+€a 3 − al 2Š—=(l − a)2=4(4l 2 − a 2 )€3l 2 − 2al − a 2Š= (l − a)3 (3l + a)4(4l 2 − a 2 ,)Poziţia punctului I ∈ [OH] faţă de O 9 este dată dePropoziţia 3. În condiţiile anterioare relativ la triunghiul ABC , au loc afirmaţiile:1) I ∈ (HO 9 ) ⇔ l > a; 2) I ∈ (OO 9 ) ⇔ l < a; 3) I ≡ O 9 sau I ≡ O sauI ≡ H ⇔ l = a (triunghiul ABC este echilateral).Demonstraţie. Utilizăm Lema precedentă. Avem:1) I ∈ (HO 9 ) ⇔ OI > OO 9 ⇔ OI 2 − OO 2 9 > 0 ⇔ (l − a) 3 > 0 ⇔ l > a . La fel sedovedesc şi afirmaţiile 2) şi 3).3. Demonstraţie geometrică a Propoziţiilor 1 şi 2. Inegalităţile (1) se obţinuşor prin aplicarea inegalităţilor triunghiulare în triunghiul IOO 9 , anume(8) OI − IO 9 ≤ OO 9 ≤ OI + IO 9 .23


Relaţiile (2), (3) şi (4) dau OI şi OO 9 exprimate prin p, R şi r; în ce priveşteIO 9 , cu teorema medianei aplicată în triunghiul IOH găsim expresia(9) IO 9 = 1 (R − 2r) ,2formulă cunoscută, demonstrată în acest număr de revistă la pag. 20 ([3], Lema 1,f. (5)) sau în alte lucrări (de ex., [6], p.123, f.(3)).Să mai observăm că OI − IO 9 ≥ 0; într-adevăr, OI ≥ IO 9 ⇔ R(R − 2r) ≥14 (R − 2r)2 ⇔ (R − 2r)(3R + 2r) ≥ 0, evident.Prin ridicare la pătrat, relaţiile (8) iau formaOI 2 + IO 2 9 − 2OI · IO 9 ≤ OO 2 9 ≤ OI 2 + IO 2 9 + 2OI · IO 9şi, în urma înlocuirii mărimilor în funcţie de p, R şi r, devin4R(R − 2r) + (R − 2r) 2 − 4(R − 2r)ÈR(R − 2r) ≤ 9R 2 + 8Rr + 2r 2 − 2p 2 ≤≤ 4R(R − 2r) + (R − 2r) 2 + 4(R − 2r)ÈR(R − 2r) ,de unde obţinem tocmai inegalităţile (1).Dacă triunghiul ABC este echilateral, atunci punctele O, I, O 9 coincid şi avemegalitate în ambii membri în (1). Dacă triunghiul ABC nu este echilateral, atunciaceste puncte nu coincid şi urmează:Egalitate în partea stângă a dublei inegalităţi precedente avem dacă şi numai dacăOI + IO 9 = OO 9 , adică I ∈ (OO 9 ), echivalent, conform Propoziţiei 3, cu l < a.Egalitate în partea dreaptă avem dacă şi numai dacă OI − IO 9 = OO 9 , adicăI ∈ (HO 9 ), echivalent, conform Propoziţiei 3, cu l > a, ceea ce încheie demonstraţiaafirmaţiilor Propoziţiilor 1 şi 2.Observaţie. 1) În [1] este utilizat triunghiul OIN , N - punctul lui Nagel, secalculează cosÕION şi se obţine (1). Punând condiţia −1 ≤ cosÕION ≤ +1, fără a seda precizări definitive pentru cazurile de egalitate.2) În [3] este utilizat triunghiul HII′ , I ′ - simetricul lui I faţă de O 9 , şi apoiinegalitatea triunghiulară în acest triunghi. Cu ajutorul unei propoziţii similare cuPropoziţia 3, ar fi fost posibilă precizarea cazurilor de egalitate până la capăt.Bibliografie1. D. Andrica, C. Barbu – A geometric proof of Blundon ′ s inequalities, Math. Ineq.&Appl., Prepint (Google: mia-2592-pre.<strong>pdf</strong>)2. W.J. Blundon – Inequalities associated with a triangle, Canad. Math. Bull., 8(1965), 615-626.3. M. Drăgan, M. Haivas – demonstraţie geometrică a inegalităţii lui Blundon,Recreaţii Matematice, nr. 1/2012, p.4. A. Lupaş – Asupra unor inegalităţi geometrice, <strong>Revista</strong> Matematică din Timişoara,nr. 1/1984, 21-23.5. T. Lalescu – Geometria triunghiului, Ed. Tineretului, Bucureşti, 1958.6. D. Sachelarie – Geometria triunghiului. Anul 2000, Matrix Rom, Bucureşti, 2000.24


Asupra unei probleme de extremRadu MIRON 1Abstract. The purpose of this Note is to give two methods for finding the extremal values of acertain type of rational functions which are subject to quadratic restrictions.Keywords: maximal value, minimal value, circle, tangent.MSC 2000: 97D50.În Gazeta Matematică 7-8-9/2010 apare următoarea problemă:E:14062. Aflaţi cea mai mică şi cea mai mare valoare a fracţiei x − 1 , ştiind căy − 1x 2 + y 2 − 6x − 6y + 17 = 0.Susana CosteaÎn numărul 3/2011 al Gazetei, pag. 143, este publicată următoarea soluţie:,,Relaţia dată se scrie sub forma (x − 3) 2 + (y − 3) 2 = 1, de unde rezultă că(x − 3) 2 ≤ 1 şi (y − 3) 2 ≤ 1 şi de aici 2 ≤ x ≤ 4, 2 ≤ y ≤ 4.Valoarea fracţiei x − 1 este minimă când numărătorul x − 1 este cât mai mic, iary − 1numitorul y − 1 este cât mai mare. Vom lua x = 2, y = 4 (1) şi cea mai mică valoarea fracţiei este 1 . Pentru valoarea maximă a fracţiei trebuie să avem x − 1 cât mai3mare şi y − 1 cât mai mic. Vom lua x = 4, y = 2 (2) şi obţinem că valoarea maximăa fracţiei este 3.”Se observă că soluţia prezentată mai sus este greşită, valorile considerate în (1) şi(2) neverificând condiţia x 2 + y 2 − 6x − 6y + 17 = 0 din enunţ.În cele ce urmează prezentăm două metode de abordare pentru problema dată, pecare le vom folosi apoi în rezolvarea altor două probleme de acest tip.Soluţia 1. Fie k = x − 1 ; atunci x = 1+k(y−1). Înlocuind în relaţia din ipoteză,y − 1obţinem că y 2 (k 2 + 1) − y(2k 2 + 4k + 6) + (k 2 + 4k + 12) = 0. Cum y ∈ R, rezultă că∆ y ≥ 0. După calcule, ∆ y = 12k 2 −+ 32k − 12 şi de aici k ∈–4 √ 7, 4 + √ 7, adică3 34 − √ 73≤ x − 1y − 1 ≤ 4 + √ 73Observăm că x − 1y − 1 = 4 − √ 73pentru orice x, y ca în ipoteza problemei.pentru x = 11 − √ 73x 2 +y 2 −6x−6y +17 = 0. Apoi, x − 1y − 1 = 4 + √ 73, y = 11 + √ 7, care verifică3pentru x = 11 + √ 73, y = 11 − √ 7,3care verifică relaţia dorită. În concluzie, valorile extreme ale fracţiei date sunt 4 − √ 7,3respectiv 4 + √ 7.31 Elev, Liceul Teoretic ,,Dimitrie Cantemir”, Iaşi25


Soluţia 2. Într-un sistem cartezian xOy, ecuaţia (x−3)2 +(y −3) 2 = 1 reprezintăun cerc de centru Q(3, 3) şi rază r = 1. Fie A(1, 1) şi M(x, y) un punct situatpe cerc, atunci x − 1y − 1 = 1 , unde m AM este panta dreptei AM. Fracţia x − 1m AM y − 1are valoare minimă atunci când m AM este maximă, decicând AM coincide cu tangenta ,,superioară” AT 2 lacerc. Analog, x − 1y − 1 are valoare maximă dacă m AM esteminimă, adică atunci când AM coincide cu tangenta ,,inferioară”AT 1 la cerc. Găsim imediat ecuaţiile tangentelorAT 1 : y − 1 = 4 − √ 7(x − 1), AT 2 : y − 1 =34 + √ 7(x − 1) şi coordonatele punctelor de contact cu3+ √ 7cercul: T 1‚11, 11 − √ 7−Œ, √ 7respectiv T 2‚114 4 4x − 1= 1 = 4 − √ 7, iary − 1‹min m AT2 3yOA.x − 1= 1 = 4 + √ 7.y − 1‹max m AT1 3T 2. . Q ..T 1M, 11 + √ 74Œ. Rezultă căProblema 1. Fie x, y numere reale astfel încât x 2 − xy − y 2 = 1. Determinaţiminimul şi maximul fracţiei y − 2 . Radu MironxSoluţie. Vom folosi metoda din prima soluţie. Nu putem avea x = 0 (s-ar obţiney 2 = −1, imposibil). Fie t = y − 2 ; atunci y = tx + 2 şi condiţia din ipoteză devinexx 2 (t 2 +t−1)+2x(2t+1)+5 = 0. Cum x ∈ R, impunem ca ∆ x ≥ 0 pentru t 2 +t−1 ≠ 0sau ca t 2 ±+t−1 = 0. Obţinem t ∈ [−3, 2]\¨−1 √ 5«, respectiv t = −1 ± √ 5adică,2 2y − 2în final, t ∈ [−3, 2]. Astfel,= 2, maxim atins pentru x = −1, y = 0 (carex‹maxverifică x 2 − xy − y 2 y − 2= 1), iar= −3, minim atins pentru x = 1, y = −1x‹min(care verifică x 2 − xy − y 2 = 1).Problema 2 (24739 din GM–9/2002). Aflaţi valorile extreme ale funcţiei f : R →R, f(x) = sin x − 3cos x + 2 .Paul Georgescu, Gabriel PopaSoluţie. Dacă A(−2, 3), M(cos x, sin x) sunt puncte în planul xOy, atunci f(x)este chiar panta dreptei AM. Pentru x ∈ R, punctul M parcurge cercul trigonometricşi, cum A ∈ Ext C(O, 1), rezultă că valorile extreme ale funcţei f sunt atinse atuncicând AM este una dintre tangentele duse din A la C.Dreapta prin A : y − 3 = m(x + 2) este tangentă la C dacă distanţa de la origine la|2m + 3|această dreaptă este 1. Obţinem ecuaţia √m2 + 1 = 1, cu soluţiile m 1,2 = −2± 2√ 33 .În concluzie, f min = −2 − 2√ 33şi f max = −2 + 2√ 33 .26x


Une classe spéciale de matrices carréesAdrien REISNER 1Abstract. This article studies some properties of matrices A which diagonal terms are its eigenvalues.We are interested in R-similar matrix with a triangular form (there exists P ∈ GL n (R) suchthat, P −1 AP is upper triangular) and focus on real symmetric and normal matrices -Propositions11 and 13.Keywords: eigenvalue, characteristic polynomial, symmetrical matrix, normal matrix.MSC 2000: 15A18.On désignera par K le corps R des nombres réels ou le corps C des nombrescomplexes. On considère les matrices de M n (K) ayant la propriété suivante:Définition. On dit qu ′ une matrice A = (a ij ) ∈ M n (K) possède la propriété Bou est une B-matrice, si ses termes diagonaux sont ses valeurs propres, donc si sonpolynôme caractéristique est de la forme χ A (X) = (a ii − X).nYi=1On désignera dans la suite par B n l ′ ensemble des B-matrices de M n (R).1. Exemples. Une matrice triangulaire supérieure A est évidemment une B-matrice. Si A est de plus inversible, alors son inverse est également une B-matrice.Proposition 1. Pour tout réel α, la matrice M(α) =„1 −1 α0 2 −α1 1 2 − αŽest uneB-matrice ([1], exercice 20.51, page 146).Démonstration. En effet, le polynôme caractéristique de la matrice M(α) estχ M(α) (X) = −X 3 + (5 − α)X 2 − (8 − 3α)X + (4 − 2α) = (1 − X)(2 − X)(2 − α − X),et ainsi le spectre de M(α) est Sp M(α) = {1, 2, 2 − α}, i.e. M(α) ∈ B 3 .Proposition 2. La matrice M(α) est diagonalisable si et seulement si α ≠ 1.Démonstration. Si α ≠ 0 et α ≠ 1 les trois valeurs propres de M(α) sontdistinctes et par suite la matrice est diagonalisable . Si α = 0, λ = 2 est valeur propredouble de M(0) admettant comme sous-espace propre associé le plan E 2 : x + y = 0et la matrice M(0) est encore diagonalisable (dim E 2 = 2).Pour α = 1 la matrice M(1) admet λ = 1 pour valeur propre double, alors que lesous-espace propre associé est la droite E 1 dirigée par le vecteur (1, −1, −1); dans cecas M(1) n ′ est donc pas diagonalisable (dim E 1 = 1).2. Etude de l ′ ensemble B 2 .Proposition 3. B 2 est l ′ ensemble des matrices triangulaires.1 TELECOM ParisTech; e-mail: Adrien.Reisner@telecom-paristech.fr27


Démonstration. Soit A = a bc d‹∈B 2 et Q(X) = (X−a)(X−d). Le polynômecaractéristique de la matrice A étant χ A (X) = X 2 − (a + d)X + (ad − bc), A est uneB-matrice si et seulement si Q(X) = χ A (X), c ′ est-à-dire si et seulement si bc = 0,d ′ où le résultat cherché.Corollaire. B 2 est une partie fermée de M 2 (R).Démonstration. L ′ ensemble T + des matrices triangulaires supérieures de M 2 (R)est un fermé comme sous-espace vectoriel de dimension finie de M 2 (R), et il en estde même pour l ′ ensemble T − des matrices triangulaires inférieures. B 2 est une partiefermée de M 2 (R) comme réunion des fermés T − et T + .3. Etude de l ′ ensemble B 3Proposition 4. Une B-matrice A ∈ M n (R) est inversible si et seulement si seséléments diagonaux sont tous non nuls.Le théorème est évident puisque le déterminant d ′ une B-matrice est égal au produitde ses éléments diagonaux.Proposition 5. A = (a ij ) ∈ M 3 (R) est une B-matrice si et seulement sidetA =a ii et a 12 a 21 + a 13 a 31 + a 23 a 32 = 0.3Yi=1Démonstration. La matrice A = (a ij ) ∈ M 3 (R) est une B-matrice si et seulementsi son polynôme caractéristique χ A (X) est égal au polynôme (a 11 − X)(a 22 −X)(a 33 − X). En développant ces deux polynômes et en identifiant leurs coefficientson trouve les deux conditions de la proposition.Corollaire. Soit A ∈ B 3 ∩ GL 3 (R) une matrice vérifiant: a 12 a 21 = a 13 a 31 =a 23 a 32 = 0. Alors la matrice A −1 est elle-même une B-matrice.Démonstration. Le cas trivial où la matrice est triangulaire inversible - voirProposition 4 - étant mis de coté, examinons le cas où la B-matrice A est de la forme1 a bA =„λ0 λ 2 0 det A ≠ 0. Le produit de deux matrices de ce type est0 c λ 3Žavecencore une matrice de cette même forme avec sur la diagonale le produit des termesdiagonaux correspondants. Si A est inversible, son inverse – qui est un polynômede degré 2 en A (conséquence du théorème de Cayley-Hamilton) – est encore unematrice de la même forme et elle vérifie la condition de la Proposition 5. Les autrescas s ′ étudient de manière similaire.Exemples de B-matrices de M 3 (R). Les quatre matrices suivantes sont des B-matrices:0 3−1 42 01 0A 1 =„−1−3 2 3 A 2 =„10 2 −4 A 3 =„40 2 0 A 4 =„00 0 00 0 2Ž,1 1 −2Ž,−2 −2 2Ž,1 0 1Ž.Remarque. Les matrices A 1 , A 2 et A 3 sont inversibles. Les inverses des matricesA 1 et A 3 sont elles-mêmes des B-matrices compte tenu du corollaire précédent.28


4. Un exemple de B-matrice de M 4 (R). On se propose de trouver des B-matrices de M 4 (R) de la forme M = A B0 C‹(1), où les éléments des matrices carréesA, B et C sont tous non nuls.On utilise le lemme suivant:Lemme.matrices carrées.Dans M n (R) on a : detDémonstration immédiate, puisqueC ∈ M s (R).A BA det C, A et C étant des0 C‹=detA B0 C‹= I r 0 BA ∈ M0 C‹A0 I r (R),s‹oùSoit alors une B-matrice M de la forme (1). Compte tenu du lemme précédent, lepolynôme caractéristique de M est: χ M (X) = χ A (X)χ C (X). Posons A = a bc d‹,C = e f Si a ou d est valeur propre de A alors χg h‹. A (X) est scindé et tr A = a + d,les valeurs propres de A sont a et d. La matrice A est alors une B-matrice et d ′ aprèsla Proposition 3 c ′ est une matrice triangulaire ce qui est impossible car A ne contientaucun élément nul. Donc les valeurs propres de A sont e et h et les valeurs propresde C sont a et d, i.e.: χ A (X) = (X − e)(X − h) et χ C (X) = (X − a)(X − d). Endéveloppant ces polynômes et en identifiant leurs coefficients on obtient les relations:a + d = e + h, ad − bc = eh, eh − gf = ad. Pour trouver une B-matrice de laforme (1) où les éléments des matrices carrées A, B et C sont tous non nuls, il suffitdonc de trouver des réels a, b, c, d, e, f, g et h tous non nuls vérifiant ces équationset de prendre une matrice B quelconque ne contenant aucun terme nul. On peutconsidérer par exemple les matrices suivantes: A = 1 2 C =2 1‹et 3 2−2 −1‹ouencore: A = 1 −1 C =1 3‹et 2 11 2‹.Remarque ([1], page 146). Plus généralement, une matrice de M n (R) trigonaleen blocs avec des blocs diagonaux qui sont un mélange des matrices triangulairessupérieures ou inférieures fournit un exemple non trivial de B-matrices.5. Quelques propriétes de B-matrices de M n (R)Proposition 6. A = (a ij ) ∈ M n (R) étant une B-matrice pour tout couple deréels (a, b) les matrices aA + bI n et a t A + bI n sont encore des B-matrices.Démonstration. A étant une B-matrice ses valeurs propres sont a 11 , a 22 , . . . , a nn .Les valeurs propres des matrices aA + bI n et a t A + bI n sont alors (aa ii + b) i:1,...,n : cesont les termes diagonaux de ces matrices, d ′ où la Proposition 6.Proposition 7. Le sous-ensemble des B-matrices de GL n (R) est un ensembledense dans B n .29


Démonstration. Soit A = (a ij ) ∈ B n . Pour k assez grand (k ≥ K 0 ) a ii + 1 k ≠ 0pour tout i : 1, . . . , n. La suite de matrices (A k ) k≥K0 où A k = A + 1 k I n est une suitede B-matrices, d ′ après la Proposition 6, inversibles, car aucun terme diagonal nul, etqui converge vers A, d ′ où la Proposition 7.Remarque. Une matrice trigonalisable n ′ est pas nécessairement une B-matrice.La matrice A = 0 11 0‹est une matrice réelle symétrique donc diagonalisable et aussitrigonalisable mais d ′ après la Proposition 3 elle n ′ est pas une B-matrice. La matricede rang 1, A = 1 11 1‹est diagonalisable – donc à fortiori trigonalisable – son spectreétant Sp A = {0, 2} mais elle n ′ est pas une B-matrice.Proposition 8. Une B-matrice est trigonalisable.Démonstration. Une B-matrice admet un polynôme caractéristique sur K etpar suite elle est trigonalisable.Proposition 9. Une matrice A ∈ M n (R) est semblable à une B-matrice si etseulement si son polynôme caractéristique est scindé sur R. Autrement dit les matricesréelles qui sont semblables à des B-matrices réelles sont les matrices trigonalisablesdans R.Démonstration. Si A est semblable à une B-matrice B alors χ A (X) = χ B (X)et χ B (X) est scindé; il en est donc de même pour le polynôme χ A (X). Inversement,si χ A (X) est scindé, A est semblable à une matrice triangulaire et toute matricetriangulaire est une B-matrice.Proposition 10. Toute matrice A ∈ M n (R) est somme de deux B-matrices. B nn ′ est pas un sous-espace vectoriel de M n (R).Démonstration. C ′ est évident puisque toute matrice est la somme d ′ une matricetriangulaire supérieure et d ′ une matrice triangulaire inférieure et chacune deces matrices est une B-matrice. Il en découle que M n (R) n ′ est pas un sous-espacevectoriel de M n (R) puisqu ′ il existe (dès que n ≥ 2) des matrices qui ne sont pas desB-matrices.Proposition 11. Les B-matrices symétriques réelles sont les matrices diagonales.La seule B-matrice antisymétrique réelle est la matrice nulle.Démonstration. a) Soit A = (a ij ) une matrice symétrique réelle. A est orthodiagonalisable,i.e. A est semblable à une matrice diagonale D = diag(λ 1 , λ 2 . . . λ n ),et il vient: trA 2 = trA t A = nPi=1anPj=12 ij = nPi=1 λ 2 i . La première partie de la Proposition11 en découle immédiatement.b) Soit A = (a ij ) ∈ M n (R) une B-matrice antisymétrique. Son spectre est réduit à{0} et par suite son polynôme caractéristique est X n . D ′ après le théorème de Cayley-Hamilton, A vérifie A n = 0, et ( t AA) n = (−A 2 ) n = (−1) n A 2n = 0. Or, la matricet AA est diagonalisable en tant que matrice symétrique réelle. Comme ( t AA) n = 0,30


toutes ses valeurs propres sont nulles. La matrice t AA est semblable à la matricenulle donc elle est nulle. On en déduit: tr( t AA) = 0 soitnPi=1anPj=12 ij = 0 et par suiteA est la matrice nulle.Proposition 12. La dimension maximale d ′ un sous-espace vectoriel F de M n (R)n(n + 1)tel que F ⊂ B n est .2Démonstration. Compte tenu de la Proposition 11, on a: F ∩ A n = {0} où A nest l ′ ensemble des matrices antisymétriques de M n (R). Il vient alors:dim F + dim A n = dim(F + A n ) ≤ dim M n (R) = n 2 ,d ′ où dim F ≤ n 2 − dim A n = n 2 n(n − 1) n(n + 1)− = . La Proposition 12 résulte2 2alors du fait que B n contient le sous-espace vectoriel des matrices triangulaires supérieures(inférieures).On se propose de trouver un sous-espace vectoriel F contenu dans B n , n > 2, dedimension maximale qui n ′ est pas constitué uniquement des matrices triangulaires.Soit F l ′ ensemble des matrices M de M n (R) de la forme M = a B a ∈ R, B ∈0 C‹oùM 1,n−1 (R) et C une matrice triangulaire inférieure d ′ ordre n − 1. De façon évidente,n(n − 1) n(n + 1)F est un sous-espace vectoriel de dimension dim F = 1+(n−1)+ = .2 2Comme (a) et C sont des B-matrices, il en résulte, compte tenu du Lemme de la page29, que la matrice M est elle-même une B-matrice – voir Remarque de la partie 4 – .6. B-matrices complexes. On rappelle qu ′ une matrice A ∈ M n (C) est normalelorsque AA ∗ = A ∗ A où A ∗ = t A est l ′ adjoint de la matrice A. Le même raisonnementque celui utilisé à la Proposition 11 permet de démontrer la propriété suivante decertaines B-matrices complexes.Proposition 13. Les deux assertions suivantes sont équivalentes: i) A est uneB-matrice normale, ii) A est une matrice diagonale.Je tiens à remercier ici mes collègues Mmes Chantal Leduc, Fadila Tradi,MM. Zahir Abela, Olivier Allain, Hatem Abichou, Gabriel Amegandji, PatrickBourgeois, Jérôme Neveu, Pierre Ortuno, Olivier Papillon et les autres pour le soutienqu ′ ils m ′ ont apporté à l ′ écriture de mes articles.Références1. R. Mneimné – Réduction des endomorphismes, Ed. Calvage & Mounet, Paris, 2006.2. J.M. Arnaudiès, H. Fraysse – Cours de mathématiques, Tome 4: Algèbre bilinéaireet géométrie, Ed. Dunod, Paris, 1990.3. J. Fresnel – Algèbre des matrices, Ed. Hermann, Paris, 1997.31


Concursul Internaţional de Matematică,,Vladimir Andrunachievici”Ediţia a II-a, Chişinău, 5 ianuarie 2012Clasa a VI-a1. Dacă la un număr natural se adună suma cifrelor lui, atunci se obţine numărul2012. Să se determine toate numerele naturale cu această proprietate.2. În sistemul zecimal, cifrele x şi y şi numărul natural n satisfac egalitateaxy + yx = 1 + 2 + ... + n.Să se determine toate valorile lui n şi toate numerele xy care satisfac această egalitate.3. Să se determine 15 numere naturale nenule cu proprietatea că, dacă fiecare dinele este mărit cu 1, atunci produsul tuturor numerelor mărite este de 2012 ori maimare decât produsul numerelor iniţiale.4. Pe tablă sunt scrise numerele 1, 2, 3, ..., 33. Un elev efectuiază următoareaoperaţie: alege două numere astfel încât unul din ele să fie multiplu al celuilalt şi apoile înlocuieşte cu câtul lor. Elevul repetă operaţia până când niciun număr de pe tablănu este multiplu al altuia. Să se determine câte numere rămân pe tablă în situaţia încare elevul nu mai poate repeta operaţia.Clasa a VII-a1. Să se determine toate perechile (m, n) de numere întregi care satisfac ecuaţia(m + n) · (m + n + 2) = 23 · 3 |m−n| + 1.2. Un pătrat de dimensiuni 8 × 8 este divizat în 64 pătrăţele de dimensiuni 1 × 1.Care este numărul maxim de diagonale care pot fi duse în pătrăţelele 1×1 astfel încâtoricare două diagonale să nu aibă puncte comune?3. Fie p un număr prim. Printre numerele naturale n de forman = (p 2 + 32) 2 − 69 · (p 2 + 32) + 2250,să se determine numărul cu cea mai mică sumă posibilă a cifrelor lui.4. Numerele a, b, c sunt elemente ale mulţimii {1, 2, ..., 2012} . Să se determinenumărul tuturor tripletelor ordonate diferite (a, b, c) cu proprietatea că suma a+b+ceste un multiplu al fiecărui din numerele a, b, c. (Două triplete <strong>format</strong>e din aceleaşitrei numere sunt triplete ordonate diferite dacă ordinea numerelor în aceste tripleteeste diferită.)32


Clasa a IX-a1. Să se determine cel mai mic număr natural nenul n astfel încât numerele14n, 16n, 18n, 20n să aibă acelaşi număr de divizori.2. Fie paralelogramul ABCD cu unghiul ABC obtuz. Punctul P interior triunghiuluiBDC este situat pe diagonala (AC) astfel încât m (∠BP D) = m (∠ABC) .Să se demonstreze că dreapta CD este tangentă la cercul circumscris triunghiuluiBCP dacă şi numai dacă |AB| = |BD| .3. Să se determine toate perechile (a, b) de numere naturale care satisfac ecuaţia4. Fie numerele reale4 a + 4a 2 + 4 = b 2 .A =È10 + √ 1 +È10 + √ 2 + ... +È10 + √ 99,B =È10 − √ 1 +È10 − √ 2 + ... +È10 − √ 99.Să se arate că numărul A B − √ 2 este natural.Clasa a XII-a1. Toate elementele mulţimiiA = {p, 3p + 2, 5p + 4, 7p + 6, 9p + 8, 11p + 10}sunt numere prime. Să se arate că numărul 17p + 2012 este compus.2. Mulţimea nevidă A conţine m numere naturale pare nenule, iar mulţimeanevidă B conţine n numere naturale impare astfel încât suma elementelor din ambelemulţimi este egală cu 2012. Să se afle cea mai mare valoare posibilă a sumei 3m + 4n.3. Să se calculeze integrala Riemann a funcţieif :0, π 2→R,f(x) = e x · cos 2x + √ 2 cos€x + π 4ŠÈ(1 + sin x) (1 + cos x) .4. Şirul de numere întregi pozitive (a n ) n∈N ∗ satisface condiţiile: a n este un multiplual lui n şi |a n − a n+1 | ≤ 5. Să se determine cea mai mare valoare posibilăa lui a 1 .| {z }nAsupra numerelor 4 din expresia4 4 . . . 4 = 28 (n ≥ 2)utilizaţi diverse operaţii uzuale cu numere pentru a o transforma într-o egalitate.N.B. Răspunsul se găseşte la pag. 37.33


Condiţii de simetrie relativ la punctele O, H, G, ITemistocle BÎRSAN 1Abstract. In this paper, the author argues on the actual possibility of a highschool pupil towork out an own paper inside the bounds of his knowledge. This point of view is supported on thebasis of an elementary study on certain symmetry properties of the O, H, G, I points of a triangle.Keywords: circumcenter, orthocenter, centroid, incenter.MSC 2000: 51M04.În rândurile care urmează, ne vom adresa acelor elevi cu talent aflaţi la primatentativă de obţinere a unei note matematice. Aceştia se întreabă în mod firesc: Deunde iau un subiect? Pot să duc la capăt subiectul ales? Răspunsul este: DA, cusiguranţă DA!Cunoştinţele dobândite din manuale sunt suficiente pentru ca un elev să poatăiniţia şi realiza o notă matematică proprie. Dacă elevul are şi o oarecare practică larevistele de matematică şi o experienţă de participant la concursuri şi olimpiade, notase poate ridica la un nivel superior de calitate.Subiectul unei Note poate fi sugerat de profesor, dar poate fi găsit şi de elevulînsuşi dacă are puţină curiozitate şi iniţiativă. Ca model, vom formula o întrebare(adică, vom introduce un subiect) pe care şi-ar putea-o pune orice elev; mai mult,tratarea completă a subiectului este elementară şi uşor de făcut de orice rezolvitor deprobleme propuse în reviste.Ne referim la punctele O, H, G, I ale unui triunghi. Din curiozitate, ne întrebăm:pot fi simetrice faţă de o latură a triunghiului sau faţă de un vârf al său două dintrepunctele O, H, G, I?Imediat, cu câteva observaţii simple aducem clarificări în privinţa subiectului:1) Punctele G şi I sunt întotdeauna interioare triunghiului, deci perechea (G, I)nu poate avea niciuna dintre proprietăţile de simetrie menţionate.2) Relativ la punctele O şi H ştim că: i) sunt în interiorul triunghiului, dacă acestaeste ascuţitunghic; ii) sunt în exteriorul triunghiului, dacă acesta este obtuzunghic;iii) în cazul triunghiului dreptunghic H coincide cu vârful unghiului drept, iar O estemijlocul ipotenuzei. Rezultă că numai pentru triunghiurile obtuzunghice pot existaperechi de puncte simetrice.3) Este utilă şi observaţia: dacă △ABC este obtuzunghic cu m(bA) > 90 ◦ , atunci Hse află în unghiul opus la vârf unghiului obtuzbA şi O se află în semiplanul determinatde dreapta BC ce nu conţine vârful A. Ca urmare, avem: i) simetrice faţă de vârfulA pot fi punctele din perechile: (H, G), (H, I) şi (H, O); ii) simetrice faţă de dreaptaBC pot fi cele din perechile: (O, G), (O, I) şi (O, H).4) În sfârşit, constatăm uşor că △ABC, m(bA) > 90 ◦ , este isoscel, cu AB = AC,dacă una dintre cele şase perechi rămase în discuţie are proprietatea de simetrie1 Prof. dr., Univ. Tehnică ,,Gh. Asachi”, Iaşi34


corespunzătoare. În acest caz, punctele O, H, G, I se află pe axa AA ′ (A ′ noteazămijlocul laturii [AB]) şi avem ordinele: A ′ − A − H şi A − A ′ − O.Aşadar, mulţimea triunghiurilor obtuzunghice şi isoscele este cadrul firesc al problemeipropuse. Aceasta se reduce la şase probleme simple, uşor de rezolvat de cătreun elev care are cunoştinţe de trigonometrie. Le vom aborda pe rând.Propoziţia 1. Dacă H şi G sunt simetrice faţă de vârful A al △ABC, atuncitriunghiul este obtuzunghic isoscel şi având cos A = − 1 4 (echivalent, 2a2 = 5l 2 , undea şi l notează lungimea bazei, respectiv a laturii triunghiului).Demonstraţie (Fig. 1). Faptul că △ABC este obtuzunghic isoscel a fostdeja stabilit. Condiţia de simetrie AH = AG se mai scrieHA ′ H − A ′ A = 2 3 A′ A, deci A ′ H = 5 3 A′ A saua2 ctg π 2 − A 2‹= 5 3 · a2 tg π 2 − A 2‹.Ca urmare, tg 2 A 2 = 5 3 şi, deci, cos A = −1 4 .Pe de lată parte, utilizând faptul că în △AA ′ B avemsin A 2 = a 2l , obţinem cos A = −1 4 ⇔ 1 − cos A = 5 4 ⇔B.A.GAOFig.1C2 sin 2 A 2 = 5 4 ⇔ a22l 2 = 5 4 ⇔ 2a2 = 5l 2 , ceea ce încheie demonstraţia.Propoziţia 2. Dacă O şi G sunt simetrice faţă de latura [BC] a △ABC, atuncitriunghiul este obtuzunghic isoscel şi îndeplineşte condiţia cos A = − 1 4 (echivalent,2a 2 = 5l 2 ).Demonstraţie (Fig. 1). Condiţia de simetrie A ′ O = A ′ G este echivalentă cuOB = BG, adică BG = R (raza cercului circumscris △ABC). În △BA ′ G avem:BG 2 = A ′ B 2 + A ′ G 2 , deci R 2 = a24 + A′ A3‹2. Cum 2R = aobţinem relaţiasin A şi A′ A = a 2 ctg A 2 ,1sin 2 A = 1 + 1 A 9 ctg2 2 echivalentă cu 9 cos2 A = 4 cos 4 A 2 . Extrăgândrădăcina pătrată, urmează: −3 cos A = 2 cos 2 A 2− 1 4 etc.⇔ −3 cos A = 1 + cos A ⇔ cos A =Observaţie. Comparând rezultatele precedente, constatăm că punctele H şi Gsunt simetrice fată de vârful A al △ABC dacă şi numai dacă O şi G sunt simetricefaţă de BC. Faptul se explică simplu dacă ţinem seama că G se află pe dreapta luiEuler şi HG = 2OG : AH = HG ⇔ HG = 2AG ⇔ 2OG = 2AG ⇔ OG = AG ⇔OG = 2A ′ G ⇔ A ′ O = A ′ G.Cu mijloace asemănătoare, vom stabili condiţiile în care punctul I este simetricullui H faţă de A sau el este simetricul lui O faţă de BC.35


Propoziţia 3. Dacă punctele H şi I sunt simetrice faţă de vârful A al △ABC,atunci triunghiul este obtuzunghic isoscel şi sin A 2 = 1 8 (√ 33−1) (echivalentă cu relaţia4a=( √ 33 − 1)l).Demonstraţie (Fig. 2). Condiţia AH = AI se scrie în forma A ′ H − A ′ A =A ′ A − A ′ I sau A ′ H = 2A ′ A − A ′ I sau, încă,a2 ctg π 2 − A 2‹=2 a 2 ctg A 2 − a 2 ctg 1 2π2 − A 2‹.Cu calcule simple se obţineHtg A 2 = 2 ctg A 2 − 1 + sin A 2cos A 2de unde, înmulţind ambii membri cu sin A 2 cos A 2 , deducem,B.A. AIC(∗) 4 sin 2 A 2 + sin A 2 − 2 = 0,OFig.2relaţie echivalentă cu sin A 2 = 1 8 (√ 33 − 1).În sfârşit, cu ajutorul relaţiei sin A 2 = a 2l (△AA′ B), se arată uşor că (*) esteechivalentă cu 4a = ( √ 33 − 1)l.Propoziţa 4. Dacă punctele O şi I sunt simetrice faţă de BC, atunci △ABCeste obtuzunghic isoscel cu A = 3π 5 (echivalent, a √5 + 1l = (secţiunea de aur)).2Demonstraţie. Condiţia de simetrie A ′ O = A ′ I revine la BI = R. În △BA′ Iavem R 2 = r 2 + a2, unde r este raza cercului înscris în △ABC. Ţinând seama de4relaţiile R =a2 sin A şi r = a 2 tg B 2 (△BA′ I), obţinem:(∗∗)1sin 2 A = tg3 B 2 + 1 ⇔ sin2 A = cos 2 B 2 ⇔ sin A = cos B 2 .Cum B + A 2 = π 2 (△AA′ B), ultima relaţie devine sin A = cosdeterminarea lui A avem ecuaţiasin A = sinπ 4 + A 4‹, A ∈π2 , π.π4 − A 4‹şi pentruObservăm că A = π 4 + A 4 , adică A = π , nu convine problemei, iar A +π 34 + A 4‹=πconduce la soluţia A = 3π 5 . 36


Vom exprima această condiţie în funcţie de laturi scriind ultima relaţie (**) înforma 8 sin 2 A A 2 cos2 2 = 1 + cos B şi înlocuind în aceasta sin A 2 = cos B = a 2l şicos A 2 =r1 − a2; vom obţine 2a24l2 l 21 −2l=1 a sau, cu t = a l , ecuaţia 2t2 1 −2‹=t1. Ecuaţia în t este echivalentă cu (t − 1)(t 2 − t − 1) = 0 şi, cum t ≠ 1, are soluţiaadmisibilă t = a l = 1 + √ 5.2A mai rămas de considerat perechea (O, H) : punctele O şi H pot fi simetrice faţăde un vârf al triunghiului? Dar faţă de o latură? Lăsăm cititorul să dovedească (cumijloace de acealaşi fel sau altele!) următoarele afirmaţii:Propoziţia 5. Dacă punctele O şi H sunt simetrice faţă de vârful A al △ABC,atunci triunghiul este obtuzunghic isoscel şi A = 2π (i.e., dacă centrul cercului lui3Euler cade într-un vârf al △ABC, atunci triunghiul este obtuzunghic isoscel cu acelvârf de 120 ◦ ).Propoziţia 6. Oricare ar fi un triunghi, punctele O şi H nu pot fi simetrice faţăde vreuna din laturile sale.Observaţii. 1) Fiecare dintre condiţiile de simetrie impuse în Propoziţile1-5 determină în mod unic (până la asemănare de triunghiuri) un anumit triunghiobtuzunghic isoscel. În total se obţin patru astfel de triunghiuri (v. Observaţia dedupă Propoziţia 2).2) Propoziţiile 1-5 admit şi reciproce valabile; altfel spus, condiţiile de simetrieimpuse sunt în fiecare caz şi necesare, nu numai suficiente (verificaţi!).Elevul care a parcurs atent rândurile acestei Note va constata că s-a confruntat cuprobleme mult mai dificile cu prilejul concursurilor şcolare sau ca rezolvitor de problemepropuse în diverse reviste. Dificultatea se află în altă parte: subiectul de lucru,tema de studiu. Nu este de ajuns să ai cunoştinţe temeinice şi să fii un bun rezolvitorde probleme! Va trebui să fii în posesia unui subiect de lucru, ce poate apărea (seiveşte!) în felurite moduri: din curiozitate, dintr-o inspiraţie fericită, întâmplător,graţie unei intuiţii profunde, sugerat de o problemă sau un articol etc. Cu timpul,producerea de subiecte devine o obişnuintă şi, confruntat cu o ,,inflaţie” de subiecte,apare necesitatea selectării şi abordării doar a celor mai valoroase dintre ele.Răspuns la ,,recreaţia” de la pag. 33.|{z}2(k−1)Avem: a) pentru 44 = 28 putem scrie 4! + 4 = 28; b) pentru 444 = 28 putem scrie4! + √ 4 + √ 4 = 28; c) pentru n = 2k (par) gândim expresia ca fiind 4 4 4 . . . 4 = 28,reducem la cazul a) şi scriem 4! + 4 + (4 − 4) + . . . + (4 − 4) = 28 (k − 1 parantezemici); d) pentru n = 2k + 1 (impar) reducem la cazul b) şi obţinem 4! + √ 4 + √ 4 +(4 − 4) + . . . + (4 − 4) = 28 (k − 1 paranteze mici).37


Aplicaţii ale teoremei lui Dirichlet la calcul de limiteGeanina HĂVÂRNEANU 1Abstract. In this Note, Dirichlet ′ s theorem in the theory of Fourier series is applied for calculatinga couple of limits of certain sequences: (6) - (13).Keywords: even and odd fuctions, Dirichlet ′ s theorems.MSC 2000: 28A03.Sunt binecunoscute câteva procedee de calculare a limitelor unor şiruri de unanumit tip cu ajutorul integralei definite. În cele ce urmează, ne vom referi la unuldintre aceste procedee.Fie f : (−π, π) → R o funcţie integrabilă şi şirurile (a k ) k∈N şi (b k ) k∈N ∗ date de(1) a k =πZπ1 f(x) cos kxdx, k ∈ N,−πb k =πZπ1 f(x) sin kxdx, k ∈ N ∗ .−πFie, de asemenea, şirul (S n (x)) n∈N ∗definit prin(2) S n (x) = a 02nXk=1+ (a k cos kx + b k sin kx), n ∈ N ∗ .Teorema lui Dirichlet. Dacă f : (−π, π) → R este o funcţie derivabilă şi cuderivată continuă, atunci şirul (S n (x)) n∈N ∗ are limită şi(3) lim S x ∈ (−π, π)n(x) =¨f(x),n→∞ 12 [f(−π + 0) + f(π − 0)] , x = ±π .Observaţii. 1) Dacă, în plus, fforma mai simplăeste o funcţie pară, atunci formulele (1) capătă(4) a k =πZπ2 f(x) cos kxdx, k ∈ N, b k = 0, k ∈ N ∗ .02) Dacă f este şi impară, formulele (1) se scriu(5) a k = 0, k ∈ N, b k =πZπ2 f(x) sin kxdx, k ∈ N ∗ .Aplicaţii. I. Fie f : (−π, π) → R , f(x) = x , care este impară. Conformformulelor (5), avem: a k = 0, k ∈ N, şi b k =πZπ2 x sin kxdx = − 2x cos kxπ+0kπ02cos kx dx = (−1)kπZπk+1 20k , k ∈ N∗ . Aşadar, S n (x) = 2 nPk=1k+1sin kx(−1) .k1 Profesoară, Liceul Teoretic ,,Al.I. Cuza”, Iaşi038


Pentru x = π 2 ∈ (−π, π), avem: S nπ2=2 nPk=1(−1) k+1 sin k π 2. Aplicând teoremaklui Dirichlet, deducem că lim S n(x) = f( π ), deci limn→∞ 2 S 2n(x) = πn→∞ 2 , adică(6) limn→∞1 − 1 3 + 1 5 − 1 7 + · · · + (−1)n−12n − 1= π 4 .II. Fie funcţia pară f : (−π, π) → R, f(x) = x 2 . În acest caz, ţinând seamade relaţiile (4), obţinem: b k = 0, k ∈ N ∗ , a 0 =πZπ2 x 2 dx = 2π2 şi a k =32πZπ0x 2 cos kxdx = 2x2kπsin kxπ−kπZπ 400x sin kxdx = 4xk 2 πkcos kx0cos kxπ0− 4kπZπ0cos kxdxk= (−1) k 4 k 2 , k ∈ N. Aşadar, S n(x) = π23 + 4 nPk=1(−1)k 2 .a) Pentru x = 0 ∈ (−π, π), obţinem S n (0) = π23 + 4 nPk=1(−1) kk 2 . Conform teoremeilui Dirichlet, lim S n(0) = f(0), cu alte cuvinten→∞(7) lim −n→∞1 1 2 2 + 1 3 2 − 1 =4 2 + · · · + (−1)n−1 π2n 2 12 .b) Pentru x = π obţinem S n (π) = π23 + 4 nPk=11. Aplicând teorema lui Dirichlet,k2 deducem că lim S n(π) = 1 [f(−π + 0) + f(π − 0)], decin→∞ 2(8) limn→∞1 + 1 2 2 + 1 3 2 + 1 4 2 + · · · + 1 π2n2‹= 6 .Observaţie. Coroborând relaţiile (7) şi (8), deducem că(9) lim 1 + 1n→∞ 3 2 + 1 5 2 + 1 7 2 + · · · + 1 π2(2n − 1) 2‹=8 .III. Fie funcţia f z : (−π, π) → R, f z (x) = cos zx, derivabilă, cu derivata continuăşi care este pară. Avem: b k = 0, k ∈ N ∗ , a 0 =πZπ2 2 sin πzcos zxdx = şiπza k =πZπ2 cos zx cos kxdx = 1 [cos(z − k)x + cos(z + k)x] dx =0πZπ0=π•sin(z 1 − k)x sin(z + k)x˜π2z sin(z + k)π+ =z − k z + kπ (z 2 − k 2 , k ∈ N ∗ .)00Aşadar, S n (x, z) =sin πzπz+ 2zπnPk=1sin(z + k)π cos kxz 2 − k 2 .39


sin πza) Pentru x = 0 ∈ (−π, π), obţinem S n (0, z) = + 2zπz πCum lim S n(0, 1n→∞ 2 ) = f 1 (0), deducem că2nPk=1sin(z + k)πz 2 − k 2 .(10) limn→∞12 2 − 1 − 14 2 − 1 + 16 2 − 1 − · · · + (−1)n−14n 2 − 1= π 4 − 1 2 .b) Pentru x = π avem S n (π, z) =sin πzπz+ 2zπnPk=1(−1) k sin(z + k)πz 2 − k 2 .Cu teorema lui Dirichlet deducem că lim S n π, 1n→∞ 2‹= 1 12”f (−π + 0) + f 1 (π − 0)—,2 2deci lim S n(π, 1 ) = 0. În cele din urmă, obţinem căn→∞ 2(11) limn→∞12 2 − 1 + 14 2 − 1 + 16 2 − 1 + · · · + 14n 2 − 1‹= 1 2 .Acest rezultat se poate obţine uşor observând că suma dintre paranteze este telescopică.c) Pentru x = π 2 şi z = 1 avem, conform teoremei lui Dirichlet, lim2f 12πAşadar, avem lim2. S π4nn→∞ 2 , 1 √2, care revine la2‹=2n→∞ S nπ2 , 1 2‹=(12) limn→∞14 2 − 1 − 18 2 − 1 + · · · + 1(8n − 4) 2 − 1 − 1(8n) 2 − 1‹= π√ 2− 1 8 2 .IV. Fie funcţia f z : (−π, π) → R, f z (x) = sin zx, derivabilă, cu derivata continuăşi care este impară. În acest caz avem: a k = 0, k ∈ N, şi b k =πZπ2 sin zx sin kxdx =01[cos(z − k)x − cos(z + k)x] dx =πZπ1 − k)x sin(z + k)x−0π•sin(z˜π=z − k z + k02k sin(z + k)ππ (z 2 − k 2 , k ∈ N ∗ . Ca urmare, S n (x, z) = 2 k sin(z + k)π sin kx)πnPk=1 z 2 − k 2 .Pentru x = π 2 ∈ (−π, π), obţinem S nπ2 , z= 2 k sin(z + k)π sin kπnPk=1π 2z 2 − k 2 .Aplicând din nou teorema lui Dirichlet, rezultă că lim S πnn→∞ 2 , 1 12‹=f2πdeci2şi(13) limn→∞‚12 2 − 1 − 36 2 − 1 + 510 2 − 1 − · · · + (−1)n−1 (2n − 1)[2(2n − 1)] 2 − 1Œ=√2π16 .40


Colegiul Naţional ,,Mihai Eminescu” din Iaşi(1865-2012)– 147 de ani de tradiţie şi excelenţă în educaţie –Legea asupra instrucţiunii din noiembrie 1864 prevedea la articolul 184: În toateoraşele unde sunt licee de băieţi, se vor stabili şcoli secundare de fete compuse dincinci clase. Pe această bază, ia fiinţă Şcoala Secundară de Fete sau ExternatulSecundar de Fete, care îşi începe cursurile în ianuarie 1865 cu o singură clasă<strong>format</strong>ă din 12 eleve care proveneau din medii foarte diferite, două dintre ele fiindorfane. Primele obiecte de studiu au fost matematica, limba română, ştiinţele naturii,geografia, istoria, limba franceză, religia, desenul, caligrafia şi lucrul manual.În 1875 Externatul Secundar de Fete a fost trans<strong>format</strong> în şcoală profesională,pentru ca, şapte ani mai târziu, în 1882, să îşi reia cursurile sub vechea denumire,Externatul Secundar de Fete. În perioada 1893-1896, în cadrul şcolii a funcţionato Secţie normală care pregătea învăţătoare suplinitoare. În 1905 Externatul Secundarde Fete a fost trans<strong>format</strong> în liceu cu opt clase, cu numele de Şcoala Secundarăde Fete. Prin decretul 2688/5.10.1908 Ministerul Instrucţiunii şi al Cultelor,condus de Spiru Haret, aprobă noua denumire – Şcoala Secundară de Fete,,Oltea Doamna”. Acest nume a fost cerut de directoarea de atunci a şcolii, TerezaStrătilescu, ca un omagiu adus mamei domnitorului Ştefan cel Mare, cu ocazia descopeririipietrei funerare a Doamnei Oltea şi aşezării acesteia în Mănăstirea Probota.41


Sub această denumire şcoala devine liceu clasic-modern cu opt clase, corespunzător culiceul modern al şcolilor de băieţi. Ziua şcolii urma să fie sărbătorită la 4 noiembrie.Din anul 1925, absolventele au susţinut examenul de bacalaureat care le permiteaaccesul la studiile universitare. În 1945, şcoala a primit denumirea de Liceul Teoreticde Fete nr. 1, pentru ca din 1952 şcoala să funcţioneze sub denumirea deŞcoala Medie de zece ani nr. 1, apoi, din 1956, Şcoala medie nr. 2.Prin ordinul 3320/20.08.1960, Ministerul Învăţământului şi Culturii decide caşcoala să se numească Liceul nr.2 ,,Mihai Eminescu”. Numele de Liceul Teoretic,,Mihai Eminescu” a fost în vigoare din 1990 până la 1 septembrie 2008, datăla care devine Colegiul Naţional ,,Mihai Eminescu” Iaşi.Pe lângă numeroasele schimbări ale denumirii şi profilului, şcoala a trecut şiprin dese schimbări de sediu. Iniţial, şcoala a funcţionat la un loc cu Şcoala Primarădin Sărărie, pentru ca în lunile aprilie-octombrie 1865 să închirieze casa Catincăi Dumitriudin apropierea bisericii Vulpe. Anul următor, Externatul Secundar a funcţionatîntr-o clădire din curtea bisericii Bărboi şi apoi în casele Donici, din strada Sărărie.Între 1867 şi 1882, şcoala a funcţionat în clădirea din curtea bisericii Sf. Sava, apoi,vreme de cinci ani, Externatul s-a instalat în casele fostului pensionat francez Jordandin strada Sărărie, după care, din 1887 până în 1896, şcoala a închiriat o proprietatea lui Alecu Şendrea, casa în care s-a născut poetul Vasile Alecsandri, pe stradelaVasile Alecsandri, loc pe care până de curând se afla Muzeul Teatrului. Din 1896,şcoala închiriază casele Bălaşei Şendrea din strada Procojnie, unde rămâne timp depatru ani.Aprobată în 1898, construcţia unui nou local se încheie în 1900, când esteinaugurată clădirea din curtea Mânăstirii Golia. La 19 august 1909, clădirea esteînsă distrusă de un puternic incendiu. Din 27 octombrie 1909 şi până azi, şcoala îşidesfăşoară activitatea în clădirea din str. Mihail Kogălniceanu, fostă proprietate alui Alexandru Mavrocordat unde funcţionase anterior o şcoală de băieţi – InstituteleUnite, şcoală la care a predat pentru scurt timp lecţii de logică Mihai Eminescu.În timpul primului război mondial, cursurile au fost întrerupte din noiembrie1916 până în noiembrie 1917, timp în care localul a fost trans<strong>format</strong> în spital militar.În anul şcolar 1918-1919, cursurile s-au desfăşurat în clădirea Seminarului Pedagogicdin strada Toma Cozma nr.2, pentru ca abia în august 1919 să revină în localul dinstrada Mihail Kogălniceanu.După încercările comitetului şcolar, din care făcea parte în calitate de cenzorMihail Sadoveanu, de a cumpăra casa Mavrocordat, la insistenţele directoarei deatunci, Tereza Strătilescu, ministrul Spiru Haret aprobă achiziţia clădirii care astfeldevine proprietatea şcolii în 1920. Doi ani mai târziu, în clădirea de lângă poartadin strada Muzelor s-a amenajat internatul, apoi, în anul 1925, s-a construit alăturide clădirea principală o sală folosită pentru gimnastică şi festivităţi, iar între 1935 şi1937 s-a construit o nouă clădire, cu 8 săli de clasă şi un laborator, dată în folosinţă la15 decembrie 1938. În timpul celui de-al doilea război mondial, liceul a fost evacuatîn comuna Balinţ, Lugoj, revenind în clădirea proprie în februarie 1945. În 1970,liceul a obţinut o nouă clădire, care servise anterior liceului pedagogic.De-a lungul celor 147 de ani de existenţă, şcoala a avut 16 directori, un singurbărbat. Prima directoare, Smaranda Macry a fost încadrată în decembrie 1864 şi a42


condus şcoala până în septembrie 1866. Din septembrie 1866 şi până în martie 1882,şcoala a fost condusă de Cleopatra Petit, profesor de limba franceză. După douăluni de direcţie a doamnei Victoria A. Urechia, în octombrie 1882 a fost numităîn funcţie Maria Amfilochi Xenopol, care a condus instituţia timp de douăzeci deani. Din ianuarie 1902 şi până în septembrie 1929, direcţia şcolii este deţinută deTereza Strătilescu, profesor de istorie, personalitate deosebită, remarcabil talentpedagogic. De numele ei se leagă momentele cele mai importante din istoria şcolii.Au urmat, pe rând, Margareta Roiu, profesor de matematică, Clemansa Grecea,profesor de istorie, Elena Partenie, profesor de fizică şi chimie, Georgeta Hagi,profesor de geografie, Debora Leibovici, Ana Aurora Goia, profesor de germană şifranceză, Margareta Năstase, profesor de chimie, Crenguţa Gâldău, profesor delimba română, Dimitrie Terzici, profesor de limba franceză, Niţa Nedea, profesorde biologie. Din 2007 şi până în prezent directorul şcolii este Gabriela Săndulescu,profesor de matematică.De departe cea mai puternică şi influentă personalitate din acest şir de directoria fost Tereza Strătilescu. Ea a fost cea care a reorganizat şcoala, căutândpermanent noi modalităţi de a oferi cât mai multe şanse elevelor şcolii. Spirit progresist,Tereza Strătilescu a anticipat multe dintre ideile educaţiei moderne, idei care pardesprinse din discursurile pedagogilor de astăzi. De pildă: ,,Niciodată şcoala să nuspună – asta nu e treaba mea, ci a părinţilor. De aceea, şcoala şi familia trebuie săse cunoască, să se înţeleagă, să se sfătuiască, să se ajute reciproc.” Sau: ,,(elevului)pe lângă învăţătură îi trebuie şi hrană corespunzătoare, aer, repaus, distracţie, gimnastică,sport, chiar dans.” Ori, şi mai bine: ,,Mai presus de toate, să nu uitămcă scopul final al educaţiei este să puie pe educat în stare de a-şi continua el însuşieducaţia după ce va isprăvi şcoala, de a-l pregăti pentru acea educaţie de sine careface în fine dintr-un om aceea ce poate el fi, omul prin excelenţă.” De menţionat căTereza Strătilescu era de fapt continuatoarea unei tradiţii de excelenţă, profesionalismşi dăruire în domeniul educaţiei.Cu asemenea valori puse la temelia educaţiei, nu este de mirare că această şcoalăa dat foarte multe nume care pot figura pe o listă de ,,prima femeie care...” Dintreprofesori, putem menţiona nume cum ar fi Ana Conta Kernbach, prima femeiedin ţară aleasă în Consiliul general al instrucţiunii, Vera Myller Lebedev, matematician,prima femeie profesor universitar într-o disciplină ştiinţifică, autoare a unorlucrări originale în teoria numerelor, algebră şi analiză matematică , Elena BuzneaMeissner, pedagog, militantă pentru emanciparea femeii, fondator al primei Societăţipentru ocrotirea copilului din ţară, pune bazele pedagogiei speciale în România, deschizândo şcoală specială pentru copiii cu dezabilităţi, şcoală care a funcţionat laBucium în vila Greierul până în 1958; Silvia Creangă, prima femeie din Româniacare a obţinut titlul de doctor în ştiinţe matematice. Ultima, profesoară la Liceul,,Oltea Doamna” în perioada 1920-1944, a publicat lucrări de geometrie diferenţialăîn reviste din ţară şi străinătate (C.R. Acad. Sc. Paris), a publicat două ediţii (1935,1943) ale volumului Probleme de matematici elementare. A ştiut, înainte de toate,să inspire dragoste pentru matematică, multe eleve talentate devenind cadre didacticeuniversitare, matematicieni cunoscuţi în lumea academică (Florica T. Câmpan,Ştefania Ruscior, Olga Costinescu).43


Dacă vorbim despre absolventele şcolii, trebuie să pomenim pe Elena PuşcariuDensuşianu, medic oftalmolog, prima femeie profesor universitar la Universitateade Medicină şi Farmacie Iaşi, cu studii de specialitate la Institutul Pasteur din Paris,doctor în medicină în 1899, fondatoarea Clinicii de oftalmologie din Iaşi; pe EllaNegruzzi, avocat, militantă pentru emanciparea femeii din România, prima femeiecare a profesat avocatura în ţara noastră şi printre primele din lume; pe RalucaRipan, absolventă din 1914, prima femeie academician din România; pe MariaRopală Cicherschi, prima femeie medic legist din Europa.Printre absolvenţii de marcă ai şcolii amintim: scriitori – Camelia Nădejde(cunoscută sub pseudonimul Lucia Mantu), Alexandra Gavrilescu (Otilia Cazimir),Profira Sadoveanu, Nichita Danilov; medici – Maria Briese, fondatoareaclinicii de endocrinologie de la Iaşi, Maria Franche, Lidia Jacotă Briese,ChiAsia Braifpail, Mărioara Tănăsescu Niculescu etc.; artişti lirici – MagdaIanculescu, Viorica Cortez; artişti plastici – Mărioara Dumitrescu Dumbravă,Cela Neamţu, autoarea lucrării monumentale de tapiţerie Ferestre pentruIerihon care decorează Catedrala Ortodoxă din Ierihon; actori şi regizori – RodicaMandache, Liviu Smîntînică, Ovidiu Lazăr, Ozana Ciubotaru, Doru Aftanasiu,Monica Bârlădeanu, Cristian Mungiu; sportivi de performanţă – DanIrimiciuc, vicecampion mondial de scrima la tineret în 1969, prima medalie olimpicăa Iaşului la scrimă, cercetător ştiinţific, Maricica Puică, medalie de aur pentru cursade 3000 m şi bronz la 1500 m la Olimpiada de la Los Angeles în 1984, deţinătoarea unuirecord mondial; psihologi, politologi – Vasile (Bebe) Mihăescu, Alina MungiuPippidi şi mulţi alţii.În domeniul matematicii, una dintre personalităţile foarte cunoscute este FloricaT. Câmpan, absolventă a Liceului ,,Oltea Doamna” în 1924, fostă elevă a SilvieiCreangă; îşi trece licenţa în fizică în 1928 şi în matematică în 1929, iar în 1942 îşisusţine doctoratul în matematică în faţa unei comisii prezidate de renumitul profesorAlexandru Myller cu o teză de geometrie diferenţială intitulată Surfaces parallèles etsemblables. Conferenţiar de matematici generale la Institutul de măsurători terestredin Iaşi, apoi la Facultatea de Matematică şi Fizică a Universităţii ,,Al. I. Cuza” Iaşi,a publicat numeroase cărţi de popularizare a matematicii şi de istoria matematicii– Istoria numărului π, Probleme celebre din istoria matematicii, Aventura geometriilorneeuclidiene, Licuricii din adâncuri, Dumnezeu şi matematica etc. Ca apreciereşi recunoştinţă, unul dintre cele mai cunoscute concursuri şcolare de matematică îipoartă numele.Înfruntând şi depăşind vicisitudinile vremurilor prin care a trecut în existenţa saîndelungată, prin aportul pe care l-a avut şi pe care-l are în modelarea a numeroasegeneraţii de elevi în spiritul valorilor morale autentice şi în îmbogăţirea minţii lor cutemeinice cunoştinţe umaniste şi ştiinţifice, Colegiul Naţional ,,M. Eminescu” seînscrie pe linia tradiţiei progresiste a învăţământului şi culturii româneşti.Prof. dr. Gabriela SăndulescuDirector al Colegiului Naţional ,,Mihai Eminescu”44


Concursul ,,Recreaţii Matematice”Ediţia a IX-a, Muncel, 24 august 2011Clasele III-IV1. a) Suma a cinci numere naturale nenule este 14. Calculaţi produsul tuturordiferenţelor care se pot forma cu cele cinci numere.b) Determinaţi a + b + c, ştiind că a − 30 = b + c şi 200 : (b + c) = 5.2. Un număr de două cifre are cifra unităţilor de 3 ori mai mare decât cifrazecilor. Aflaţi numărul, ştiind că răsturnatul său este cu 36 mai mare ca număruliniţial.Recreaţii Matematice3. Din cauza unei erori a tipografului, la numerotarea paginilor unei cărţi în loculcifrei 8 s-a scris peste tot cifra 3. Astfel, apare de 66 de ori cifra 3. Câte pagini arecartea?Subiecte selectate de inst. Doina NechiforClasa a V-a1. Determinaţi numerele de forma 5abc care, împărţite la abc5, dau câtul de 595de ori mai mic decât restul.Recreaţii Matematice2. Determinaţi numerele abc, scrise în baza 10, divizibile cu 22, dacă c = 2 · b.Artur Bălăucă3. Problema lui Poisson. Cineva avea într-o damingeană 12 litri de vin şi doreasă dăruiască din el jumătate. Nu avea decât două vase: unul de 8 litri şi altul de 5litri. Cum se poate proceda pentru a separa 6 litri de vin în vasul de 8 litri?Notă. Siméon Denis Poisson (1781-1840): Această problemă mi-a determinat soarta,am hotărât să mă fac neapărat matematician.Clasa a VI-a1. Dacă fracţia 3n + 7 , n ∈ N, este reductibilă, determinaţi ultima cifră a lui n.2n + 3Recreaţii Matematice2. Rezolvaţi în N ecuaţia x 2y + 144 = 25x y .3. În triunghiul ABC se ştie că m(∢ABC) = 15o iar unghiul <strong>format</strong> de bisectoareaşi înălţimea corespunzătoare unghiului ∢BAC are măsura de 15 o . Aflaţi măsurileunghiurilor triunghiului ABC.Artur BălăucăClasa a VII-a1. Să se rezolve în Z ecuaţia 1 x + xx + y + 1 = 0.Artur Bălăucă2. Se consideră triunghiul isoscel ABC cu AB=AC şi m(∢BAC)


Dacă ∢ADE ≡ ∢ABC, arătaţi că punctul D este mijlocul segmentului (BC).Recreaţii Matematice3. Fie triunghiul ABC şi D, E, F picioarele bisectoarelor duse din vârfurile A, B,respectiv C. Dacă punctul I este centrul cercului înscris triunghiului ABC, arătaţică IAAD · IBBE · ICCF ≤ 8 27 .Clasa a VIII-a1. Arătaţi că:a) nu putem aşeza pe un cerc numerele 1, 2, 3, ..., 2011 astfel încât modulul diferenţeidintre oricare două numere alăturate să fie acelaşi;b) putem aşeza pe un cerc numerele 1, 2, 3, ..., 2011 astfel încât modulul diferenţeidintre oricare două numere alăturate să fie 2 sau 3.Gheorghe Iurea2. a) Să se arate că din orice şapte numere aparţinând intervalului (1; 13) se potalege trei care să reprezinte lungimile laturilor unui triunghi.b) Să se arate că există şapte numere aparţinând intervalului [1; 13] astfel încâtoricare trei nu pot reprezenta lungimile laturilor unui triunghi.Recreaţii Matematice3. Fie ABCD un tetraedru şi A ′ , B ′ , C ′ , D ′ centrele de greutate ale feţelor (BCD),(ACD), (ABD), respectiv (ABC). Arătaţi că:a) dreptele AA ′ , BB ′ , CC ′ , DD ′ sunt concurente (notăm cu G punctul de concurenţăal acestora);b) AG = 3 4 AA′ .Clasa a IX-a1. Suma a patru numere naturale nenule este 2011. Arătaţi că cel mai micmultiplu comun al numerelor este cel puţin 670.Gheorghe Iurea2. Fie ABCD un dreptunghi şi M un punct din planul său. Arătaţi că MA ·<strong>MB</strong> + MC · MD ≥ AB · BC.Gheorghe Iurea3. Demonstraţi că tg x > 4 sin x − 2, oricare ar fi x ∈h0; π 2i.Recreaţii MatematiceClasa a X-a1. În câte moduri putem împărţi 6 banane la 4 babuini, ştiind că fiecare babuinprimeşte cel puţin o banană?Al. G. Mîrşanu2. Considerăm şirul (x n ) n≥0 , definit astfel: x 0 = x 1 = 0 şi x n+2 − 2x n+1 + x n =15 3 · n · 16 n−1 , ∀n ≥ 0.a) Arătaţi că x n = 15n + 2 + (15n − 32) · 16 n−1 , ∀n ≥ 0.b) Găsiţi restul împărţirii lui x 2011 la 13.Lucian Lăduncă3. Fie z = x+iy ∈ C, cu x, y ∈ Q şi |z| = 1. Demonstraţi că |z 2n −1| ∈ Q, ∀n ∈ N ∗ .Gheorghe Iurea46


Concursul interjudeţean ,,Speranţe Olimpice”Ediţia a XI-a, Paşcani, 12 noiembrie 2011Clasa a III-a1. a) Găseşte regula şi completează cu numerele corespunzătoare:3517 1245 6326 3217 142588 39 98b) Află numărul necunoscut: 200 − a + (100 − 68) = 126 − 78.2. a) Câte numere de 8 se găsesc în intervalul de la 5 la 90?b) Reconstituiţi adunarea: 2a7b + 51c6 = 8d94.3. a) La o librărie s-a adus 749 caiete: dictando, de matematică şi vocabulare.Caietele dictando şi cele de matematică împreună sunt cu 229 mai puţine decât celede matematică împreună cu vocabularele. Câte caiete s-au adus de fiecare fel, dacăcele dictando împreună cu cele de matematică sunt 396?b) În dreptunghiurile de mai jos sunt scrise numere astfel încât suma numerelordin fiecare patru dreptunghiuri alăturate să fie aceeaşi.Care este numărul din ultima căsuţă?25 10Clasa a IV-a1. a) Aflaţi a din egalitatea: [(a : 7 + 107) : 4 × 10 − 192] : 4 = 87.b) Dacă a × b = 8, iar c × a = 10, calculaţi a × (b + c) : 7 şi a × (b − c) : 8.2. Suma a trei numere este 88. Dacă din primul număr luăm 15, din al doilea 19,iar din al treilea 21, obţinem trei numere consecutive. Aflaţi numerele.3. a) Determinaţi toate numerele de forma abc, ştiind că a + c = b × b.b) Demonstraţi că dintre şapte numere naturale, există cel puţin două care dauacelaşi rest la împărţirea prin 6.Clasa a V-a1. a) Să se scrie numărul 189 2011 ca sumă de trei pătrate perfecte.b) Într-un grup de 28 copii, fiecare copil oferă câte o floare fiecărei fete. Ştiind căîn total s-au oferit 270 de flori, să se determine numărul de băieţi şi de fete.2. a) Determinaţi cel mai mic număr natural care începe cu 2011, se termină cu2011 şi are suma cifrelor 2011.b) Numărul A este scris cu 66 cifre de 3, iar numărul B este scris cu 33 de cifrede 6. Determinaţi produsul A × B.3. Dacă a, b, c sunt numere naturale nenule astfel încât a 2 = b 2 + c 2 , arătaţi căoricare ar fi n ∈ N ∗ , există p şi q numere naturale nenule astfel încât a 2n = p 2 + q 2 .Clasa a VI-a1. Pe o tablă sunt scrise numerele 1, 2, 3, . . . , 9999. La un pas se pot şterge câtevadintre numerele scrise şi în locul lor se scrie restul de la împărţirea sumei numerelorşterse prin 11. După câţiva paşi pe tablă au rămas scrise două numere, unul dintreele fiind 1023. Care este cel de-al doilea număr?47


2. Fie E(n) = 52n+1 + 2 2n+3 × 3 n2 n+2 + 3 n+2 × 5 n , n ∈ N∗ .a) Calculaţi E(1).b) Arătaţi că E(n) > 1, pentru n ≥ 2.c) Arătaţi că E(n) se simplifică cu 13.3. Determinaţi numerele naturale n, n ≤ 23, astfel încât 5 divide a, unde a =2 2n+1 + 3 3n+2 + 4 4n+3 .Clasa a VII-a1. Se dă şirul de numere 37, 337, 3337, . . ..a) Daţi exemplu de doi termeni ai şirului al căror raport este număr natural.b) Arătaţi că şirul dat conţine o infinitate de numere care nu sunt prime.2. a) Dacă 0 ≤ a ≤ 5, 0 ≤ b ≤ 7 şi 2a + 3b = 24, arătaţi că ab ≤ 27.b) Să se arate că numărulN = 800119 1 + 2 + 3 + . . . + 100 + 11 + 2 + 3 + . . . + 101 + 11 + 2 + 3 + . . . + 1999‹este cub perfect.3. Punctul P este situat în interiorul △ABC, iar M şi N sunt simetricele punctuluiP în raport cu mijloacele laturilor [AB], respectiv [AC]. Arătaţi că P se află peînălţimea din A a △ABC dacă şi numai dacă [BN] ≡ [CM].Clasa a VIII-a1. Arătaţi că există o infinitate de mulţimi finite nevide A şi B, cu elementenumere naturale nenule, care verifică simultan relaţiile:a) Card A = Card B + 1;b) x < y, oricare ar fi x ∈ A şi oricare ar fi y ∈ B;c) elementele lui A ∪ B sunt numere consecutive;d) suma elementelor lui A este egală cu suma elementelor lui B.2. Fie a, b, c ∈ (0; +∞) cu a + b + c = 1. Demonstraţi că:rabab + c +rbcbc + a +Écaca + b ≤ 3 2 .3. Considerăm patrulaterul convex ABCD cu proprietatea că A, D sunt situateîntr-un plan α, iar B şi C nu aparţin planului α. Fie punctele M ∈ (AB), N ∈ (CD)astfel încât A<strong>MB</strong>M = DN1= k. Arătaţi că dacă MN =CN k + 1 AD + k BC, atuncik + 1BC∥α.Vizitaţi pagina web a revistei Recreaţii Ştiinţifice (1883-1888) :http://www.recreatiistiintifice.ro48


Soluţiile problemelor propuse în nr. 2/2011Clasele primareP.206. Fie numerele a = 1 + ⃝ şi b = 9 − □. Înlocuiţi cercul şi pătratul cu cifrecorespunzătoare astfel încât a + b = 15.(Clasa I)Amalia Munteanu, elevă, IaşiSoluţie. 15 = 10 + ⃝ − □, de unde ⃝ − □ = 5, cu posibilităţile: 9 − 4 = 5;8 − 3 = 5; 7 − 2 = 5; 6 − 1 = 5 şi 5 − 0 = 5.P.206. O elevă a desenat un trenuleţ cu 23 vagoane pe care le-a colorat folosind,pe rând, culorile roşu, galben, albastru, roşu, galben ş.a.m.d. Ce culoare a folositpentru vagonul din mijloc?(Clasa I)Mihaela Gîlcă, elevă, IaşiSoluţie. Deoarece 23 = 11 + 1 + 11, înseamnă că al 12-lea vagon este în mijloc.Cum 12 = 3 + 3 + 3 + 3, deducem că pentru vagonul din mijloc a folosit culoareaalbastră.P.218. Mihaela are 14 ani. Ea s-a născut când sora sa avea 7 ani, dar cu 5 aniînaintea fratelui său. Câţi ani au împreună cei trei fraţi?(Clasa a II-a)Maria Racu, IaşiSoluţie. Sora Mihaelei are 14+7 = 21 (ani), iar fratele 14−5 = 9 (ani). Împreunăau 14 + 21 + 9 = 44 (ani).P.219. Un cioban păzea câteva oi şi câteva capre, în total 24 picioare. Câte oi şicâte capre sunt, dacă oile sunt mai multe decât caprele?(Clasa a II-a)Andreea Bîzdîgă, elevă, IaşiSoluţie. Din descompunerea 24 = 4 + 4 + 4 + 4 + 4 + 4, rezultă că sunt în total6 oi şi capre. Pot fi cinci oi şi o capră sau patru oi şi două capre.P.220. Suma a două numere este 2010. Dacă ştergem cifra miilor unuia dintreele, obţinem celălalt număr. Aflaţi cele două numere.(Clasa a III-a)Mihaela Cianga, IaşiSoluţie. Putem avea 1000 + 2x = 2010 sau 2000 + 2x = 2010, unde x este aldoilea număr. Avem două soluţii: (1505; 505) şi (2005, 5).P.221. Suma dintre un număr şi succesorul său este cu 10 mai mare decât predecesorulsău. Calculaţi produsul dintre număr şi vecinii săi.(Clasa a III-a)Petru Miron, PaşcaniSoluţie. Se deduce că succesorul numărului căutat este 9, deci numărul este 8.Obţinem 7 × 8 × 9 = 504.P.222. Pe o farfurie sunt cireşe şi vişine. Un copil mănâncă o treime din cireşeşi o jumătate din vişine şi constată că are 17 sâmburi. Pot rămâne pe farfurie 20fructe? dar 34?(Clasa a III-a)Tatiana Ignat, elevă, Iaşi49


Soluţie. Din c : 3 + v : 2 = 17, obţinem că 2c + 3v = 102. Dacă pe farfurie rămân20 de fructe, atunci c + v = 37 şi deducem că c = 9, v = 28, deci răspunsul la primaîntrebare este afirmativ. Dacă pe farfurie ar rămâne 34 de fructe, atunci c + v = 51şi am obţine v = 0, imposibil; răspunsul la a doua întrebare este negativ.P.223. Se consideră numerele naturale x, 4x, 2x + 3, x + 2 şi 3x + 2, unde x > 2.a) Ordonaţi crescător numerele.b) Dacă notăm cu m cel mai mic număr şi cu M pe cel mai mare, care trebuie săfie valoarea lui x pentru ca şirul m, m + 1, . . . , M să conţină 130 numere?(Clasa a IV-a)Mariana Nastasia, elevă, IaşiSoluţie. a) x < x + 2 < 2x + 3 < 3x + 2 < 4x; b) 4x − x + 1 = 130, de undex = 43.P.224. Un elev îşi ţine banii în două buzunare. Dacă ar cheltui un sfert din sumadin primul buzunar şi o doime din cea din al doilea, suma totală s-ar micşora cu 48lei. Care ar fi suma totală dacă, fără a cheltui nimic, elevul ar dubla suma din aldoilea buzunar?(Clasa a IV-a)Petru Asaftei, IaşiSoluţie. Dacă în primul buzunar avem a lei şi în al doilea b lei, atunci a : 4 + b :2 = 48. Mărind de 4 ori fiecare membru obţinem a + 2b = 192. Suma totală ar fi 192lei.P.225. Aflaţi numerele de trei cifre distincte abc, dacă abc + bca + cab = 666.(Clasa a IV-a)Nicolae Ivăşchescu, CraiovaSoluţie. Egalitatea abc + bca + cab = 666 se reduce la a + b + c = 6. Cum a, bşi c sunt cifre nenule distincte, convine doar situaţia 6 = 1 + 2 + 3. Numerele sunt:123, 132, 231, 213, 312 şi 321.Clasa a V-aV.137. Se consideră numerele naturale A = 1 2010 + 2 2010 + . . . + 9 2010 şi B =1 2011 + 2 2011 + . . . + 9 2011 . Demonstraţi că B − A se divide cu 10.Mariana Mărculescu şi Dumitru Cotoi, CraiovaSoluţie. Folosind faptul că 2010 = M 4 +2, iar 2011 = M 4 +3, deducem că U(A) =U(1+4+9+6+5+6+9+4+1) = 5 şi U(B) = U(1+8+7+4+5+6+3+2+9) = 5.Atunci U(B − A) = 0, prin urmare B − A . .10.V.138. Găsiţi un multiplu al lui 13 a cărui scriere în baza 10 conţine doar cifrede 1.Nicolae Ivăşchescu, CraiovaSoluţia 1. 111111 = 1001 · 111 = (7 · 11 · 13) · (3 · 37) = M 13 .Soluţia 2 (Cătălin Gulin, elev, Craiova). Cum 1 76923= 0, (076923) =13 999999 =8547111111 , rezultă că 111111 = 13 · 8547 = M 13.V.139. Scrieţi numărul 17689 ca diferenţă de două pătrate perfecte nenule.Liviu Smarandache, CraiovaSoluţia 1. 17689 = 7 2 · 19 2 = (25 2 − 24 2 ) · 19 2 = 475 2 − 456 2 .50


Soluţia 2 (Cătălin Gulin, elev, Craiova). Orice număr impar n = 2k + 1 sepoate scrie sub forma (k + 1) 2 − k 2 . În cazul nostru, 17689 = 88452 − 8844 2 .V.140. Determinaţi numerele de forma 5abc care, împăţite la abc5, dau câtul de595 ori mai mic decât restul.Petru Asaftei, IaşiSoluţie. Dacă q este câtul împărţirii, atunci 5abc = abc5·q +595q şi 595q < abc5.Obţinem că 5000+abc = 10q·abc+600q, de unde c = 0. Astfel, 500+ab = 10q·ab+60qşi de aici rezultă că b = 0. Deducem că 50+a = 10q ·a+6q, egalitate care se realizeazădoar când a = q = 2. În concluzie, singura soluţie a problemei este 5200.V.141. Se consideră numerele naturale a 1 , a 2 , . . . , a n astfel încât a 1 = 1 şi fiecarenumăr, începând cu al doilea, este triplul sumei tuturor numerelor dinaintea lui. Dacăa 1 + a 2 + . . . + a n = 2 20 , determinaţi n.Mirela Marin, IaşiSoluţie. Observăm că a 2 = 3, a 3 = 3 · (1 + 3) = 3 · 4, a 4 = 3(1 + 3 + 3 · 4) = 3 · 4 2 ,a 5 = 3(1 + 3 + 3 · 4 + 3 · 4 2 ) = 3 · 4 3 . În general, a n = 3 · 4 n−2 , n ≥ 2, şi atuncia 1 + a 2 + . . . + a n = 1 + 3 + 3 · 4 + 3 · 4 2 + . . . 3 · 4 n−2 = 4 n−1 . Din 4 n−1 = 2 20 obţinemcă n = 11.| {z }2011| {z }2011| {z }2011| {z }2011| {z }2011| {z }2011V.142. Arătaţi că numărul A = 11 . . . 11 22 . . . 22 + 33 . . . 33 44 . . . 44 − 11 . . . 11 0este pătrat perfect.Andrei Nedelcu, IaşiSoluţie. Dacă notăm x = 11 . . . 11 , atunci A = x · 10 2011 + 2x + 3x · 10 2011 +| {z }20114x − 10x = x(10 2011 + 2 + 3 · 10 2011 + 4 − 10) = x(4 · 10 2011 − 4) = 4x(10 2011 − 1) =4x · 99 . . . 99 = 4x · 9x = (6x) 2 , deci A este pătrat perfect.V.143. Reconstituiţi înmulţireaalăturată, ştiind că literele distinctereprezintă cifre distincte.a b c d e f ×* * * * * *f a b c d ee f a b c dd e f a b cc d e f a bb c d e f aa b c d e f* * * * * * * * * * *Cătălin Calistru, IaşiSoluţie. Fie xyzuvw al doilea factor al produsului. Avem că abcdef ·w = fabcde;cu notaţia N = abcde, obţinem că (10N + f) · w = 100000f + N. Evident că f ≠ 0şi e ≠ 0 (apar ca primă cifră) şi atunci w ≠ 0. Luând, pe rând, w ∈ {1, 2, . . . , 9},găsim unica variantă convenabilă w = 5, f = 7, N = 14285, aşadar abcdef = 142857.Înlocuim şi deducem că produsul este egal cu 18949266765, iar factorul al doilea seobţine prin împărţire, fiind 132645.51


Clasa a VI-aVI.137. Dacă fracţia 3n + 7 , n ∈ N, este reductibilă, determinaţi ultima cifră a2n + 3lui n.Eugeniu Blăjuţ, BacăuSoluţie. Dacă d = (3n + 7, 2n + 3), d ≥ 2, atunci d|2(3n + 7) − 3(2n + 3), adicăd|5. Obţinem că d = 5 şi de aici se deduce uşor că n = 5k + 1, k ∈ N, prin urmareultima cifră a numărului n poate fi 1 sau 6.VI.138. Determinaţi numerele prime abc cu proprietatea că dintre cele cincinumere (nu obligatoriu distincte) care se obţin prin permutarea cifrelor, există măcardouă pătrate perfecte.Nicolae Ivăşchescu, CraiovaSoluţie. Scriem toate pătratele perfecte de trei cifre şi urmărim care dintre ele,prin permutarea cifrelor, dau naştere unui alt pătrat perfect şi unui număr prim.Problema are două soluţii: 691 este prim şi 169, 196, 961 sunt pătrate, apoi 211 esteprim şi 121, 121 sunt pătrate.VI.139. Demonstraţi că 287|(8 15 + 8 5 + 5)(42 117 + 42 39 + 39).Bogdan Victor Grigoriu, FălticeniSoluţie. Observăm că 287 = 7 · 41. Cum 8 15 + 8 5 + 5 = (7 + 1) 15 + (7 + 1) 5 + 5 =(M 7 + 1) + (M 7 + 1) + 5 = M 7 şi 42 117 + 42 39 + 39 = (41 + 1) 117 + (41 + 1) 39 + 39 =(M 41 + 1) + (M 41 + 1) + 39 = M 41 , urmează concluzia problemei.VI.140. Determinaţi numerele întregi nenule n 1 < n 2 < . . . < n 7 , dacă 2 n1 +2 n 2+ . . . + 2 n 7= 9851024 . Mihai Haivas, IaşiSoluţie. Trecând numărul 985 în baza 2, avem că 985 (10) = 1111011001 (2) . Din2 9 + 2 8 + 2 7 + 2 6 + 2 4 + 2 3 + 1 = 2 n7+10 + 2 n6+10 + . . . + 2 n1+10 deducem că n 1 = −10,n 2 = −7, n 3 = −6, n 4 = −4, n 5 = −3, n 6 = −2, n 7 = −1. Cum scrierea unui numărîn baza 2 este unică, problema are o singură soluţie.VI.141. Demonstraţi că ecuaţia 12x + 15y + 20z = 73 nu are soluţii (x, y, z) cutoate componentele numere naturale, dar are o infinitate de soluţii cu toate componentelenumere întregi.Gheorghe Iurea, IaşiSoluţie. Presupunem că x, y, z ∈ Z sunt astfel încât 12x + 15y + 20z = 73; atunciz + 1 = 3(4x + 5y + 7z − 24) = 3k, k ∈ Z, deci z = 3k − 1, k ∈ Z. Înlocuind, obţinemcă 12x + 15y + 60k = 93, de unde x + 1 = 5(x + y + 4k − 6) = 5n, n ∈ Z, adicăx = 5n − 1, n ∈ Z. De aici, 4n + y + 4k = 7, prin urmare y = 7 − 4n − 4k. Rezultăcă (x, y, z) = (5n − 1, 7 − 4n − 4k, 3k − 1), unde n, k ∈ Z, constituie soluţiile ecuaţieidiofantice din enunţ, având toate componentele întregi.Pentru a demonstra prima afirmaţie a problemei, să presupunem prin absurd căar exista n, k ∈ Z pentru care 5n − 1, 7 − 4n − 4k şi 3k − 1 ar fi simultan nenegative.Din 5n − 1 ≥ 0, n ∈ Z, rezultă că n ≥ 1; cum 3k − 1 ≥ 0, k ∈ Z, obţinem k ≥ 1.Atunci 7 − 4n − 4k ≤ −1, contradicţie. Astfel, soluţia problemei este completă.VI.142. Fie ABC un triunghi isoscel cu AB = AC. Dacă D este simetricul lui B52


faţă de C şi mediana din B taie AD în E, arătaţi că CE este mediatoarea segmentuluiBD.Silviu Boga, IaşiSoluţie. Notăm cu F şi G mijloacele segmentelor AB, respectiv AC. Din△BCF ≡ △CBG (L.U.L.) obţinem căÕF CB ≡ÕGBC. AÎnsă CF este linie mijlocie în △BAD şi atunci CF ∥AD,prin urmareÕF CB ≡ÕADB. Rezultă căÕEBD ≡ÕEDB,Edeci △EBD este isoscel cu baza BD; mediana CE va fiF Gmediatoarea segmentului BD.VI.143. Se consideră triunghiul ABC cu m(ÕACB) =45 ◦ şi (ÕCBA) = 30 ◦ , iar M este un punct pe segmentul B C DAB. Arătaţi că M este mijlocul lui AB dacă şi numai dacă m(ÖMCB) = 15 ◦ .Andrei Paşa, elev, şi Narcisa Paşa, IaşiSoluţie. Fie D piciorul înălţimii din A; triunghiul ACD va fi dreptunghic isoscel,cu AD = CD, iar triunghiul ABD este dreptunghic în D, cu m(ÒB) = 30 ◦ , prinurmare AD = 1 2 AB.Presupunem că M este mijlocul lui AB.Atunci,DM = 1 AB = AM = AD, deci triunghiul ADM este2echilateral. Deducem că DM = DC şi m(ÖCDM) = 150 ◦ .Obţinem că m(ÖDCM) = 1 2 (180◦ − 150 ◦ ) = 15 ◦ .A M BReciproc, fie m(ÖBCM) = 15 ◦ şi să presupunem că M nu ar fi mijlocul lui AB.Notăm cu M ′ mijlocul lui AB şi, folosind directa, deducem că m(ÖBCM ′ ) = 15 ◦ .Astfel, M ′ = M şi ajungem la o contradicţie.Notă. D-l Titu Zvonaru găseşte trei soluţii ale acestei probleme, iar autoriioferă o a patra soluţie, distinctă.Clasa a VII-aVII.137. Dacă x, y ∈ R, arătaţi că (x 2 + x + 1)(y 2 + y + 1) ≥ x + y + 1.Gheorghiţă Stănică şi Iulian Stănică, Apele Vii (Dolj)Soluţie. Efectuând calculele, inegalitatea din enunţ revine la x 2 y 2 + x 2 + y 2 +x 2 y + xy 2 + xy ≥ 0, care este binecunoscuta a 2 + b 2 + c 2 + ab + bc + ca ≥ 0 pentrua = xy, b = x, c = y. Egalitatea se atinge când x = y = 0.VII.138. Dacă a, b, c sunt numere întregi distincte, arătaţi că a 2 + b 2 + c 2 − ab +ac + bc ≥ 1. Când se realizează egalitatea?Elena Iurea, IaşiSoluţie. Cum a 2 + b 2 + c 2 − ab + ac + bc = 1 2 [(a − b)2 + (a + c) 2 + (b + c) 2 ] > 0(întrucât a ≠ b), rezultă că numărul întreg a 2 + b 2 + c 2 − ab + ac + bc este cel puţinegal cu 1. Avem egalitate dacă (a − b) 2 = 1, (a + c) 2 = 1 şi (b + c) 2 = 0 sau dacă(a − b) 2 = 1, (a + c) 2 = 0 şi (b + c) 2 = 1, deci pentru tripletele (a, b, c) de forma(−n, 1 − n, n); (−n, −1 − n, n); (1 − n, −n, n) sau (−1 − n, −n, n) cu n ∈ Z ∗ .53CD


aaVII.139. Determinaţi cifrele a, b şi c, dacăÊ=Ê 33·É b, b(bc) ∈ N.Romanţa Ghiţă şi Ioan Ghiţă, BlajSoluţie. Observăm că a nu poate fi 0, iar b şi c nu pot fi simultan 0 sau simultan 9.aaCum x =É990 · 11ab, b(bc) 1099b + c = 10a10a∈ N, se impune ca1099b + c 1099b + c =p 2q 2 , cu p, q ∈ N∗ , (p, q) = 1 şi q|33. Atunci q 2 ∈ {1, 9, 121, 1089}. Cum b şi c suntcifre şi 10a nu se divide cu 11, cazurile q 2 = 121 şi q 2 = 1089 se elimină. În celelaltedouă situaţii, b nu poate fi decât 0. Dând lui c toate valorile 1, 2, . . . , 9, găsim soluţiilea = 2, b = 0, c = 5; a = 5, b = 0, c = 2; a = 8, b = 0, c = 5.VII.140. Determinaţi toate perechile (x, y) de numere întregi cu proprietatea că2 x+y (2 x2 +y 2 + 1) = 1.Neculai Stanciu, BuzăuSoluţie. Folosind inegalitatea mediilor, obţinem că 1 = 2 x2 +y 2 +x+y + 2 x+y ≥2 · √2 x2 +y 2 +2x+2y= √ 2 (x+1)2 +(y+1) 2 , prin urmare 2 (x+1)2 +(y+1) 2 ≤ 1. Deducem că(x + 1) 2 + (y + 1) 2 = 0, adică (x, y) = (−1, −1).VII.141. Dacă ABCD este un patrulater convex, arătaţi că există un unic punctM ∈ (BD) astfel încât triunghiurile ABM şi CDM să fie echivalente.Cecilia Deaconescu, PiteştiSoluţie. Fie {O} = AC ∩ BD şi A ′ , C ′ proiecţiile pe BD ale vârfurilor A,respectiv C. Condiţia A ABM = A CDM revine la BM · AA ′ = MD · CC ′ , adicăBMAMD = CC′ CC′. ÎnsăAA′AA ′ = CO (din asemănareaAO△AOA ′ ∼ △COC ′ ), prin urmare punctul căutatM este unicul punct interior segmentului (BD) careBCîl împarte în raportul k = CODA OAO .VII.142. Determinaţi valoarea minimă a arieiunui paralelogram circumscris unui cerc de rază r.Adrian Corduneanu, IaşiCSoluţie. Fie ABCD paralelogram circumscris cercului C(O, r); atunci AB+CD =AD+BC, prin urmare ABCD este, de fapt, romb. Fie α = m(ÕABD) şi M = P r AB O;atunci AB = AM +<strong>MB</strong> = OMctg α + OMtg α = r cos αsin α + sin α cu egalitate cândcos α‹≥2r,cos α = sin α, adică α = 45 ◦ AB · OM. Deducem că A ABCD = 4 · A OAB = 4 ≥24r 2 , cu egalitate pentru α = 45 ◦ . În concluzie, valoarea minimă căutată a arieiparalelogramului este 4r 2 , atinsă în cazul în care ABCD este pătrat.VII.143. În interiorul triunghiului ascuţitunghic ABC cu m(bA) = 60 ◦ se considerăun punct M astfel încât m(ÖBMC) = 150 ◦ . Un cerc ce trece prin A şi M taie(AB) în Q şi (AC) în R, iar cercul circumscris triunghiului MQB taie (BC) în P .Demonstraţi că triunghiul P QR este dreptunghic.Neculai Roman, Mirceşti (Iaşi)54


Soluţie. Folosind faptul că patrulaterele MP BQ şi MQAR sunt inscriptibile,obţinem imediat că şi patrulaterul MP CR este inscriptibil. Atuncim(ÕQP R) = m(ÖMP Q) + m(ÖMP R) = m(Ö<strong>MB</strong>Q) + Am(ÖMCR) = m(ÒB) − m(Ö<strong>MB</strong>C) + m(ÒC) − m(ÖMCP ) =[180 ◦ − m(bA)] − [180 ◦ − m(ÖBMC)] = 90 ◦ .Notă. Rezultatul este o generalizare a problemeiQM RVII.40 din RecMat 1/2003.Clasa a VIII-aVIII.137. Fie V ABCD o piramidă patruletară regulată,M şi N mijloacele muchiilor V A, respectiv V D,iar P punctul în care înălţimea V O a piramidei înţeapăplanul (<strong>MB</strong>C). Arătaţi că V O = 3 · OP.Adrian Corduneanu, IaşiSoluţie. Notăm cu Q şi R mijloacele muchiilor AD,respectiv BC şi {S} = V Q ∩ MN. Cum MN este liniemijlocie în △V AD, urmează că V S = SQ. Deducem căP este punctul de intersecţie a medianelor triunghiuluiV QR, deci V O = 3 · OP.B P CVIII.138. Rezolvaţi în R ecuaţia 25 · {x} 2 − 10x + 1 = 0.Bogdan Chiriac, student, IaşiSoluţie. Înlocuind x = [x] + {x}, ecuaţia devine (5 · {x} − 1)2 = 10 · [x] şi, cum5 · {x} − 1 ∈ (−1, 4), rezultă că 10 · [x] ∈ [0, 16), deci [x] ∈ {0, 1}. Dacă [x] = 0, atunci{x} = 1 5 şi obţinem soluţia x 1 = 1 5 . Dacă [x] = 1, atunci 5{x} − 1 = ±√ 10, adică{x} = 1 ± √ 10.5 Însă {x} ∈ [0, 1), aşadar reţinem doar soluţia x 2 = 1 + 1 + √ 10=56 + √ 10.5VIII.139. Numerele naturale a 1 , a 2 , . . . , a 100 au proprietatea că N = 6 a1 + 6 a2 +. . . + 6 a 100este pătrat perfect. Arătaţi că numărul a 1 + a 2 + . . . + a 100 se divide cu 5.Andrei Eckstein, TimişoaraSoluţie. Deoarece 6 n = M 5 + 1, ∀n ∈ N, rezultă că N . .5. Cum N este pătratperfect, deducem că N . .25. Însă, întrucât 6 n = M 25 + 5n + 1, ∀n ∈ N, avem căN = M 25 + 5(a 1 + a 2 + . . . + a 100 ) + 100 şi de aici rezultă concluzia.VIII.140. Fie n ∈ N ∗ şi x 1 , x 2 , . . . , x n ∈ Z\{n} astfel încât n 3 +x 2 1+x 2 2+. . .+x 2 n ≤n[1 + 2(x 1 + x 2 + . . . + x n )]. Demonstraţi că x 1 , x 2 , . . . , x n ∈ N.Dan Nedeianu, Drobeta Tr. SeverinSoluţie. Din ipoteză obţinem că [(x 1 −n) 2 −1]+[(x 2 −n) 2 −1]+. . .+[(x n −n) 2 −1] ≤0. Cum x k ≠ n, rezultă că (x k − n) 2 ≥ 1, ∀k = 1, n, prin urmare fiecare dintreparantezele pătrate este nenegativă. Deducem că fiecare dintre ele este nulă şi atuncix k ∈ {n − 1, n + 1}, ∀k = 1, n, adică x k ∈ N, ∀k = 1, n.55AMQDS NV.POBRC


VIII.141. Dacă a, b, c, x, y, z > 0 şi ax + by + cz = 1, demonstraţi că ayz + bxz +cxy ≥ 27abc.D.M. Bătineţu-Giurgiu, BucureştiSoluţie. Eliminând numitorii, inegalitatea de demonstrat revine la ax+by +cz ≥27abcxyz, adică la 1 ≥ 27abcxyz. Însă (ax + by + cz)3 ≥ 27 · ax · by · cz (inegalitateamediilor) şi de aici rezultă ceea ce dorim. Egalitatea se atinge pentru x = 13a , y = 1 3b ,z = 1 3c .VIII.142. Fie a, b, c, d ∈ R şi E(x, y) =pa 2 x 2 + b 2 y 2 , F (x, y) =pc 2 x 2 + d 2 y 2 ,∀x, y ∈ R. Dacă |ad| = |bc|, demonstraţi că(∗)ÈE(x, z) · F (x, z) ≤ÈE(x, y) · F (x, y) +ÈE(y, z) · F (y, z), ∀x, y, z ∈ R.Valentina Blendea şi Gheorghe Blendea, IaşiSoluţie. Dacă b = 0, atunci E(x, y) = |ax|, ∀x, y ∈ R. Din |ad| = |bc|, obţinemcă a = 0 sau d = 0. În primul caz, (*) devine 0 ≤ 0, iar în al doilea, (∗) ⇔È|ax| · |cx| ≤È|ax| · |cx| +È|ay| · |cy|, adevărat. Analog se tratează cazul în carec = 0.Dacă bc ≠ 0, atunci ad ≠ 0 şi putem scrie căaObservămcă E(x, y) = |b| ·pk 2 x 2 + y 2 , F (x, y) = |d| ·pk 2 x 2 + y 2 , deci (*) revineb=cd=k>0.la √ k 2 x 2 + z 2 ≤pk 2 x 2 + y 2 +pk 2 y 2 + z 2 ; o simplă ridicare la pătrat arată căaceastă ultimă inegalitate este adevărată.a 2011VIII.143. Dacăa 2 + b 2 = b2011b 2 + c 2 = c2011c 2 , demonstraţi că numerele reale+ a2 pozitive a, b şi c sunt egale.Cristina Ene, elevă, CraiovaSoluţie. Dacă presupunem că a < b, atunci 2a 2011 < a 2011 + a 2009 · b 2 , de undea 2011a 2 + b 2c 2011< a20092 . Deducem că b 2011b 2 + c 2< a20092< b2009, deci b < c. Obţinem că2c 2 + a 2 < c2009, prin urmare c < a şi ajungem la contradicţia a < b < c < a. Analog2se arată că nu putem avea a > b şi rămâne că a = b, apoi a = b = c.Clasa a IX-a+ sinIX.121. Arătaţi că1 4 x + cos 4 x1 + sin 4 =y + cos y21 + sin8 x + cos 8 x4 1 + sin 8 , ∀x, y ∈ R.y + cos 8 yMihály Bencze, BraşovSoluţie. Folosind identitatea (a 4 + b 4 + (a 2 + b 2 ) 2 ) 2 = 2(a 8 + b 8 + (a 2 + b 2 ) 4 ),∀a, b ∈ R şi scriindu-l pe 1 ca (sin 2 + cos 2 x) 2 în stânga respectiv (sin 2 x + cos 2 x) 4 îndreapta, obţinem ceea ce dorim.56


IX.122. Fie a, b, c ∈ R cu b ≥ c > 0 şi a b ≥ 1 + √ 5. Dacă numerele a, b, c pot fi2laturile unui triunghi, demonstraţi că şi 1 a , 1 b , 1 pot fi laturi ale unui triunghi.cSoluţie. Evident că a > b ≥ c (deoarece 1 + √ 52Ovidiu Pop, Satu Mare> 1) şi c > a − b (întrucât a, b, cpot fi laturi ale unui triunghi). Întrucât 1 a < 1 b ≤ 1 , va fi suficient să demonstrămccă 1 c < 1 a + 1 , echivalent cu c >abba − b >a + b .Vom arăta chiar mai mult, anume căaba + b ; după calcule, această inegalitate revine la a2 − ab − b 2t 2 − t − 1 ≥ 0, unde t = a ∈–1 + √ 5, ∞Œ, fapt evident adevărat.b 2≥ 0 sauIX.123. Considerăm patrulaterul ABCD şi fie M, N, P, Q mijloacele laturilorAB, BC, CD respectiv DA, iar T un punct interior patrulaterului. Notăm cu G 1 , G 2 ,G 3 şi G 4 centrele de greutate ale patrulaterelor T NCP, T P DQ, T QAM, T <strong>MB</strong>N.Arătaţi că −−→ AG 1 + −−→ BG 2 + −−→ CG 3 + −−→ DG 4 = −→ 0 dacă şi numai dacă {T } = MP ∩ NQ.Florin Stănescu, GăeştiSoluţie. Fie G centrul de greutate al patrulaterului ABCD, adică {G} = MP ∩NQ; atunci −−→ AG 1 + −−→ BG 2 + −−→ CG 3 + −−→ DG 4 = −→ 0 ⇔ 1 4 (−→ AT + −−→ AN + −→ AC + −→ AP ) + 1 4 (−→ BT +−→BP + −−→ BD + −→ 1 BQ) +4 (−→ CT + −→ −→ −−→ 1CQ + CA + CM) +4 ( −→ −−→ −−→ −−→DT + DM + DB + DN) =−→ −→ −→ −→ −→ 10 ⇔ ( AT + BT + CT + DT ) +2 ( −→ −→ 1AB + AC) +2 ( −→ −→ 1AD + AC) +2 ( −→ −−→ BC + BD) +12 ( −→ −−→ 1BA + BD) +2 (−−→ CD + −→ CA) + 1 2 (−→ CA + −→ 1 CB) +2 ( −→ −−→ 1DA + DB) +2 (−−→ DB + −−→ DC) = −→ 0 ⇔4 −→ GT = −→ 0 ⇔ G = T ⇔ {T } = MP ∩ NQ.IX.124. Dacă ABCD este un patrulater inscriptibil, arătaţi că BC 2 · S ACD +CD 2 · S ABC = AC 2 · S BCD .D.M. Bătineţu-Giurgiu, BucureştiSoluţie. Din teorema lui Ptolemeu, BC · AD + CD · AB = AC · BD. Atunci:BC 2 · S ACD + CD 2 · S ABC = BC 2 · AD · CD · sin(π − B)++ CD 2 · AB · BC · sin B = BC · CD · (BC · AD + CD · AB) · sin B =BD · sin B= BC · CD · AC · BD · sin B = 2 · AC · S BCD · =sin C= 2 · AC · S BCD · 2R · sin B = 2 · AC 2 · S BCD .IX.125. Dacă x, y ∈ R, x > y > 1, arătaţi că xy + 4 > x + 3y.Dan Nedeianu, Drobeta Tr. SeverinSoluţie. Considerăm funcţia f : [y, ∞) → R, y > 1 fixat, f(x) = x(y −1)+4−3y.Funcţia f este stirct crescătoare (deoarece y − 1 > 0) şi, cum f(y) = (y − 2) 2 ≥ 0,rezultă că pentru x > y, f(x) > f(y) ≥ 0 şi de aici inegalitatea cerută.57


Clasa a X-aX.121. Dacă a ∈ R ∗ +, rezolvaţi în R ecuaţia (2a) x2 · a x = 2.Luminiţa Mihalache, CraiovaSoluţie. Observăm că ecuaţia poate fi scrisă sub forma (2 x−1 ·a x ) x+1 = 1. Atuncix + 1 = 0, de unde obţinem soluţia x 1 = −1, sau 2 x−1 · a x = 1, adică (2a) x = 2,ecuaţie care are soluţia x 2 = log 2a 2 dacă a ≠ 1 2 şi care nu are soluţii când a = 1 2 .X.122. Demonstraţi că triunghiul ABC este echilateral dacă şi numai dacă h aac +h bab + h cbc = h abc + h bac + h c(notaţiile sunt cele uzuale).abPetruSoluţie. Înlocuind h a = 2S a , h b = 2S b , h c = 2S c3É 1a 2 c + 1b 2 a + 1c 2 b = 3abc . Însă 1a 2 c + 1b 2 a + 1c 2 b ≥ 3 ·1a 3 b 3 c 3 = 3abcb 2 a = 1c 2 bAsaftei, Iaşi, relaţia din enunţ devine, prin urmareegalitatea din problemă are loc dacă şi numai dacă 1a 2 c = 1 , i.e. a = b = c.X.123. Dacă n ∈ N, n ≥ 3 şi x ∈ (−1, 1), demonstraţi inegalitatea n( n√ 1 + x +n√ √ √ 1 − x) ≤ 2( 1 + x + 1 − x + n − 2).Lucian Tuţescu şi Petrişor Rocşoreanu, CraiovaSoluţie. Folosind inegalitatea mediilor, obţinem căn√ nÈ√ √ 2 √ 1 + x + n − 21 + x = 1 + x · 1 + x · 1 · 1 · . . . · 1 ≤, (factorul 1 de n − 2nori) şi, analog,n√ 2 √ 1 − x + n − 21 − x ≤ . Adunând aceste inegalităţi, obţinemnconcluzia problemei. Egalitatea se atinge când x = 0.X.124. Aflaţi numerele complexe nenule x, y, z cu proprietatea căx(x + y)(x + z) = y(y + x)(y + z) = z(z + x)(z + y) = −1.Vasile Chiriac, BacăuSoluţie. Se impun condiţiile x + y ≠ 0, y + z ≠ 0, z + x ≠ 0, deoarece altfel seajunge la contradicţia 0 = 1. Scăzând două câte două ecuaţiile sistemului, obţinem că(x − y)(x + y + z) = (y − z)(x + y + z) = (z − x)(x + y + z) = 0. Distingem situaţiile:(i) x = y = z; atunci 4x 3 = −1 şi sunt‹k soluţii ale sistemului tripletele (x, x, x), cu(2k + 1)π (2k + 1)πx ∈§13√ cos + i sin = 0, 1, 2ª;4 33(ii) x = y ≠ z; atunci z = −2x şi obţinem soluţiile (x, x − 2x), cu x ∈ A =§13√ cos 2kπ 2kπ+ i sin3‹k = 0, 1, 2ª. În cazul în care y = z ≠ x găsim soluţiile2 3(−2y, y, y), y ∈ A, iar în cazul în care x = z ≠ y obţinem tripletele (x, −2x, x), x ∈ A.(iii) x ≠ y ≠ z ≠ x; atunci x + y + z = 0 şi fiecare dintre ecuaţile sistemuluirevine la xyz = −1. Prin urmare, x + y = −z şi xy = − 1 , deci x şi y sunt soluţiilezecuaţiei t 2 + tz − 1 −z + u= 0. Deducem că x = , y = −z − u , unde z ∈ C estez 2258


astfel încât u 2 = z 2 + 4 . Cum x, y, z sunt distincte, impunem ca u ≠ 0, u ≠ ±3z,zadică z 3 ≠ −4, z 3 ≠ − 1 2 şi obţinem soluţiile−v + u, −v − u , v, cu v ∈ C, v ≠ 0,2 2v 3 ≠ −4, v 3 ≠ − 1 2 şi u2 = v 2 + 4 v .În concluzie, sistemul dat admite o infinitate de soluţii.X.125. Dacă x, y ∈ N sunt astfel încât numărulpx 2 + 2y + 1+ 3py 3 + 3x 2 + 3x + 1este raţional, arătaţi că x = y.Gheorghe Iurea, Iaşi√Soluţie. √ Pentru început, vom arăta că dacă a, b ∈ R, a ≥ 0 sunt astfel încâta +3b ∈ Q, atunci √ a ∈ Q şi3√ √ b ∈ Q. Într-adevăr, fie x = a + 3√ b ∈ Q; din3√ √ b = x − a, deducem că b = x 3 − 3x 2 · √a + 3a − a √ a, deci √ a = x3 + 3ax − b3x 2 ∈ Q+ a(dacă 3x 2 + a = 0, concluzia este evidentă). Din x, √ a ∈ Q, urmează că 3√ b ∈ Q.În aceste condiţii, din ipoteza problemei rezultă că x 2 + 2y + 1 este pătrat perfect,strict mai mare ca x 2 , aşadar x 2 + 2y + 1 ≥ (x + 1) 2 , de unde y ≥ x. Analog,y 3 +3x 2 +3x+1 este cub perfect, strict mai mare ca y 3 , deci y 3 +3x 2 +3x+1 ≥ (y+1) 3şi de aici (x − y)(x + y + 1) ≥ 0, prin urmare x ≥ y. În final, deducem că x = y.Clasa a XI-a1 sin A sin C 2 2XI.121. Dat triunghiul ABC, arătaţi că sin A 1 sin B ≤ 1 22sin C sin B 2 .12 2Bogdan Victor Grigoriu, FălticeniSoluţie. Într-un triunghi ABC este adevărată egalitatea sin 2 A 2 + sin2 B 2 +sin 2 C 2 = 1 − 2 sin A 2 sin B 2 sin C şi atunci valoarea determinantului din enunţ este24 sin A 2 sin B 2 sin C 2 = 4r(p − b)(p − c) − a)(p − c) − a)(p − b)·r(p·r(p= 4S2bcacab pabc =SRp = r R . Conform inegalităţii lui Euler R ≥ 2r, acest din urmă raport este ≤ 1 2 .XI.122. Fie A, B ∈ M n (Q) cu A 2 + B 2 = −2I n ; arătaţi că det(AB + BA) ≥ 0.Dumitru Crăciun, FălticeniSoluţie. Observăm că (A + B + i √ 2I n )(A + B − i √ 2I n ) = A 2 + B 2 + 2I n + AB +BA = AB + BA şi atunci det(AB + BA) = det(A + B + i √ 2I n )det(A + B − i √ 2I n ) =1 + a 2 0 . . .’0A 0 0 1 . . . 0=ˆa−1 −a 0 . . . 0. . . . . . . . . . . . . . . . . . . .0 0 0 . . . 11 + b 2 0 . . .’0şi B 0 0 1 . . . 0 , a, b ∈ Q,=ˆb−1 −b 0 . . . 0. . . . . . . . . . . . . . . . . . . .0 0 0 . . . 1det(A + B + i √ 2I n ) · det(A + B + i √ 2I n ) = (det(A + B + i √ 2I n )) 2 ≥ 0.Notă. Există matrice A, B care verifică ipoteza A 2 + B 2 = −2I n . De exemplu,59


au proprietatea că A 2 = B 2 = −I n , deci A 2 + B 2 = −2I n .XI.123. Fie A, B ∈ M 2 (R) astfel încât detA = 10, detB = 12, tr A = tr B = 7.Determinaţi numerele naturale n pentru care tr A n = tr B n .Răzvan Ceucă, elev, IaşiSoluţie. Cum detA = 10 şi trA = 7, valorile proprii ale matricei A sunt 2 şi 5,deci trA n = 2 n + 5 n . Analog se arată că trB n = 3 n + 4 n şi atunci condiţia din enunţrevine la 3 n − 2 n = 5 n − 4 n . Evident că n = 0 şi n = 1 sunt soluţii. Dacă n ≥ 2,aplicând teorema lui Lagrange funcţiei f(x) = x n pe intervalele [2, 3] şi [4, 5], găsimc ∈ (2, 3) şi d ∈ (4, 5) pentru care 3 n − 2 n = nc n−1 şi 5 n − 4 n = nd n−1 , iar egalitateanc n−1 = nd n−1 cu n ≥ 2, c ≠ d, nu este posibilă.XI.124. Calculaţi limn→∞É2 20 +q2 21 +È2 22 + . . . + √ 2 2n + 1.Gheorghe Iurea, IaşiSoluţie. Dacă a n este şirul din enunţ, atunci a n = 2Ê1 +r1 + . . . +É1 + 12 2n ,numărul radicalilor fiind n + 1. Considerăm şirul x n =É1 +q1 + . . . +È1 + √ 2(n + 1 radicali), care verifică relaţia de recurenţă x 0 = √ 2, x n+1 = √ 1 + x n , ∀n ∈ N.Se arată că (x n ) este convergent, cu lim x n = 1 + √ 5. Cum 2x n−1 < a n < 2x n ,n→∞ 2∀n ∈ N ∗ , rezultă că lim a n = 1 + √ 5.n→∞XI.125. Demonstraţi că ecuaţia 25 x + 4 x = 10 x + 9 x are cel puţin o soluţie realănegativă.Ionuţ Ivănescu, CraiovaSoluţie. Considerăm funcţia f : R → R, f(x) = 25 x + 4 x − 10 x − 9 x şi săpresupunem prin absurd că f(x) > 0, ∀x ∈ − 1 2 , 0‹. Deducem că 25x − 1+ 4x − 1 0 şi, cum f esteClasa a XII-aXII.121. Fie a ∈ N ∗ şi G = (a, +∞) pe care definim operaţia x ∗ y = (x − a)(y −a) + a, ∀x, y ∈ G. Dacă H este subgrup al lui G astfel încât N ∩ G ⊂ H, arătaţi căQ ∩ G ⊂ H.D.M. Bătineţu-Giurgiu, Bucureşti şi Neculai Stanciu, BuzăuSoluţie. Grupul (G, ∗) are elementul neutru e = 1 + a, iar simetricul lui x estex ′ = a + 1x − a ∈ G. Fie q ∈ Q ∩ G; atunci există m, n ∈ N∗ încât q = a + m n . Dacă60


p ∈ N ∩ G, atunci p ∈ H, prin urmare p + a ∈ H şi (p + a) ′ = a + 1 ∈ H. Deducempcă a + m şi a + 1 sunt în H, deci (a + m) ∗n aastfel Q ∩ G ⊂ H.+ 1 n‹=a + m n∈ H, adică q ∈ H şiXII.122. Fie n ∈ N, n ≥ 2 şi polinomul f = X n − 2nX n−1 + (2n 2 − 4)X n−2 +a 3 X n−3 + . . . + a n ∈ C[X]. Demonstraţi că f are toate rădăcinile reale dacă şi numaidacă n = 2.Florin Stănescu, GăeştiSoluţie. Dacă n = 2, atunci f = X 2 −4X+4 are rădăcinile x 1 = x 2 = 2. Reciproc,fie x 1 , x 2 , . . . , x n rădăcinile lui f, presupuse reale. Cum (x 1 + x 2 + . . . + x n ) 2 ≤n(x 2 1 + x 2 2 + . . . + x 2 n), rezultă că 4n 2 ≤ 8n, deci n ≤ 2 şi atunci n = 2.XII.123. Calculaţi I =Zarccos0√659tg x √ sin xdx.Vasile Chiriac, Bacău9 s √ sSoluţie. Cu schimbarea de variabilă sin x = s, obţinem că I =Z40 1 − s 2 ds.Apoi, substituţia √ s = t conduce laI =Z2302t 41 − t 4 dt = −2t + arctg t − 1 2 ln 1 − tt‹231 +0= − 4 3 + arctg 2 3 + ln √ 5.XII.124. Fie f : R → R o funcţie continuă cu proprietatea că (f ◦ f)(x) = sin x,∀x ∈ R. Demonstraţi căZ1f(x)dx ≤ 1.0Dumitru Crăciun, FălticeniSoluţie. Din (f ◦f)◦f = f ◦(f ◦f), deducem că sin f(x) = f(sin x), ∀x ∈ R; atuncif(sin x) ≤ 1, ∀x ∈ R, prin urmare f(sin x) · cos x ≤ cos x, ∀x ∈h0, π Integrând pe2i.h0, π 2rezultă căZπf(sin x) · (sin x)2i, ′ 2dx ≤ sin xπ, adicăZ1f(x)dx ≤ 1.0Rămâne întrebarea: există vreo funcţie f care să verifice ipoteza?XII.125. Fie f : [0, 1] → R o funcţie derivabilă cu f ′ integrabilă. Dacă f(0) = 0,arătaţi căZ1(f ′ (t)) 2 dt ≥Z1f 2 (t)dt.0Soluţie. Pentru x ∈ [0, 1], avem căf(x) =Zxf ′ (t)dt ≤Zx000|f ′ (t)|dt ≤ÊZx000Ciprian Baghiu, Iaşi(f ′ (t)) 2 ≤ÊZ1dt (f ′ (t)) 2 dt,0deci f 2 (x) ≤Z1(f ′ (x)) 2 dx. Integrând pe [0, 1], obţinem cerinţa problemei.061


Soluţiile problemelor pentru pregătireaconcursurilor propuse în nr. 2/2011A. Nivel gimnazialG206. Câte submulţimi ale mulţimii M = {1, 2, 3, . . . , 100} au 50 de elemente şinu conţin nicio pereche de numere consecutive?Gheorghe Iurea, IaşiSoluţie. Fie A = {x 1 , x 2 , . . . , x 50 } cu x 1 < x 2 < . . . < x 50 şi x 2 − x 1 ≥ 2,x 3 − x 2 ≥ 2, . . . , x 50 − x 49 ≥ 2. Evident, cel mult una dintre aceste inegalităţi poatefi strictă. Dacă toate inegalităţile se transformă în egalităţi, atunci A = {x 1 , x 1 +2, . . . , x 1 + 98}, cu x 1 = 1 sau x 1 = 2. Dacă există o inegalitate strictă, atunciA = {1, 3, . . . , 2n − 1, 2n + 2, 2n + 4, . . . , 100}, cu n ∈ {1, 2, . . . , 49}. În total, obţinem2 + 49 = 51 de submulţimi A cu proprietăţile din enunţ.G207. Arătaţi că numărul N = 2 2009 + 3 2010 + 4 2011 nu este pătrat perfect.Andrei Eckstein, TimişoaraSoluţia 1. Un pătrat perfect dă, la împărţirea prin 13, unul dintre resturile0, 1, 3, 4, 9, 10 sau 12. Pe de altă parte, avem că 2 2009 = 2 5 (2 6 ) 334 = 32(M 13 + 1) 334 =32(M 13 + 1) = M 13 + 6; 3 2010 = 27 670 = (M 13 + 1) 670 = M 13 + 1; 4 2011 = 4(4 3 ) 670 =4(M 13 + 1) 670 = 4(M 13 + 1) = M 13 + 4, prin urmare N = M 3 + 11 şi atunci N nupoate fi pătrat perfect.Soluţia 2. Cum U(2 2009 ) = 2, U(3 2010 ) = 9, U(4 2011 ) = 4, rezultă că U(N) = 5.Dacă N ar fi pătrat perfect, penultima sa cifră ar trebui să fie 2. Puterile lui 2 îşirepetă ultimele două cifre din 20 în 20 şi, cum 2009 = M 20 +9, atunci 2 2009 are aceleaşiultime două cifre ca şi 2 9 , adică 12. Puterile lui 3 îşi repetă ultimele două cifre totdin 20 în 20; întrucât 3 10 = 243 2 = . . . 49, rezultă că 3 2010 = . . . 49. Puterile lui 4 îşirepetă ultimele două cifre din 10 în 10, aşadar 4 2011 = . . . 04, fiindcă 2011 = M 10 + 1.În final, N = . . . 12 + . . . 49 + . . . 04 = . . . 65 şi de aici urmează că N nu poate fi pătratperfect.G208. Demonstraţi că ecuaţia x 2 + y 2 = z(x + y + 1) are o infinitate de soluţiiîn mulţimea numerelor naturale.Cosmin Manea şi Dragoş Petrică, PiteştiSoluţie. Notăm x + y + 1 = t ∈ N; atunci z = x2 + (t − x − 1) 2= t − 2x − 2 +t2x 2 + 2x + 1ar trebui să fie tot număr natural. Alegem t = 2x 2 + 2x + 1, x ∈ N şitobţinem că y = 2x 2 + x, z = 2x 2 . În concluzie, tripletele de forma (n, 2n2 + n, 2n 2 ),n ∈ N, sunt soluţii ale ecuaţei date.G209. Rezolvaţi în numere naturale ecuaţia 4abc = (a + 2)(b + 1)(c + 1).Titu Zvonaru, ComăneştiSoluţie. Evident că a, b, c sunt nenule. Dacă a = 1, ecuaţia poate fi adusă la forma(b − 3)(c − 3) = 12, deci obţinem soluţiile (1, 15, 4); (1, 4, 15); (1, 9, 5); (1, 5, 9); (1, 7, 6);(1, 6, 7). Dacă b = 1, ajungem la (a − 2)(c − 1) = 4 şi găsim soluţiile (3, 1, 5); (4, 1, 3);62


(6, 1, 2). Analog, când c = 1 vom avea soluţiile (3, 5, 1); (4, 3, 1); (6, 2, 1). În continuare,fie a, b, c ≥ 2. Din a = ≥ 2 rezultă că (b − 1)(c − 1) ≤ 22(b + 1)(c + 1)4bc − (b + 1)(c + 1)şi, de aici, (b, c) ∈ {(2, 2); (2, 3); (3, 2)}. Obţinem atunci soluţiile (2, 2, 3) şi (2, 3, 2).G210. Demonstraţi că fracţia a3n+2 − a 3n+1 + (−1) na 3n+8 − a 3n+7 + (−1) n este reductibilă pentruorice a, n ∈ N, a ≥ 2.Dan Popescu, SuceavaSoluţie. Amplificând fracţia cu (−1) n şi notând b = −a, ar trebui să arătămcă fracţia b3n+2 + b 3n+1 + 1b 3n+8 + b 3n+7 + 1 este reductibilă pentru n ∈ N. Observăm că b3n =(b 3 − 1 + 1) n = M(b 3 − 1) + 1 = M(b 2 + b + 1). Rezultă că b 3n+2 + b 3n+1 + 1 =[M(b 2 +b+1)+b 2 ]+[M(b 2 +b+1)+b]+1 = M(b 2 +b+1) şi, analog, b 3n+8 +b 3n+7 +1 =M(b 2 +b+1), deci fracţia în b se simplifică prin b 2 +b+1 ≥ 3 (deoarece b ∈ Z, b ≤ 2).G211. Demonstraţi că expresia‹2 ‹2x 2 (a 1 + a 2 ) + x 3 a 1 x 1 (a 1 + a 2 ) + x 3 a 2E = y 1 + y 2 +x 1 + x 2 + x 3 x 1 + x 2 + x 3x 1 a 1 − x 2 a 21+ y 3 −·x 1 + x 2 + x 3‹2y 1 + y 2 + y 3x 2 (a 1 + a 2 ) + x 3 a 1 x 1 (a 1 + a 2 ) + x 3 a 2 x 1 a 1 − x 2 a 2·•−y 1 + y 2 + y 3 ,x 1 + x 2 + x 3 x 1 + x 2 + x 3 x 1 + x 2 + x 3˜2unde a i , x i , y i ∈ R ∗ +(i = 1, 2, 3), nu depinde x 1 , x 2 , x 3 .Mircea Bîrsan, Iaşi1Soluţie. Se scoate forţat în factor(x 1 + x 2 + x 3 ) 2 şi, după calcule(y 1 + y 2 + y 3 )ce pun în evidenţă y 1 y 2 , y 2 y 3 , y 3 y 1 , se obţineE =1y 1 + y 2 + y 3[y 1 y 2 (a 1 + a 2 ) 2 + y 2 y 3 a 2 2 + y 3 y 1 a 2 1].G212. Se consideră triunghiul ABC cu m(ÒB) = 135 ◦ şi m(ÒC) = 30 ◦ . Determinaţimăsurile unghiurilor triunghiului ABD, unde D este simetricul lui C faţă de B.AEugeniu Blăjuţ, BacăuSoluţia 1 (a autorului). Fie M = Pr BC A şi O mijloculsegmentului AC. Se arată imediat că △MAB esteOdreptunghic isoscel şi că △AOM este echilateral, prinurmare MO = <strong>MB</strong>, iar m(ÖO<strong>MB</strong>) = 30 ◦ . Rezultă căm(ÖOBM) = 75 ◦ , deci m(ÕOBC) = 105 ◦ . Pe de altă parte, M D B COB este linie mijlocie în △CAD, aşadar OB∥AD şi atunci m(ÕADC) = 105 ◦ , apoim(ÕDAB) = 30 ◦ .63


Soluţia 2 (Titu Zvonaru). Considerăm punctul D ′ pe semidreapta opusă lui(BC astfel încât m(ÖBAD ′ ) = 30 ◦ . Triunghiul ACD ′ constituie configuraţia din problemaVI.143. Cum m(ÕCAB) = 15 ◦ , rezultă că B este mijlocul lui CD ′ , adică D ′coincide cu D. Deducem acum că m(ÕBAD) = 30 ◦ , m(ÕDBA) = 45 ◦ , m(ÕADB) = 105 ◦ .G213. Se consideră triunghiul ABC cu proprietatea că există M şi N puncte îninteriorul său astfel încât BN=CM şi △ABM∼△ACN. Demonstraţi că AB=AC.Crisitan Lazăr, IaşiSoluţie. Avem că ABAC = AMAN = BMCN = k şi m(ÕBAN) =m(ÖCAM) = α. Folosind teorema cosinusului în triunghiurileABN şi ACM şi ţinând seama de condiţia BN = CM, obţinemcă AB 2 + AN 2 − 2AB · AN cos α = AC 2 + AM 2 − 2AM ·AC cos α ⇔ k 2 AC 2 + AN 2 − 2k · AC · AN · cos α = AC 2 +k 2 AN 2 −2k·AN ·AC ·cos α ⇔ k 2 AC 2 +AN 2 = AC 2 +k 2 AN 2 ⇔(k 2 − 1)(AC 2 − AN 2 ) = 0. Întrucât N ∈ Int ABC, nu putemavea AN = AC; ar rezulta că AM = AB, deci m(ÕACN) =12 (180◦ − α) < m(ÒC) şi m(ÖABM) = 1 2 (180◦ − α) < m(ÒB);sumând, am obţine că 180 ◦ − α < m(ÒB) + m(ÒC) şi de aici contradicţia α > m(bA).Rezultă că AN ≠ AC şi atunci k 2 − 1 = 0, deci k = 1 şi de aici urmează cerinţaproblemei.AG214. Se consideră triunghiul isoscel ABC cu AB = AC şim(bA) < 90 ◦ . Construim înălţimea CF şi fie E mijlocul segmentuluiBF , iar D un punct pe segmentul BC. DacăÕADE ≡ÒB,arătaţi că D este mijlocul segmentului BC.Claudiu Ştefan Popa şi Gabriel Popa, IaşiSoluţia 1. Fie M mijlocul lui (BC) şi să presupunem prinabsurd că M ≠ D; considerăm că D ∈ (MC), cazul D ∈ (BM)tratându-se similar. Cum ME este linie mijlocie în △BCF ,rezultă că ME∥CF, deci ME ⊥ AB. Deducem că m(ÖAME) =90 ◦ − m(ÖBAM) = m(ÒB), prin urmareÖAME ≡ÕADE. AtunciB M D Cpatrulaterul ADME va fi inscriptibil şi rezultă că m(ÕAED) = m(ÖAMD) = 90 ◦ . Astfel,în E am putea ridica două perpendiculare distincte pe AB (anume ED şi EM),fapt imposibil! Rămâne că M ≡ D, adică D este mijlocul segmentului BC.Soluţia 2. Deoarece △ADE ∼ △ADB, rezultă că AD 2 = AE · AB. Dacă Meste mijlocul lui BC, cum ME ⊥ AB, deducem că AM 2 = AE · AB. Obţinem căAM = AD, cu M, D ∈ (BC) şi, de aici, M ≡ D.G215. În planele paralele P 1 şi P 2 se consideră cercurile C 1 = C(O 1 , R 1 ), respectivC 2 = C(O 2 , R 2 ). Fie K 1 conul de vârf O 2 şi bază C 1 şi K 2 conul de vârf O 1 şi bază C 2 .Arătaţi că intersecţia celor două conuri este un cerc şi determinaţi poziţia centruluişi mărimea razei acestuia.Temistocle Bîrsan, Iaşi64BEFMANC


Soluţie. Fie P un semiplan limitat de linia centrelor O 1 O 2 , {A 1 } = P ∩ C 1şi {A 2 } = P ∩ C 2 . În semiplanul P , generatoarele O 2 A 1 şi O 1 A 2 se intersecteazăîn M, iar paralela prin M la A 1 O 1 taie O 1 O 2 în O. Din AO△MO 1 A 1 ∼△MA 2 O 2 rezultă că MA 1MO 2= R 1R 2. Cu teorema luiThales obţinem că OO 1= R 1, prin urmare OO 2 = ·OO 2 R 2 R 1 + R 2O 1 O 2 (1), relaţie ce determină poziţia lui O pe segmentulO 1 O 2 . În △O 2A 1 O 1 cu MO∥A 1 O 1 avem căMO = O 2O,A 1 O 1 O 2 O 1R 22 2adică MO = R 1R 2R 1 + R 2(2).În concluzie, conurile K 1 şi K 2 se intersectează după un cerc situat în planul paralelcu P 1 (şi cu P 2 ) şi care trece prin punctul O precizat de (1), are centrul în O şi razaOM dată de (2).Notă. Problema poate fi generalizată la cazul în care vârfurile V 1 şi V 2 aleconurilor K 1 şi K 2 sunt situate arbitrar pe dreapta O 1 O 2 (şi nu neapărat V 1 ≡ O 2şi V 2 ≡ O 1 ). Situaţia se reduce la cea tratată anterior prin înlocuirea planelor P 1 şiP 2 cu planele P ′ 1 (paralel cu P 1 prin punctul V 2 ) şi, respectiv, P ′ 2 (paralel cu P 2 prinpunctul V 1 ).B. Nivel licealL206. Fie P un punct pe mediana din A a triunghiului ABC. Paralela prin P laAC taie AB în M, iar simetricul lui P faţă de mijlocul lui AC este N. Arătaţi căMN∥BC dacă şi numai dacă P este centrul de greutate al triunghiului ABC.Silviu Boga, IaşiSoluţie.Fie T mijlocul lui (BC), k = APATşi {R} = AN ∩ BC. Din MP ∥QT rezultă că AMAQ = APAT∈ (0, 1), T Q∥MP (cu Q ∈ AB)A1MOO= k; însă T Q este liniemijlocie în △ABC, prin urmare AQ = 1 AB şi deducemcă AMMA2AB = k . Observăm că AP CN este paralelogram,aşadar AT ∥CN, de unde RNQ P2 NRA= NCAT =APAT= k, deci ANAR = 1 − k. Astfel, MN∥BC ⇔A<strong>MB</strong> T C RAB = ANAR ⇔ k 2 = 1 − k ⇔ k = 2 ⇔ P este centrul de greutate al △ABC.3Notă. Soluţie corectă a dat dl. Ioan Viorel Codreanu, Satulung (Maramureş).L207. Fie ABCD un patrulater convex şi M, N, P puncte pe segmentele AB, CDrespectiv BC astfel încât <strong>MB</strong>AB= NDDC = BPBC = k. Dacă R şi S sunt mijloaceleRSsegmentelor AP respectiv MN, calculaţi (în funcţie de k) raportulAD .Titu Zvonaru, Comăneşti651


Soluţia 1 (a autorului). Fie T intersecţia dreptei AC cu paralela prin N la AD.Deoarece CTDAT = CNND = CP , rezultă că T P ∥AM.P BÎnsă MP ∥AC, prin urmare AMP T este paralelogram,unde mijlocul R al diagonalei AP va fi ATNmijloc şi pentru MT . Astfel, RS este linie mijlocieîn △MT N şi atunci RS = 1 R2 NT . CumST NMAD = CNRS= 1 − k, rezultă căCD AD = 1 − k .2Să notăm că RS∥T N∥AD, aşadar −→ RS =1 − k −→B PC· AD.2Soluţia 2 (Gheorghe Iurea). În planul complex, vom nota cu x afixul punctuluiX. Obţinem imediat că m = b(1 − k) + ak, p = b(1 − k) + ck, n = d(1 − k) + ck,r = 1 2 (a + p), s = 1 2 (m + n). Atunci s − rd − a = 1 − k ∈ R + şi de aici urmează că2RS∥AD, iar RSAD = 1 − k .2Notă. O soluţie folosind calculul vectorial a dat dl. Ioan Viorel Codreanu,Satulung (Maramureş).L208. Un cilindru circular drept de axă d şi rază R 1 şi o sferă de centru O şirază R 2 sunt tangente exterior în punctul A. Fie B simetricul lui A în raport cu dşi fie π planul ce trece prin B, este perpendicular pe planul determinat de O şi d şiface cu axa d un unghi de 30 ◦ . Calculaţi raportul razelor celor două suprafeţe ştiindcă secţiunile lor cu planul π au arii egale.Temistocle Bîrsan, IaşiSoluţie. Figura indică secţiunea cilindrului şi sferei cu planul determinat deO şi d. Dreapta BD este intersecţia acestuia cu planulDdπ. Cilindrul este secţionat de π după o elipsă cu lungimilesemiaxelorER 1Csin 30 ◦ = 2R 1 şi R 1 , iar sfera dupăun cerc cu raza dată de ED 2 = OD 2 − OE 2 = OD 2 − B A OOB 2 cos 2 30 ◦ = R2 2 − 3 4 (2R 1 + R 2 ) 2 = 1 4 R2 2 − 3R1 2 −3R 1 R 2 . Egalitatea ariilor secţiunilor revine la π · 2R 1 ·R 1 = π( 1 4 R2 2 − 3R 2 1 − 3R 1 R 2 ) sau, cu notaţia k = R 2R 1, k 2 − 12k − 20 = 0, de undek = 6 + √ 56. Aşadar, R 2= 2(3 + √ 14).R 1L209. Se consideră triunghiul ABC şi punctele M, N, P, Q, R, S definite prin−−→BM = k · −−→ MC, −−→ CN = k · −−→ NA, −→ −→ −−→ −−→ −−→ −−→AP = k · P B, AM = p · MQ, BN = p · NR,−→CP = p · −→ P S, unde k, p ∈ R ∗ \{−1}. Demonstraţi că S MNP ≥ 1 4 · S ABC, iar S QRS ≥p + 32p‹2· S ABC .66Marius Olteanu, Rm. Vâlcea


Soluţia 1. Raportăm planul la un reper cartezian faţă de care coordonatelevârfurilor triunghiului să fie A(0, 0), B(x 2 , 0), C(x 3 , y 3 ). Obţinem imediat căM x 2 + kx 31 + k , ky 3 x 3N1 + k‹,1 + k , y 3 kx 2P1 + k‹,1 + k , 0‹,Q p + 1 · x2 + kx 3p 1 + k , p + 1 ky 3· Rp 1 + k‹, p + 1 x 3·p 1 + k − x 2p , p + 1 y 3·p 1 + k‹,S p + 1 kx 2·p 1 + k − x 3p , −y 3Atunci S ABC =p‹. 1 2 · |x 2y 3 |, iar S MNP = 1 2 · |∆ 2|,M y M 1unde ∆ 2 =x x N y N 1x P y P 1= x 2y 3(1 + k) 2 (k2 − k + 1), prin urmare S MNP = k2 − k + 1(k + 1) 2 ·S ABC . Întrucât k2 − k + 1(k + 1) 2 ≥ 1 4 , găsim prima inegalitate din enunţ. Apoi, S QRS =1Q y Q 12 · |∆ 3|, unde ∆ 3 =x x R y R 1x S y S 1= x 2y 2k2 3 − k + 1p (k + 1) 2 · (p + 1) 2 + p + 2. Însăk 2 − k + 1(k + 1) 2 ≥ 1 4 ⇒ k2 − k + 1(k + 1) 2 · (p + 1) 2 (p + 1)2(p + 3)2+ p + 2 ≥ + p + 2 = ≥ 0,44de unde rezultă cea de-a doua cerinţă a problemei.Soluţia 2 (Ioan Viorel Codreanu). În planul complex, considerăm puncteleA(a), B(b), C(c), M(m), N(n), P (z), Q(q), R(r) şi S(s). Avem: m = b + kc1 + k ,n = c + ka1 + k , z = a + kb m(1 + p) − a n(1 + p) − b z(1 + p) − c, q = , r = , s = .1 + k pppAplicând teorema lui Kiril Docev, obţinem că S MNP = 1 2 ·Im(nm+zn+mz) = . . . =k 2 − k + 1(k + 1) 2 S ABC şi S QRS = 1 2 · Im(rq + sr + qs) = . . . = 1 p 2 [(p + 1)2 S MNP + (p +2)S ABC ]. Concluzia rezultă ca în Soluţia 1.L210. Cercul A-exînscris triunghiului ABC este tangent prelungirilor laturilorAB şi AC în P , respectiv Q. Bisectoarele exterioare ale unghiurilor B şi C intersecteazădreapta P Q în S respectiv T . Demonstraţi că P Q ≤ ST + BC.Titu Zvonaru, ComăneştiSoluţie. Fie BE, E ∈ (AC), bisectoarea interioară a unghiului B şi {M} =EACS ∩ AB. Conform teoremei bisectoarei,EC = c a şi SCSM = BCBM . Se ştie căBP = p − c, CQ = p − b, AP = AQ = p,AP Q = 2p sin A şi fie x = P M. Folosind teorema2lui Menelaus în △AMC cu transversala Q − S − P,obţinem căQAQC · SCSM · P MP A = 1 ⇔ pp − b ·ax + (p − c) · x = 1 ⇔ ax = x(p − b) + (p − b)(p − c)p⇔ x(p − c) = (p − b)(p − c) ⇔ x = p − b. AtunciPBSTECQ67M


ABBM = p − c + p − b = a, deciBM = AE , adică BE∥SC. Cum BE ⊥ BS, deducemECcă BS ⊥ SC. Analog se arată că BT ⊥ T C.Observăm că m(ÕSBT ) = m(ÕSBC)−m(ÕT BC) = 90 ◦ − 1 2 m(ÒB)−(90 ◦ −m(ÕT CB)) =90 ◦ − 1 2 m(ÒB) − 1 2 m(ÒC) = 1 2 m(bA). Punctele S şi T sunt situate pe cercul de diametruBC; cu teorema sinusurilor, deducem că ST = a sin A . Atunci P Q ≤ ST + BC ⇔2(a + b + c) sin A 2 ≤ a sin A 2 + a ⇔ sin A 2 ≤ a , adevărat (inegalitatea lui Ballieu).b + cNotă. Soluţie corectă a dat dl Ioan Viorel Codreanu.sin 3 xL211. Arătaţi că(1 + sin 2 x) + cos 3 x2 (1 + cos 2 x) 2 ≤ 3√ 3, ∀x ∈ R.16Mihàly Bencze, BraşovSoluţie. Pentru t ∈ R, (1 + t 2 ) 2 − 16√ 3t = 1 9 9 (√ 3t − 1) 2 (3t 2 + 2 √ 3t + 9) ≥ 0.tRezultă că(1 + t 2 ) 2 ≤ 3√ 316 , prin urmare t 3(1 + t 2 ) 2 ≤ 3√ 3sin 3+P ≤P x16 ·t2 . Astfel,(1 + sin 2 x) + 2cos 3 x(1 + cos 2 x) 2 ≤ 3√ 3 sin 2 x+ 3√ 3 cos 2 x= 3√ 316 16 16 .L212. Demonstraţi că 3 ab(ab + c 2 ) 22 a 2 + b 2 (a 2 + c 2 )(b 2 + c 2 (sumele fiind)ciclice) pentru orice numere reale a, b, c printre care nu se găsesc două egale cu 0.Marian Tetiva, BârladSoluţie. Are loc identitateaP(a 2 − c 2 )(b 2 − c 2 )(a − b) 2 = (a − b) 2 (a − c) 2 (b − c) 2 .Atunci inegalitatea 0 ≤ (a − b) 2 (a − c) 2 (b − c) 2 se transcrie succesiv0≤X(a 2 + c 2 )(b 2 + c 2 )(a − b) 2 −2Xc 2 (a 2 + b 2 )(a − b) 2 ⇔2Xc 2 (a 2 + b 2 )(a − b) 2 ≤≤X(a 2 + c 2 )(b 2 + c 2 )(a − b) 2 ⇔ 2X(a 2 + b 2 )((a 2 + c 2 )(b 2 + c 2 ) − (ab + c 2 ) 2 ) ≤≤X(a 2 + c 2 )(b 2 + c 2 )(a 2 + b 2 − 2ab) ⇔ 3(a 2 + b 2 )(a 2 + c 2 )(b 2 + c 2 )++ 2Xab(a 2 + c 2 )(b 2 + c 2 ) ≤X(a 2 + b 2 )(ab + c 2 ) 2 ,de unde inegalitatea din enunţ se obţine prin împărţire cu 2(a 2 +b 2 )(a 2 +c 2 )(b 2 +c 2 ) >0. Egalitatea are loc dacă şi numai dacă (a − b) 2 (a − c) 2 (b − c) 2 = 0, adică a = b saua = c sau b = c.L213. Fie m 1 , . . . , m k numere naturale nenule şi α un număr iraţional.a) Arătaţi că există x 1 , . . . , x k ∈ N ∗ astfel încât [x 1α]m 1= . . . = [x kα]m k.b) Arătaţi că există y 1 , . . . , y k ∈ N ∗ astfel încât m 1 [y 1 α] = . . . = m k [y k α].Marian Tetiva, BârladSoluţie. Demonstrăm că, pentru x ∈ N ∗ şi y ∈ R cu x{y} < 1, avem că [xy] =x[y]. Într-adevăr, xy = x[y]+x{y}, cu x[y] ∈ Z şi x{y} ∈ (0, 1), conform ipotezei, decix{y} este chiar partea fracţionară a numărului xy, iar x[y] este partea sa întreagă.68


a) Fie n ∈ N ∗ care verifică {nα} < 1 m j⇔ {nα}m j < 1, ∀1 ≤ j ≤ k (există unastfel de număr, conform teoremei de densitate a lui Kronecker). Vom avea atunci că[m j nα] = m j [nα], ∀1 ≤ j ≤ k, deci pentru x j = m j · n, 1 ≤ j ≤ k, cerinţa se verifică.b) Fie M j = m 1 . . . m km j, 1 ≤ j ≤ k şi alegem N ∈ N ∗ astfel încât {Nα}


Probleme propuse 1Clasele primareP.226. Scrieţi vecinii numărului care rezultă din compunerea alăturată.(Clasa I )Mihaela Cucoranu, elevă, Iaşi1 22 5P.227. La plecare în vacanţă, trei elevi au convenit să-şi trimită felicitări:fiecare să trimită o singură felicitare la unul dintre ceilalţi doi. Este posibil caunul dintre elevi să primească felicitare de la elevul căruia el însuşi i-a trimis felicitare?(Clasa I )Lavinia Dascălu, elevă, IaşiP.228. Priviţi cu atenţie exerciţiul de mai jos:3 + 7 + + += 10020 60Calculaţi: a) + b) 100 - ⃝.(Clasa I )Ştefania Gavril, elevă, IaşiP.229. Într-o cutie sunt 17 bile albe şi 19 bile negre. Sorin ia la întâmplare 5 bile.Câte bile de fiecare culoare rămân în cutie?(Clasa a II-a)Inst. Maria Racu, IaşiP.230. Arătaţi că oricum am lua 6 numere din şirul 11, 12, 13, . . . , 20 există douăcare au suma 31.(Clasa a II-a)Mihaela Gâlcă, elevă, IaşiP.231. Asupra numerelor 10, 11, 12 şi 13 se efectuează operaţia următoare: numerelepare se înlocuiesc cu predecesoarele lor şi cele impare se înlocuiesc cu succesoarelelor. În al doilea pas se repetă această operaţie asupra rezultatului obţinut; secontinuă în acelaşi fel în paşii următori. Aflaţi de câte ori se repetă scrierea iniţială anumerelor între paşii 101 şi 230 ai şirului de operaţii.(Clasa a II-a)Paula Zaharia, elevă, IaşiP.232. Cei şase membri ai unei echipe care participă la un concurs de matematicăau vârste diferite, de cel puţin 10 ani şi cel mult 15 ani. În timpul concursului membriiechipei s-au aşezat la masă în ordinea vârstelor. Să se afle ce vârstă are fiecare ştiindcă Ioana este cea mai mică, Anca este cea mai mare, Bogdan se află lângă Ioana şinu se află lângă Bianca şi Andrei, iar Alexandra se află între doi băieţi.(Clasa a III-a)Constanţa Tudorache şi Nelu Tudorache, IaşiP.233. Dacă a × b = 441 şi b se împarte exact la a, calculaţi a şi b.(Clasa a III-a)Tatiana Ignat, elevă, Iaşi1 Se primesc soluţii până la data de 31 iunie 2012.70


P.234. Aflaţi numerele naturale a şi b astfel încât (a + 2) : (b + 1) = a.(Clasa a III-a)Codruţa Filip, elevă, IaşiP.235. O carte are rupte mai multe foi consecutive. Prima pagină, de pe primafoaie ruptă, are numărul 163, iar ultima pagină are numărul <strong>format</strong> din aceleaşi cifre.Pot fi împărţite foile rupte în grupe de câte 3?(Clasa a III-a)Andreea Bîzdîgă, elevă, IaşiP.236. În două cutii sunt mingi de tenis, în prima fiind de două ori mai multedecât în a doua. Dacă din prima cutie se scot 30 de mingi şi din a doua 20, atunciîn prima cutie rămân de 3 ori mai multe mingi decât în a doua. Câte mingi sunt înfiecare cutie?(Clasa a IV-a)Înv. Petru Miron, PaşcaniP.237. Într-o cameră sunt scaune cu 3 picioare şi cu 4 picioare. Când toatescaunele sunt ocupate, numărul picioarelor din cameră este 39. Câte scaune cu 4picioare sunt în cameră?(Clasa a IV-a)Inst. Laura Chirilă, IaşiP.238. Aflaţi numerele care se măresc de 11 ori prin adăugarea unei cifre diferitede zero la sfârşitul lor.(Clasa a IV-a)Nicoleta Cumpătă, elevă, IaşiP.239. Dreptunghiul alăturat reprezintă o grădină care este <strong>format</strong>ă din două aleişi patru straturi dreptunghiulare egale. Aflaţi dimensiunile grădiniiştiind că lăţimea fiecărei alei este de 2m, diferenţa dintre dimensiunileunui strat este de 1m, iar lungimea unui strat reprezintă 2 5 dinlungimea grădinii.(Clasa a IV-a)Petru Asaftei, IaşiClasa a V-aV.144. Aflaţi numerele naturale abc cu proprietatea că abc = a + 19b + 10c.Nicolae Ivăşchescu, CraiovaV.145. Demonstraţi că numărul A = 2 1 + 2 2 + 2 3 + . . . + 2 2012 nu este pătratperfect.Anca Chiriţescu, Ţigănaşi (Iaşi)V.146. Se consideră mulţimea A = {abc | a, c cifre pare, b cifră impară}. Determinaţicardinalul lui A şi suma elementelor lui A.Bogdan Chiriac, student, IaşiV.147. Putem aşeza pe un cerc numerele 1, 2, 3, . . . , 2012 astfel încât sumaoricăror patru numere consecutive să se dividă cu 5?Gheorghe Iurea, IaşiV.148. Arătaţi că 2371 + 26 · 3 2373 240 < 3371 + 124 · 5 2475 250 .Petru Asaftei, IaşiV.149. Demonstraţi că 6 n (n ∈ N) nu poate fi scris ca sumă de două pătrateperfecte nenule.Elena Iurea, Iaşi71


V.150. Despre un număr natural a 1 a 2 . . . a n spunem că este număr bun dacăexistă o infinitate de pătrate perfecte care au suma cifrelor egală cu a n−1 a n . Arătaţică 2012 nu este număr bun, însă 2013 este număr bun.Cristian Lazăr, IaşiClasa a VI-aVI.144. Fie p un număr prim impar. Arătaţi că există un singur număr naturalnenul k pentru care p 2 + k 2 este pătrat perfect.Marian Panţiruc, IaşiVI.145. Fie a ∈ N, a ≥ 2 şi mulţimea A = {x ∈ Z|a − a 2 + 1 ≤ x ≤ a + a 2 + 1}.Determinaţi cardinalul lui A şi suma elementelor din A.Ionel Nechifor, IaşiVI.146. Se pot împărţi numerele 1, 2, 3, . . . , 2012 în câteva submulţimi disjuncteastfel încât cel mai mare element al fiecărei submulţimi să fie egal cu produsul celorlalteelemente ale respectivei submulţimi?Mihai Crăciun, PaşcaniVI.147. Găsiţi două numere raţionale mai mari decât 40 al căror produs să fie2012, fiecare dintre ele având câte o infinitate de zecimale nenule.Cristian Lazăr, IaşiVI.148. Determinaţi fracţiile ireductibile a care se scriu sub formă zecimală cabfracţii periodice, cu zecimala de pe poziţia b egală cu b.Gabriel Popa, IaşiVI.149. Se consideră triunghiul ABC cu m(ÒB) = 3·m(ÒC). Fie M, N ∈ AC astfelîncâtÖABM ≡Ö<strong>MB</strong>N ≡ÕNBC şi AP ⊥ BN, cu P ∈ BN; notăm {I} = BM ∩ AP.Demonstraţi că NI este bisectoarea unghiuluiÕANB.Nicolae Ivăşchescu, CraiovaVI.150. Fie ABC un triunghi. Notăm cu D punctul de intersecţie dintre perpendicularaîn B pe BC şi mediatoarea laturii AB şi cu E punctul de intersecţie dintreperpendiculara în C pe BC şi mediatoarea laturii AC. Dacă α = m(ÕBAC), caculaţim(ÕDAE) în funcţie de α.Adrian Zanoschi, IaşiClasa a VII-aVII.144. Fie ABCD un trapez cu AB∥CD şi AB=3·CD. Dacă E şi F sunt simetricelepunctelor B şi A faţă de C, respectiv D, arătaţi că CDEF este paralelogram.Eugeniu Blăjuţ, BacăuVII.145. În triunghiul ABC, se consideră mediana AD şi bisectoarea CE. Notăm{P } = AD ∩ CE şi {F } = P B ∩ AC. Demonstraţi că triunghiul EF C este isoscel.Valentina Blendea şi Gheorghe Blendea, IaşiVII.146. Fie H ortocentrul triunghiului ascuţiunghic ABC şi D, E, F intersecţiiledreptelor AH, BH respectiv CH cu cercul circumscris triunghiului. Ştiind că patrulatereleHBDC, HCEA şi HAF B au ariile egale, arătaţi că △ABC este echilateral.Adriana Dragomir şi Lucian Dragomir, Oţelu-Roşu72


VII.147. Trapezul dreptunghic ABCD (AB∥CD, AD ⊥ AB) este circumscriscercului de centru O. Arătaţi că A ABCD < 1 2 (OB + OC)2 .Daniela Munteanu, IaşiVII.148. Rezolvaţi în numere întregi ecuaţia x(x + 4) = 5(3 y − 1).Neculai Stanciu, BuzăuVII.149. Fie n ∈ N, n ≥ 2 şi a = 2 n3 −n+2 , b = 5 8n3 −2n+2 . Arătaţi că produsula · b se poate scrie ca sumă de patru cuburi perfecte nenule.Constantin Dragomir, PiteştiVII.150. Fie x, y numere reale strict pozitive cu x > y. Demonstraţi că x 2 +y 2 >2Èxy · (x 2 − y 2 ) şi interpretaţi geometric rezultatul.Cosmin Manea şi Dragoş Petrică, PiteştiClasa a VIII-aVIII.144. Fie ABCDA ′ B ′ C ′ D ′ un cub şi V A ′ B ′ C ′ D ′ o piramidă patrulaterăregulată cu toate muchiile egale şi vârful V în exteriorul cubului. Aflaţi sinusulunghiului dintre dreapta A ′ C şi planul (V A ′ B ′ ).Mirela Marin, IaşiVIII.145. Fie a ∈ (1, ∞) şi x 1 , x 2 , . . . , x n numere reale astfel încât x 1 +x 2 +. . .+x n = a + n − 1 şi x 2 1 + x 2 2 + . . . + x 2 n = a 2 + n − 1. Determinaţi cea mai mare valoareposibilă a lui x n .Lucian Tuţescu, Craiova şi Dumitru Săvulescu, BucureştiVIII.146. Rezolvaţi în R 2 sistemul¨x 2 − xy = 348x 2 + 4xy(x + 1) 2 = (x + 1) 4 .Vasile Chiriac, BacăuVIII.147. Determinaţi bazele de numeraţie x ∈ N\{0, 1} pentru care numărulN = 11111 (x) este pătrat perfect.Cătălin Calistru, IaşiVIII.148. Stabiliţi câte submulţimi {a, b} ale mulţimii A = {1, 2, 3, . . . , 100} auproprietatea că a 3 + b 3 se divide cu 12.Dorel Luchian, IaşiVIII.149. Demonstraţi că abc(a + b + c) 2 ≤ 3(a 5 + b 5 + c 5 ), ∀a, b, c ∈ R ∗ +.Gheorghe Struţu şi Adrian Stan, BuzăuVIII.150. Determinaţi mulţimea A =§x ∈ Rxx 4 − x 3 + 1 ∈ Zª.Elena Iurea, IaşiClasa a IX-aIX.126. În triunghiul ABC notăm cu O centrul cercului circumscris şi cu O ′centrul cercului circumscris triunghiului median MNP . Atrătaţi că −−→ O ′ O = −−→ O ′ A +−−→O ′ B + −−→ O ′ C.Ion Pătraşcu, Craiova73


IX.127. Fie ABC un triunghi cu AB ≠ AC, I centrul cercului înscris, G centrulde greutate, {D} = AI ∩ BC şi {T } = IG ∩ BC. Demonstraţi că GD∥AT dacă şinumai dacă 3a = b + c. (A se vedea şi articolul din RecMat–2/2011, pag. 132-133.)Titu Zvonaru, ComăneştiIX.128. Fie n ∈ N ∗ şi x 1 , x 2 , . . . , x n numere reale pozitive cu suma egală cu n.Demonstraţi că nPi=1 x 2 i (x i + n) ≥ n 2 + n.Ion Nedelcu, Ploieşti şi Lucian Tuţescu, CraiovaIX.129. Fie a, b, c numere reale pozitive cu a ≤ b ≤ 24000 şi √ a + 2012 +√ √ a + bb + 2012 = 2 c + 2012. Determinaţi partea întreagă a numărului .cIonel Tudor, Călugăreni (Giurgiu)IX.130. Rezolvaţi în numere naturale ecuaţia 2 a + 1 = 3b 2 .Adrian Zanoschi, IaşiClasa a X-aX.126. Fie a, b, c ∈ R astfel încât ab + bc + ca ≥ 0. Demonstraţi că |a + bi| +|b + ci| + |c + ai| ≥È6(ab + bc + ca).Ovidiu Pop, Satu MareX.127. Dacă a, b, c ∈ (1, ∞), demonstraţi că are loc inegalitateaa(log a b )2log a ab + (log a a c )2log a ac + (log a bc) 2≥ 1.alog 2a bcD.M. Bătineţu-Giurgiu, BucureştiX.128. În raport cu reperul cartezian xOy se consideră punctul A(a, b), 0 < a < b.a) Arătaţi că există o infinitate de puncte B, cu ambele coordonate strict pozitive,pentru care min M∈Oy (MA + <strong>MB</strong>) = min N∈Ox (NA + NB).b) Expuneţi un procedeu de obţinere a punctelor B folosind doar rigla şi compasul.Cecilia Deaconescu, PiteştiX.129. Fie z 1 , z 2 , z 3 numere complexe distincte de modul 1. Arătaţi că(z 1 + z 2 ) 4 z 2 3 + (z 2 + z 3 ) 4 z 2 1 + (z 3 + z 1 ) 4 z 2 2 ≥ 3z 2 1z 2 2z 2 3.Florin Stănescu, GăeştiX.130. Fie α ∈ R\{kπ|k ∈ Z}; arătaţi că ecuaţia x 4 + 4x 3 + (4 − 4 sin α −2 sin 2 α)x 2 − (8 sin α + 4 sin 2 α)x + (4 sin 2 α + 4 sin 3 α + sin 4 α) = 0 are toate soluţiilereale.Ionel Tudor, Călugăreni (Giurgiu)Clasa a XI-aXI.126. Fie A ∈ M n (R) o matrice cu proprietatea că A 4 + (A + I n ) 4 = O n .Demonstraţi că matricea A 2 + A + I n este inversabilă.Dan Nedeianu, Drobeta Tr. Severin74


XI.127. Fie (x n ) n∈N ∗ un şir de numere din intervalul (0, 1) şi (F n ) n∈N ∗ şirul luiFibonacci (F 1 = F 2 = 1, F n+2 = F n+1 + F n , ∀n ∈ N ∗ ). Arătaţi căF kx k (1 − xnXk=12 k ) ≥ 3√ 32 (F n+2 − 1), ∀n ∈ N ∗ .D.M. Bătineţu-Giurgiu, Bucureştix nx n + a ,∀n ∈ N, unde a, x 0 ∈ (0, ∞) sunt date.Adrian Corduneanu, IaşiXI.128. Studiaţi convergenţa şirului (x n ) n∈N definit prin x n+1 =ÉnQx i = e n . Arătaţi că ex n ln 2 x 1 < x 1 x n < ex 1 ln 2 x n .i=1XI.129. Determinaţi cel mai mic număr real α pentru care tg x ≥ 4 sin x − α,∀x ∈h0, π (În legătură cu problema IX.109. din RecMat–1/2010.)2.Gabriel Popa, IaşiXI.130. Se consideră numerele reale 1 < x 1 < x 2 < . . . < x n < e 2 astfel încât√xi = nnPi=1√ e şi=ZMihai Haivas, IaşiClasa a XII-ax 2 − 1XII.126. Calculaţi I dx, x ∈ [1, 2].xÈ5x 2 − (x 2 − x + 1)2Constantin Dragomir, PiteştiXII.127. Calculaţi I =Z21 + x + 11/2 x‹e x− 1 x dx.Adrian Corduneanu, IaşiXII.128. Fie f : R → R o funcţie de două ori derivabilă, cu f ′′ (x) < 0, ∀x ∈[a, a + c]. Demonstraţi căZ2c0f(x)dx + 2cf(2a) ≤ 4Za+cf(x)dx.aMihai Haivas, Iaşi şi I.V. Maftei, BucureştiXII.129. Fie m, p ∈ N, cu m ≥ 2 şi p număr prim. Demonstraţi că există ungrup finit G care are p m2 elemente, în care fiecare element diferit de elementul neutruare ordinul p.Constantin Şcheau, PloieştiXII.130. Determinaţi perechile de polinoame de gradul doi, cu coeficienţii realişi unitare, ce verifică condiţia că rădăcinile unuia sunt coeficienţii celuilalt (se au învedere coeficienţii lui X şi X 0 ). Indicaţi polinoamele de acest fel care intră în perechecu ele însele.Temistocle Bîrsan, Iaşi75


Probleme pentru pregătirea concursurilorA. Nivel gimnazialG216. Într-un pătrat 3 × 3 se aşază numerele de la 1 la 9 astfel încât produsulnumerelor de pe linia k sau produsul numerelor de pe coloana k să fie pătrat perfect,pentru fiecare k ∈ {1, 2, 3}. Este posibil ca în centrul pătratului să se afle un numărimpar?Marius Mâinea, GăeştiG217. Pe tablă sunt desenate p pătrăţele, p ∈ N ∗ . Ionuţ colorează un pătrăţel,Ana colorează trei pătrăţele, apoi Ionuţ colorează cinci, Ana şapte ş.a.m.d. Pierdecopilul care nu mai are pe tablă suficiente pătrăţele de colorat atunci când îi vinerândul. Determinaţi numerele p pentru care câştigătorul jocului este Ionuţ şi stabiliţicâte pătrăţele i-ar rămâne de colorat Anei (în funcţie de p).Gheorghe Iurea, IaşiG218. Se consideră numerele reale a 1 , a 2 , . . . , a n (n ∈ N, n ≥ 2). Demonstraţi căexistă o submulţime A ⊆ {1, 2, . . . , n} cu proprietatea că |Xi∈Aa i | ≥ 1 4nXi=1|a i |.Radu Miron, elev, IaşiG219. Fie a, b, c numere nenule, a impar, b>c astfel încât a= 2bc şi (a, b, c) = 1.b − cArătaţi că abc este pătrat perfect.Neculai Stanciu, Buzău şi Titu Zvonaru, Comăneşti|{z}n oriG220. Determinaţi cifrele a cu proprietatea că există pătrate perfecte de forma2 aa . . . a 6.Adriana Dragomir şi Lucian Dragomir, Oţelu-RoşuG221. Determinaţi numerele naturale n pentru careA = 1681[ √ n 2 + n + 15] − 1[ √ n 2 + n + 16]∈N.Mircea Fianu, BucureştiG222. Rezolvaţi ecuaţia (x + 2) 3 = x(x 2 − 2) 5 , x ∈ (0, ∞).Dan Nedeianu, Drobeta Tr. SeverinG223. Pentru x, y, z ≥ 0, demonstraţi că are loc inegalitateaxy(x 2 − y 2 ) 2 + xz(x 2 − z 2 ) 2 + yz(y 2 − z 2 ) 2 ≥ 4(x − y) 2 (x − z) 2 (y − z) 2 .Marian Tetiva, BârladG224. Trapezul isoscel ABCD are baza mare AB şi diagonalele perpendiculareîn O. Paralela prin O la baze taie laturile neparalele BC şi AD în P , respectiv R.Punctul Q este simetricul lui P faţă de mijlocul lui BC. Dreapta RQ intersecteazăAC şi BD în punctele E, respectiv F . Demonstraţi că:76


a) RQ ⊥ AD şi RQ = AD;b) RE = F Q = CP şi P Q = EF .Claudiu-Ştefan Popa, IaşiG225. Lucian-Georges are o placă triunghiulară omogenă ABC de masă 40 şi obalanţă cu două talere. El doreşte să taie placa ABC după m drepte paralele cu BCastfel încât, folosind aceste plăci ca greutăţi dispuse pe talerele dorite ale balanţei, săpoată cântări orice obiect cu masa număr natural n, cu 1 ≤ n ≤ 40. Cum îl sfătuiţisă procedeze, astfel încât m să fie minim posibil?Dan Brânzei, IaşiB. Nivel licealL126. Tangentele unghiurilor unui triunghi ABC sunt numere raţionale. Arătaţică numerele E n = sin n A · sin n B · sin n C + cos n A · cos n B · cos n C sunt raţionale,oricare ar fi n ∈ N.Cătălin Calistru, IaşiL127. Măsurile unghiurilor B şi C ale triunghiului ABC sunt de 70 ◦ , respectiv30 ◦ . Pe latura AB se consideră punctele E şi F astfel încâtÕACE ≡ÕECF ≡ÕF CB.Fie AD înălţimea din A, D ∈ BC, iar {M} = AD ∩ CF. Demonstraţi că <strong>MB</strong> estebisectoarea unghiuluiÖDMF .Eugeniu Blăjuţ, BacăuL128. Fie ABC un triunghi isoscel cu AB = AC şi D un punct pe latura BC.Considerăm punctele E şi F pe laturile AB, respectiv AC astfel încât BD = DE şiCD = CF. Notăm {T } = BF ∩ CE. Arătaţi că patrulaterul BDT E este inscriptibildacă şi numai dacă patrulaterul DCF T este inscriptibil.Titu Zvonaru, ComăneştiL129. Fie date un triunghi ABC şi numerele naturale m ≥ n ≥ 1. Construiţi curigla şi compasul punctele A ′ din planul triunghiului pentru care triunghiul A ′ BC areperimetrul şi aria de m, respectiv n ori mai mare decât cele ale triunghiului ABC.Temistocle Bîrsan, IaşiL130. a) Fie n ∈ N, n ≥ 10. Arătaţi că există o infinitate de n-uple (x 1 , x 2 , . . . , x n ),cu x i ∈ (0, 1), ∀i = 1, n şi x i =nPi=1n 2 .b) Pentru un n-uplu ca la a), notăm E n = x 1x 2+1 − x 1 (1 − x 1 )(1 − x 2 ) + . . . +x n(1 − x 1 ) . . . (1 − x n ) . Arătaţi că 1se exprimă ca număr zecimal în care celE n + 1puţin primele 3 ·hnsunt zerouri.10izecimaleCecilia Deaconescu, PiteştiL131. Fie n un număr natural impar. Stabiliţi câte numere naturale nenule pau proprietatea că p 2 + n 2 este pătrat perfect şi determinaţi cel mai mare asemeneanumăr.Marian Panţiruc, Iaşi77


L132. Pentru a, b, c numere reale pozitive, demonstraţi inegalitateaa1b 2 + 1 c 2‹+b1c 2 + 1 a 2‹+c1a 2 + 1 b 2‹≥ 18a + b + c .XL133. Dacă a, b, c sunt numere reale pozitive cu a + b + c = 3, arătaţi căabab + a + b + 1 − b)9X(a 2ab + a + b ≤ 1.Florin Stănescu, GăeştiTitu Zvonaru, ComăneştiL134. Fie n şi k numere întregi pozitive. Demonstraţi identităţile:a)n−1Xj=1•jk˜=nhnki− k 2hnkihnb)ki+1;nXj=1•kÉnj˜=[ k√ n]Xq=1•nq k˜.Marian Tetiva, BârladL135. Fie A, B ∈ M n (R) şi X un vector nenul din R n astfel încât AX = O şiexistă Y ∈ R n pentru care AY = BX. Notăm cu A j matricea obţinută înlocuindcoloana j a matricei A cu coloana j a matricei B. Arătaţi că nPj=1 detA j = 0.Adrian Reisner, ParisTraining problems for mathematical contestsA. Junior highschool levelG216. The numbers from 1 to 9 are arranged on a square with 3 × 3 cells sothat the product of the numbers on row k or column k be a perfect square, for anyk ∈ {1, 2, 3}. It is possible to set a odd number in the central cell of the square?Marius Mâinea, GăeştiG217. A number of p small squares are drawn on the blackboard. Johnny coloursa square, Ann colours three squares, Johnny coulours five of them, Ann colours sevenones and so on. The child who has not sufficient squares to coulour when his/her turncomes on loses. Determine the number p for which the winner of the game is Johnnyand establish how many small squares would remain for Ann|Pto colour (as a functionof p ) .Gheorghe Iurea, IaşiG218. The real numbers a 1 , a 2 , . . . , a n (n ∈ N, n ≥ 2) are considered. Show thata subset A ⊆ {1, 2, . . . , n} exists with the property that a i | ≥ 1 | a i | .i∈A 4nPi=1Radu Miron, elev, IaşiG219. Let a, b, c be nonzero numbers, a odd and b > c, such that a = 2 b cb − c and(a, b, c) = 1. Show that a b c is a perfect square.Neculai Stanciu, Buzău and Titu Zvonaru, Comăneşti78


|{z}n timeG220. Determine the digits a with the property that perfect squares of the form2 aa . . . a 6 exist.Adriana Dragomir and Lucian Dragomir, Oţelu-RoşuG221. Determine the natural number n such thatA = 1681[ √ n 2 + n + 15] − 1[ √ n 2 + n + 16]∈N.Mircea Fianu, BucureştiG222. Solve the equation (x + 2) 3 = x (x 2 − 2) 5 , x ∈ (0, ∞).Dan Nedeianu, Drobeta Tr. SeverinG223. For x, y, z ≥ 0, show that the following inequality holds.x y (x 2 − y 2 ) 2 + x z (x 2 − z 2 ) 2 + y z (y 2 − z 2 ) 2 ≥ 4 (x − y) 2 (x − z) 2 ( y − z) 2 .Marian Tetiva, BârladG224. The isosceles trapezium ABCD has the larger base AB and its diagonalsare perpendicular at the point O. The parallel line through O to the bases intersectsthe non-parallel sides at P , respectively R . The point Q is the symmetric point ofP with respect to the mid-point of BC. The line RQ intersects AC and BD at thepoints E, respectively F . Prove thata) RQ ⊥ AD and RQ = AD;b) RE = F Q = CP and P Q = EF .Claudiu-Ştefan Popa, IaşiG225. Lucian-Georges has a homogeneous triangular plate ABC of mass = 40,and a pair of scales. He wants to cut the plate along m straight lines, parallel to BCsuch that − using these smaller plates as weights placed on the balance pans − hewould be able to weigh any object of mass = n, with 1 ≤ n ≤ 40. How would youadvise him to proceed so that m be the minimum possible?Dan Brânzei, IaşiB. Highschool LevelL126. The tangents of the angles of a triangle ABC are rational numbers. Showthat the numbers E n = sin n A · sin n B · sin n C + cos n A · cos n B · cos n C are rational,any would be n ∈ N.Cătălin Calistru, IaşiL127. The measures of the angles B and C of the triangle ABC are equal to70 ◦ , respectively 30 ◦ . The points E and F are considered on the side AB suchthatÕACE ≡ÕECF ≡ÕF CB. Let AD be the perpendicular from A, D ∈ BC, and{M} = AD ∩ CF. Show that <strong>MB</strong> is the bisector of angleÖDMF .Eugeniu Blăjuţ, BacăuL128. Let ABC be an isosceles triangle with AB = AC and D a point on theside BC. Let us consider the points E and F on the sides AB, respectively AC such79


that BD = DE and CD = CF. Denote {T } = BF ∩CE. Show that the quadrilateralBDT E can be inscribed in a circle if and only if the quadrilateral DCF T possesesa circumcircle, too.Titu Zvonaru, ComăneştiL129. Let be given a triangle ABC and the natural numbers m ≥ n ≥ 1. Build,with a rule and a compass, the points A ′ in the plane of the triangle such that A ′ BChas its perimeter and its area m times, respectively n times larger than those oftriangle ABC.Temistocle Bîrsan, IaşiL130. a) Let n ∈ N, n ≥ 10. Show that infiunitely many n-tuples (x 1 , x 2 , . . . , x n ),with x i ∈ (0, 1), ∀i = 1, n and nPi=1 x i = n 2 , exist.b) For an n-tuple as under a), denote E n = x 1+1 − x 1x n(1 − x 1 ) . . . (1 − x n ) . Show that 1E n + 1that at least 3 ·hn10iof its decimal digits are zeros.x 2(1 − x 1 )(1 − x 2 ) + . . . +can be expressed as a decimal number suchCecilia Deaconescu, PiteştiL131. Let n be an odd natural number. Establish how many nonzero naturalnumbers p have the property that p 2 +n 2 is a perfect square and determine the largestnumber with this property.Marian Panţiruc, IaşiL132. For the positive real numbers a, b, c prove the inequality1b 2 + 1 c 2‹+b1c 2 + 1 a 2‹+c(a1a 2 + 1 b 2‹≥18a + b + c .aXL133. If a, b, c are positive real numbers with a + b + c = 3, show thata9X ba b + a + b + 1− b) 2a b + a + b ≤ 1.Florin Stănescu, GăeştiTitu Zvonaru, ComăneştiL134. Let n and k be positive integer numbers. Prove the identities :a)n−1Xj=1•jk˜=nhnki− k 2hnkihnki+1; b)nXj=1•kÉnj˜=[ k√ n]Xq=1•nq k˜.Marian Tetiva, BârladL135. Let A, B ∈ M n (R) and X a nonzero vector in R n , such that AX = 0 andthere exists a Y ∈ R n such that AY = BX. We denote by A j the matrix obtained afterreplacing column j of matrix A by column j of matrix B. Show that nPj=1 detA j = 0.Adrian Reisner, Paris80


Pagina rezolvitorilorCRAIOVAColegiul Naţional ,,Fraţii Buzeşti”. Clasa a IV-a (înv. POPESCU Roxana).POGACI Alexandru-Andrei: P(216-221,225). Clasa a VI-a (prof. MĂRCULESCUMariana). GULIN Cătălin Vasile: P(224,225), V(138,139), VI.137. Clasa a X-a(prof. TUŢESCU Lucian). GOLEA Monica: VIII(141,143), IX.121, X(121-123).IAŞIŞcoala nr. 3 ,,Al. Vlahuţă”. Clasa a IV-a (inst. MAXIM Gabriela). DASCĂLULorena: P(218-222,224); NICA Daniel: P(218-222,224); ROBU Carmen: P(218-222,224); ŞERBĂNOIU Alexandru: P(218-222,224). Clasa a V-a (prof. MARINMirela). ENEA Codruţ: P(223-225), V(137,138,142); HUHU Paula: P(224,225),V(137,138,142); POPOVICI Teodor-Andrei: P(223-225); V(137,138,142); ROMILĂAndreea: P(223-225), V(137,138,142). Clasa a VI-a (prof. MARIN Mirela). CIO-COIU Alexandra: V(137,138,140-142); MARIN Marius: P(223-225), V.138, VI(137,139); VLAD Ioana: P.225, V(138,141,142), VI(137,139).Şcoala nr. 11 ,,Otilia Cazimir”. Clasa a IV-a (inst. MARDARE Carmen).OLENIUC Iulian: P(216-219,221,224,225).Şcoala nr. 26 ,,G. Coşbuc”. Clasa a III-a (inst. RACU Maria). CIOPEICĂSebastian-Andrei: P(216-219,221,225); GROSU Victor-Alessandru: P(216-219,221,222,225). LUCHIAN Maria-Clara: P(216-219,221,225); MANOLE Alexandra: P(216-219,221,225); PASNICU Cosmin-Constantin: P(216-219,221); POPESCU Andrei-Eduard: P(216-219,221,225); RĂILEANU Ana-Maria: P(216-219,221); RĂILEANURăzvan-Constantin: P(216-219,221,225); VASILE Raluca-Andreea: P(216-219,221,225).Colegiul ,,Naţional”. Clasa a V-a (prof. LAZĂR Cristian). POPA IoanaMaria: P(220-225), V(137-143), VI(138-139), G(206,207). Clasa a VI-a (prof.POPA Gabriel). AŞTEFANEI Cosmin: P(224,225), V(139,142), VI(137,139); ENELina: P.225, V(137,138,142), VI.137; OBADĂ Ştefan: V(137,138,142); VI(137,139);SLĂTINEANU Mara-Diana: P.225, V(138,139,142), VI(137,139). Clasa a VII-a(prof. POPA Gabriel). ANDONE Raluca Ioana: V(137,142), VI(137,143), VII(137,139); BACIU Alexandru: V(137-139,142), VI.137; BERBINSCHI Tudor: V(137-140,142), VI(139,142,143); BUDESCU Andrada: V(138,139), VII(138,141), G.206;COJOCARIU Victor: V(137-139,142), VI.137; DOMINTE Ştefan: VI(137,139-143),VII(138-140), VIII(138-140,143), G.206; GRIGORAŞ Andrei: V.141, VI(137,143),VII(138,141). Clasa a XII-a (prof. POPA Gabriel). CEUCĂ Răzvan: X(121-125),XI(121-123), XII(121-125).Colegiul Naţional ,,C. Negruzzi”. Clasa a V-a (prof. ZANOSCHI Adrian). ZA-HARIUC Vlad: P(224,225), V(137,141,142).Grupul Şcolar ,,Virgil Madgearu”. Clasa a X-a (prof. OLENIUC Claudia).ILIEF George Claudiu: IX(121,124,125), X.121, XI.125; MEIU Luminiţa: IX(121,124,125), X.121, XI.125; SÎRGHI Nicoleta: IX(121,124,125), X.121, XI.125; SOFRONEADaniela-Petronela: IX(121,125), X(121,122), XI.125.81


PAŞCANIColegiul Naţional ,,Mihail Sadoveanu”. Clasa a VI-a (prof. MARCOVSCHIIonica). BUZATU Andrea: V(197-142), VI(137,139).ŢIGĂNAŞI (IAŞI)Şcoala cu clasele I-VIII ,,M. Kogălniceanu”. Clasa a II-a (înv. SAMSON Daniel-Mihai). DUCA Daria-Călina-Stela: P(216-221). Clasa a III-a (prof. înv. primarBĂDIŢĂ Aurica). DUCA Anamaria: P(218-222,224); DUCA Ema Ştefania: P(218-222,224); PAŞCANU Cosmin: P(218-222,224); SANDU Marta: P(218-222,224).Elevi rezolvitori premiaţiColegiul Naţional ,,Fraţii Buzeşti”, Craiova1. GOLEA Monica (cl. a X-a): 1/2011(5pb), 2/2011(6pb), 1/2012(6pb).Şcoala nr. 3 ,,Al. Vlahuţă”, Iaşi2. DASCĂLU Lorena (cl. a IV-a): 1/2011(5pb), 2/2011(6pb), 1/2012(6pb).3. ROBU Carmen (cl. a IV-a): 1/2011(5pb), 2/2011(6pb), 1/2012(6pb).4. ŞERBĂNOIU Alexandru (cl. a IV-a): 1/2011(5pb), 2/2011(6pb), 1/2012(6pb).5. CIOCOIU Alexandra (cl. a VI-a): 1/2011(5pb), 2/2011(5pb), 1/2012(5pb).6. MARIN Marius (cl. a VI-a): 1/2011(5pb), 2/2011(5pb), 1/2012(6pb).Şcoala nr. 11 ,,Otilia Cazimir”, Iaşi7. OLENIUC Iulian (cl. a IV-a): 1/2011(5pb), 2/2011(7pb), 1/2012(7pb).Şcoala nr. 26 ,,G. Coşbuc”, Iaşi8. VASILE Raluca-Andreea (cl. a III-a): 2/2010(5pb), 2/2011(5pb); 1/2012(6pb).Colegiul ,,Naţional”, Iaşi9. DOMINTE Ştefan (cl. a VII-a): 2/2010(11pb), 2/2011(21pb), 1/2012(14pb).10. CEUCĂ Răzvan (cl. a XII-a): 2/2010(6pb), 2/2011(15pb), 1/2012(13pb).82


RecenzieD. Brânzei şi Al. Negrescu - PROBLEME DE PIVOTARE(Colecţia ,,Recreaţii Matematice” – Nr. 1 )Motto: ,,Carte frumoasă, cinste cui te-a scrisÎncet gândită, gingaş cumpănită;Eşti ca o floare, anume înflorităMâinilor mele, care te-au deschis.”(Tudor Arghezi - Ex libris)În luna august a anului 2011, în ambianţa distinsă a Taberei Naţionale de Matematicăde la Muncel, a fost lansată o carte-eveniment: Probleme de pivotarede Dan Brânzei şi Alexandru Negrescu (al doilea ,,a creditat calculatorul şiprograme de tip Corel sau Sketchpad pentru a convinge figurile să intre în text”,după mărturia autorilor), prima din Colecţia ,,Recreaţii Matematice” apărutăla editura cu acelaşi nume.Cartea se înscrie în seria celor mai frumoase cărţi de geometrie sintetică, alăturide Culegerea de probleme de geometrie a lui Gheorghe Ţiţeica şi de minunataGeometrie a triunghiului de Traian Lalescu. Surpriza pe care o are cititorul este căasupra acestui domeniu ce pare aproape epuizat chiar de grecii antici, poate găsi operspectivă nouă, neaşteptată.Geometria e de la Dumnezeu; de aceea ea nu se învaţă, ci se descoperă. Frumuseţeaşi măreţia sa şi se dezvăluie pe măsură ce o cunoşti şi cu cât o cunoşti mai bine înţelegică ceea ce vezi e doar o parte din ţesătura ei spectaculoasă de conexiuni; şi te provoacă,astfel, să descoperi mereu altele.Sutem preveniţi în prefaţă că e o carte adresată celor care ştiu geometrie. Euaş spune că a fost scrisă cu dragoste de un geometru adevărat pentru discipoli, maitineri sau mai bătrâni, care iubesc geometria. Celor din urmă le dăruieşte ,,un ocheanmiraculos”, prin care privind, regăsesc punctele, dreptele, triunghiurile, relaţiile şimişcările planului, toate plecând de la un triunghi şi un punct, sau chiar de la unpunct şi o ,,formă”, în căutarea altor triunghiuri. Celor tineri le oferă, în plus, ometodă de lucru în geometrie.Un critic literar ar încadra cartea la genul suprarealist, pentru modul de prezentare.Dar ea este dublată de stilul metodic inconfundabil care ne-ar fi lăsat să ghicim autorulpe toţi cei care am găsit mereu, în lucrările sale, nu doar probleme frumoase şiprofunde, ci şi răspunsuri limpezi şi sistematice la întrebări pe care ni le-am pus oricare ne-au fost puse. Recunoaştem ,,crezul” său în abordarea geometriei: construcţia,mai întâi tatonată, apoi din ce în ce mai precisă, pe măsura desfăşurării argumentaţieice deschide calea demonstraţiei, singura capabilă să dezvăluie întreaga complexitatea configuraţiei respective.Cartea e organizată în patru părţi: Etalare, Explicaţii, Soluţii, Probleme deconcurs şi cuprinde, în final, Anexe. Am avertizat, însă, că e genul suprarealist; nuse parcurge liniar, ci de la Etalare mergi mereu la Explicaţii şi, să o recunoaştem, laSoluţii (chiar dacă ai rezolvat problema, e bine să verifici dacă nu ţi-a ,,scăpat” vreun83


detaliu). Primele două părţi sunt presărate cu ,,PFI-uri” (pauze de fortificare intelectuală);mai exact, sunt probleme ce pot să apară în diferite etape ale demonstraţiilor,ori care întregesc un rezultat obţinut şi care îţi poartă paşii prin întreg universulsurprinzător al geometriei triunghiului.Etalarea începe cu problema construcţiei unui triunghi asemenea şi la fel orientatcu un triunghi dat, înscris în alt triunghi dat. Găsirea unei soluţii arată mai multemoduri de a obţine o infinitate şi sugerează o nouă problemă: aceea de a circumscrieunui triunghi triunghiuri asemenea cu un triunghi dat. Această ultimă cercetareconduce la descoperirea, la intersecţia unor cercuri, a unui punct, pivot, ce dă şititlul cărţii: un patrulater complet şi punctul de concurenţă al celor patru triunghiuriale sale – punctul lui Miquel. Citez concluzia şi îndemnul maestrului de la finalulEtalării: ,,ne place să credem că am pus la îndemâna cititorului un ochean miraculos.Aruncaţi o privire. Odată cu △ABC vedeţi interiorul circum-cercului său O.Concentraţi-vă privirea şi mai ales gândirea asupra unui punct P din această imagine!Nu este doar un punct, ci esenţa formelor de triunghi”. Apoi PFI.13 invită cititorulsă găsească şi alte procedee de a executa ,,hărţi” în care fiecare punct să reprezinte oformă de triunghi.Dar învăţarea se realizează prin exemplu (Einstein întăreşte afirmaţia spunândcă învăţarea este exemplu). Astfel, ultimul capitol ,,exemplifică” utilizarea metodeipivotului în rezolvarea unor probleme de concurs; aşa cum ne aşteptam soluţiile suntclare, exhaustive, însoţite de precizări şi comentarii (unele critice). Una dintre problemee un răspuns posibil la provocarea din finalul primului capitol.În prefaţa autorilor este invocată (cu modestie şi autoironie) ,,utilitatea” acesteilucrări. Revenind la destinatarii săi, cred că cele de mai sus o recomandă cu prisosinţătinerilor discipoli. Iar ,,seniorii”, pătrunşi de înţelepciunea Micului Prinţ, vor spuneca şi el: ,,C ′ est utile parce que c ′ est beau! ”A fost subiectul, pe scurt, al unui ,,film de artă”. Dar frumuseţea unui film deartă nu se poate ,,povesti”, căci fiecare detaliu îi dă frumuseţe. Aşadar, trebuie săaveţi în bibliotecă, la loc de cinste, această carte şi să o parcurgeţi pas cu pas: ea văva ,,îmblânzi”, precum Micul Prinţ pe vulpea lui Antoine de Saint-Exupéry.Prof. Mihaela CIANGAColegiul Naţional ,,C. Negruzzi”, IaşiPrimul număr al Colecţiei ,,Recreaţii Matematice”,1. D. Brânzei, Al. Negrescu – Probleme de pivotare,poate fi procurat printr-o simplă cerere la adresa: t birsan@yahoo.com şi indicareaadresei poştale proprii. Cartea va fi trimisă cu plata ramburs la adresa indicată contrasumei de 25 lei (inclusiv taxe poştale).84


<strong>Revista</strong> semestrială RECREAŢII <strong>MATEMATICE</strong> este editată deASOCIAŢIA “RECREAŢII <strong>MATEMATICE</strong>”. Apare la datele de 1 martie şi1 septembrie şi se adresează elevilor, profesorilor, studenţilor şi tuturor celorpasionaţi de matematica elementară.În atenţia tuturor colaboratorilorMaterialele trimise redacţiei spre publicare (note şi articole, chestiuni demetodică, probleme propuse etc.) trebuie prezentate îngrijit, clar şi concis; eletrebuie să prezinte interes pentru un cerc cât mai larg de cititori. Se recomandă catextele să nu depăşească patru pagini. Evident, ele trebuie să fie originale şi sănu fi apărut sau să fi fost trimise spre publicare altor reviste. Rugăm ca materialeletehnoredactate să fie însoţite de fişierele lor (trimise la adresa de mai jos).Problemele destinate rubricilor: Probleme propuse şi Probleme pentrupregătirea concursurilor vor fi redactate pe foi separate cu enunţ şi demonstraţie/rezolvare(câte una pe fiecare foaie) şi vor fi însoţite de numele autorului, şcoalaşi localitatea unde lucrează/învaţă.Redacţia va decide asupra oportunităţii publicării materialelor primite.În atenţia elevilorNumele elevilor ce vor trimite redacţiei soluţii corecte la problemele dinrubricile de Probleme propuse şi Probleme pentru pregatirea concursurilorvor fi menţionate în Pagina rezolvitorilor. Elevii vor ţine seama de regulile:1. Pot trimite soluţii la minimum cinci probleme propuse în ultimul numărapărut al revistei. Pe o foaie va fi redactată soluţia unei singure probleme şivor fi menţionate datele: numele şi prenumele, clasa, profesorul, şcoala şilocalitatea.2. Elevii din clasele VI-XII au dreptul să trimită soluţii la problemelepropuse pentru clasa lor, pentru orice clasă mai mare, din două clase mai mici şiimediat anterioare. Elevii din clasa a V-a pot trimite soluţii la problemele propusepentru clasele a IV-a, a V-a şi orice clasă mai mare, iar elevii claselor I-IV pottrimite soluţii la problemele propuse pentru oricare din clasele primare şi orice clasămai mare. Orice elev poate trimite soluţii la problemele de concurs (tip G şi L).3. Oricine poate trimite soluţii la problemele de tip L; la publicare, soluţiileacestora vor fi urmate de numele tuturor rezolvitorilor.4. Plicul cu probleme rezolvate se va trimite prin poştă (sau va fi adusdirect) la adresa Redacţiei:Prof. dr. Temistocle BîrsanStr. Aurora, nr. 3, sc. D, ap. 6,700 474, IaşiJud. IAŞIE-mail: t_birsan@yahoo.com


CUPRINSÉVARISTE GALOIS (1811 - 1832).........................................................................................1Profesorul ADOLF HAIMOVICI – centenarul naşterii sale (T. Bîrsan)............................... 3ARTICOLE ŞI NOTEM. TETIVA – O teoremă de reprezentare (II) ......................................................................... 5F. STĂNESCU – Aplicaţii ale numerelor complexe în geometria triunghiului..................... 11C.-L. BEJAN – Extinderea unui rezultat al lui Johann Hudde............................................... 14N. STANCIU, T. ZVONARU – O condiţie de existenţă a triunghiurilor de arie şiperimetru date.....................................................................................................16M. DRĂGAN, M. HAIVAS, I.V. MAFTEI – O demonstraţie geometricăa inegalităţii lui Blundon..................................................................................20T. BÎRSAN – Dubla inegalitate a lui Blundon revizitată.........................................................22NOTA ELEVULUIR. MIRON – Asupra unei probleme de extrem.......................................................................25CORESPONDENŢEA. REISNER – Une classe spéciale de matrices carrées........................................................ 27Concursul Internaţional de Matematică „Vladimir Andrunachievici”, ed. II, Chişinău, 2012...32CUM CONCEPEM... CUM REZOLVĂMT. BÎRSAN – Condiţii de simetrie relativ la punctele O, H, G, I........................................... 34CHESTIUNI COMPLEMENTARE MANUALELORG. HĂVÂRNEANU – Aplicaţii ale teoremei lui Dirichlet la calcul de limite....................... 38ŞCOLI ŞI DASCĂLIG. SĂNDULESCU – Colegiul Naţional „M. Eminescu” din Iaşi (1865 - 2012) –147 de ani de tradiţie şi excelenţă în educaţie .................................................. 41CONCURSURI ŞI EXAMENEConcursul „Recreaţii Matematice”, ed. a IX-a, Muncel, 2011............................................... 45Concursul interjudeţean „Speranţe Olimpice”, ed. a XI-a, Paşcani, 2011................................ 47PROBLEME ŞI SOLUŢIISoluţiile problemelor propuse în nr. 2/2011.......................................................................... 49Soluţiile problemelor pentru pregătirea concursurilor propuse în nr. 2/2011...................... 62Probleme propuse..................................................................................................................... 70Probleme pentru pregătirea concursurilor .............................................................................. 76Training problems for mathematical contests ....................................................................... 78Pagina rezolvitorilor .............................................................................................................. 81Elevi rezolvitori premiaţi........................................................................................................ 82RECENZIE: M. CIANGA – D. Brânzei, Al. Negrescu – Probleme de pivotare..................83ISSN 1582 – 17658 lei

Hooray! Your file is uploaded and ready to be published.

Saved successfully!

Ooh no, something went wrong!